Вы находитесь на странице: 1из 221

Pgina del Colegio de Matemticas de la ENP-UNAM Conjuntos Autor: Dr.

Jos Manuel Becerra Espinosa


1

a
e
i
o
u
V
CONJUNTOS

UNIDAD I


I.1 DEFINICIN DE CONJUNTO

Un conjunto es un grupo de elementos u objetos especificados en tal forma que se puede afirmar con
certeza si cualquier objeto dado pertenece o no a la agrupacin. Para denotar a los conjuntos, se usan
letras maysculas.

Cuando un elemento
1
x pertenece a un conjunto A se expresa de forma simblica como: A x
1
. En
caso de que un elemento
1
y no pertenezca a este mismo conjunto se utiliza la notacin: A y
1


Existen cuatro formas de enunciar a los conjuntos:

1) Por extensin o enumeracin: los elementos son encerrados entre llaves y separados por
comas. Es decir, el conjunto se describe listando todos sus elementos entre llaves.

2) Por comprensin: los elementos se determinan a travs de una condicin que se establece
entre llaves. En este caso se emplea el smbolo | que significa tal que". En forma simblica es:

( ) { } { }
n
x , , x , x , x x P x A = =
3 2 1


que significa que el conjunto A es el conjunto de todos los elementos x tales que la condicin ( ) x P es
verdadera, como
3 2 1
x , x , x , etc
1
.

3) Diagramas de Venn: son regiones cerradas que sirven para visualizar el contenido de un
conjunto o las relaciones entre conjuntos
2
.

4) Por descripcin verbal: Es un enunciado que describe la caracterstica que es comn para los
elementos.

Ejemplo.
Dada la descripcin verbal el conjunto de las letras vocales, expresarlo por extensin, comprensin y
por diagrama de Venn.

Solucin.
Por extensin: { } u , o , i , e , a V =
Por comprensin: { } vocal una es x x V =
Por diagrama de Venn:






1
La notacin ( ) x P no representa un producto, es una condicin que deben satisfacer los elementos para pertenecer a un
conjunto.
2
En el caso particular de que un conjunto tenga un slo elemento numrico, a menos de que se haga la distincin, no representa el
nmero de elementos que posee el conjunto.
Pgina del Colegio de Matemticas de la ENP-UNAM Conjuntos Autor: Dr. Jos Manuel Becerra Espinosa
2

Ejemplo.
Expresar de las tres formas al conjunto de los planetas del sistema solar.

Solucin.
Por extensin: { } Plutn , Neptuno , Urano , Saturno , Jpiter , Marte , Tierra , Venus , Mercurio P =
Por comprensin: { } solar sistema del planeta un es x x P =
Por diagrama de Venn:

Mercurio
Venus
Tierra
Marte
Saturno
Urano
Neptuno
Jpiter
Plutn
P



Si cada elemento de un conjunto A es tambin un elemento del conjunto B , se dice que A es un
subconjunto de B . La notacin B A significa que A est incluido en B y se lee: A es subconjunto
de B o A est contenido en B .

Si no todos los elementos de un conjunto A son elementos del conjunto B , se dice que A no es
subconjunto de B . En este caso la notacin B A significa que A no es un subconjunto de B .

Grficamente, esto es:





En los ejemplos anteriores, si { } o , e , a F = es el conjunto de las vocales fuertes y
{ } Venus , Mercurio S = es el conjunto de planetas que no poseen satlites, entonces se cumple que:
V F y que P S . De la misma forma, ntese como: P F , V S , S F y F S .

La cardinalidad de un conjunto se define como el nmero de elementos que posee. Se denota por medio
de los smbolos o # .
De los conjuntos anteriores: ( ) 5 = V , ( ) 3 = F , ( ) 9 = P y ( ) 2 = S .
A
B
A B
B A

A B
B A

A B
B A

A
B
A
B
Pgina del Colegio de Matemticas de la ENP-UNAM Conjuntos Autor: Dr. Jos Manuel Becerra Espinosa
3

I.2 CONJUNTOS CON NOMBRES ESPECFICOS

Un conjunto vaco o nulo es aquel que no posee elementos. Se denota por: o bien por { }. El
conjunto vaco siempre forma parte de otro, as que es subconjunto de cualquier conjunto.

Ejemplos.
{ } actualidad la en viven que s dinosaurio los son x x =
{ } { } aos de mayores bres hom los son x x 300 =
{ } cero que menores positivos nmeros son x x =

Un conjunto universal es aquel que contiene a todos los elementos bajo consideracin. Se denota por
U . Grficamente se le representar mediante un rectngulo.

Ejemplos.
{ } { } domingo , sbado , viernes , jueves , mircoles , martes , lunes semana la de das los son x x U = =
{ } { } viernes , jueves , mircoles , martes , lunes inglesa semana la de das los son x x A = =
{ } { } domingo , sbado semana de fin del das los son x x B = =
{ } { } sbado , jueves , martes , lunes letras siete de menos con semana la de das los son x x C = =
Ntese cmo: U C , U B , U A

Un conjunto finito es aquel cuyos elementos pueden ser contados.

Ejemplos.
{ } junio de mes del da un de nmero el es x x J =
{ } 4
2
= = x x K
{ } Mxico de ciudad la en autos de cantidad la es x x L =

Un conjunto infinito es aquel cuyos elementos no pueden ser contados, es decir, su cardinalidad no
est definida.

Ejemplos.
{ } = , , , , , , N 11 9 7 5 3 1
{ } = , , , , , , M 12 10 8 6 4 2
{ } lnea una en puntos de cantidad la es x x Q =

Dos conjuntos son iguales si tienen exactamente los mismos elementos. Se denota por el smbolo = .

Ejemplo.
{ } 0 9 8 7 6 5 4 3 2 1 , , , , , , , , , R =
{ } dgito un es x x S =
S R =

Dos conjuntos son desiguales si por lo menos difieren en un elemento, es decir, si no tienen
exactamente los mismos elementos. Se denota por el smbolo .

Pgina del Colegio de Matemticas de la ENP-UNAM Conjuntos Autor: Dr. Jos Manuel Becerra Espinosa
4

Ejemplo.
{ } 9
2
= = x x D
{ } 2 2, E =
E D

Dos conjuntos son equivalentes si tienen la misma cantidad de elementos, es decir, si poseen la
misma cardinalidad. Se denota por el smbolo .

Ejemplos.
{ } ao del estaciones las son x x W =
{ } cardinal punto un es x x Z =
( ) 4 = W
( ) 4 = Z
Z W

Cuando los conjuntos son equivalentes existe una correspondencia uno a uno o biunvoca. Esto significa
que se puede establecer una relacin que asocie a cada elemento del primer conjunto con un nico
elemento del segundo conjunto sin que sobren elementos en ningn conjunto.

En el ejemplo anterior:

Primavera
W Z
Verano
Otoo
Invierno
Norte
Sur
Este
Oeste




I.3 OPERACIONES CON CONJUNTOS

La unin de los conjuntos A y B es el conjunto de todos los elementos de A con todos los
elementos de B sin repetir ninguno y se denota como B AU . Esto es:

{ } B x o A x x B A = U

Grficamente:
Pgina del Colegio de Matemticas de la ENP-UNAM Conjuntos Autor: Dr. Jos Manuel Becerra Espinosa
5

B AU
A B
U


Ejemplo.
{ } sanda , manzana , naranja , uva , ciruela , mango A =
{ } pltano , sanda , naranja , uva , meln , durazno B =
{ } o tan pl , meln , durazno , sanda , manzana , naranja , uva , ciruela , mango B A = U

La interseccin de los conjuntos A y B es el conjunto de los elementos de A que tambin
pertenecen a B y se denota como B AI . Esto es:

{ } B x y A x x B A = I

Grficamente:

B A I
A B
U


Ejemplo.
{ } sanda , manzana , naranja , uva , ciruela , mango A =
{ } pltano , sanda , naranja , uva , meln , durazno B =
{ } sanda , naranja , uva B A = I

Pgina del Colegio de Matemticas de la ENP-UNAM Conjuntos Autor: Dr. Jos Manuel Becerra Espinosa
6

Dos conjuntos son ajenos o disjuntos cuando su interseccin es el conjunto vaco, es decir, que no tienen
nada en comn. Por ejemplo:
{ } sanda , manzana , naranja , uva , ciruela , mango A =
{ } cereza , mandarina , pera , fresa limn, E =
= E AI

El complemento del conjunto A con respecto al conjunto universal U es el conjunto de todos los
elementos de U que no estn en A y se denota como ' A . Esto es:

{ } A x U x ' A =

Grficamente:

U
' A
A


Ejemplo.
{ } pltano , meln limn, , durazno , sanda , manzana , cereza , naranja , pera , uva , ciruela , kiwi , mango U =
{ } sanda , manzana , naranja , uva , ciruela , mango A =
{ } pltano , meln limn, , durazno , cereza , pera , kiwi ' A =

En este ejemplo se puede notar como ( ) ( ) ( ) U ' A A = +

De esta definicin, se puede advertir que se cumplen las siguientes expresiones:

( ) A ' ' A =
U ' =
= ' U

La diferencia de los conjuntos A y B (en ese orden) es el conjunto de los elementos que
pertenecen a A y no pertenecen a B y se denota como B A . Esto es:

{ } B x y A x x B A =

Grficamente:
Pgina del Colegio de Matemticas de la ENP-UNAM Conjuntos Autor: Dr. Jos Manuel Becerra Espinosa
7

U
B A
A B


Ejemplo.
{ } sanda , manzana , naranja , uva , ciruela , mango A =
{ } pltano , sanda , naranja , uva , meln , durazno B =
{ } manzana , ciruela , mango B A =
{ } pltano , meln , durazno A B =

Se puede advertir como A B B A .

Del diagrama de Venn anterior se deducen las siguientes expresiones:

' B A B A I =
B A : s slo y s , B A =
B A : s slo y s , A B B A = =
= = B A : s slo y s , A B A I
( ) A B A
A A =
' A ' B B A =
Los conjuntos A B , B A , B A I son mutuamente ajenos (su interseccin es el conjunto vaco).

Ejemplo.
Sean los conjuntos:
{ } n , m , l , k , j , i , h , g , f , e , d , c , b , a U =
{ } n , l , k , h , g , e , d , a A =
{ } m , l , k , g , f , c , a B =
Obtener:
a) B A U b)
B A I
c) ' A
d) ' B e) B A f) A B
g) B ' A U h) ' B AI i) ' B ' A I
j) ' B ' A k) ( )' B AU l) ( )' B A I

Pgina del Colegio de Matemticas de la ENP-UNAM Conjuntos Autor: Dr. Jos Manuel Becerra Espinosa
8

Solucin.
a) { } n , m , l , k , h , g , f , e , d , c , a B A = U b) { } l , k , g , a B A = I
c) { } m , j , i , f , c , b ' A = d) { } n , j , i , h , e , d , b ' B =
e) { } n , h , e , d B A = f) { } m , f , c A B =
g) { } m , l , k , j , i , g , f , c , b , a B ' A = U h) { } n , h , e , d ' B A = I
i) { } j , i , b ' B ' A = I j) { } m , f , c ' B ' A =
k) ( ) { } j , i , b ' B A = U
l) ( ) { } n , m , j , i , h , f , e , d , c , b ' B A = I

De acuerdo con las definiciones de unin, complemento y diferencia, se puede establecer que sus
respectivas cardinalidades se pueden obtener a travs de:

( ) ( ) ( ) ( ) B A B A B A I U + =
( ) ( ) ( ) A U ' A =
( ) ( ) ( ) B A A B A I =

Ejemplo.
En una unidad habitacional viven 120 familias y se sabe que 70 de ellas tienen automvil, que 30
poseen un reproductor de DVD y que 17 tienen ambas cosas. Se desea conocer: a) cuntas familias
tienen exclusivamente automvil?, b) cuntas familias son dueos exclusivamente de un reproductor
DVD, c) cuntas familias son propietarias de un automvil o de un reproductor DVD?, y d) cuntas
familias no poseen ni automvil ni reproductor DVD?

Solucin.
Identificando los datos por su cardinalidad:
Nmero de familias del conjunto universal, ( ) 120 = U
Nmero de familias con automvil, ( ) 70 = A
Nmero de familias con reproductor DVD, ( ) 30 = D
Nmero de familias con automvil y con reproductor DVD, ( ) 17 = D A I

U
A
D
17 30 17 70
17
( )' D AU


Del diagrama en donde se muestran el nmero de elementos de los conjuntos se aprecia que:
Pgina del Colegio de Matemticas de la ENP-UNAM Conjuntos Autor: Dr. Jos Manuel Becerra Espinosa
9

a) El nmero de familias que exclusivamente tienen automvil es:
( ) ( ) 53 17 70 = = D A A I
b) El nmero de familias que son dueos exclusivamente de un reproductor DVD es:
( ) ( ) 13 17 30 = = D A D I
c) El nmero de familias que son propietarias de un automvil o de un reproductor DVD es: ( ) B A U ,
as que: ( ) ( ) ( ) ( ) 83 17 30 70 = + = + = D A D A D A I U
d) El nmero de familias que no poseen ni un automvil ni un reproductor DVD es: ( )' B AU , por lo que:
( ) ( ) ( ) 37 83 120 = = = D A U ' D A U U


I.4 PROPIEDADES DE LOS CONJUNTOS

Sean los conjuntos C , B , A dentro del universo U . Las seis propiedades que rigen las operaciones con
esos conjuntos son las siguientes:

1. Propiedades de identidad:

A A = U
U U A = U
A U A = I
= I A

2. Propiedades de idempotencia:

A A A = U
A A A = I

3. Propiedades de complemento:

U ' A A = U
= ' A AI

4. Propiedades asociativas:

( ) ( ) C B A C B A U U U U =
( ) ( ) C B A C B A I I I I =

5. Propiedades conmutativas

A B B A U U =
A B B A I I =

6. Propiedades distributivas

( ) ( ) ( ) C A B A C B A U I U I U =
( ) ( ) ( ) C A B A C B A I U I U I =
Pgina del Colegio de Matemticas de la ENP-UNAM Conjuntos Autor: Dr. Jos Manuel Becerra Espinosa
10

I.5 LEYES DE DMORGAN

Estas leyes establecen los complementos de la unin e interseccin entre conjuntos:

Primera ley. El complemento de la unin de dos conjuntos es la interseccin de sus complementos.

( ) ' B ' A ' B A I U =

( )' B AU ' B ' A I
=
U U
A B A B


En el diagrama de la izquierda, B AU viene dada por la regin en blanco y ( )' B AU est representado
por el rea sombreada verticalmente. Por su parte en el diagrama de la derecha, ' A es la regin
sombreada horizontalmente, ' B es el rea sombreada verticalmente, por lo que ' B ' A I est
representado por la superficie cuadriculada. Las regiones resultantes son iguales.

Segunda ley. El complemento de la interseccin de dos conjuntos es la unin de sus complementos:

( ) ' B ' A ' B A U I =

( )' B AI ' B ' A U
=
U U
A B A B

Pgina del Colegio de Matemticas de la ENP-UNAM Conjuntos Autor: Dr. Jos Manuel Becerra Espinosa
11

En el diagrama de la izquierda, B AI est dada por la regin sombreada horizontalmente y ( )' B AI
est representado por el rea sombreada verticalmente. Por su parte, en el diagrama de la derecha, ' A
es la regin sombreada horizontalmente, ' B es el rea sombreada verticalmente, por lo que ' B ' A U est
representado por la superficie que no es blanca. Las regiones resultantes son iguales.

Ejemplo.
Dados los siguientes conjuntos:
{ } 12 11 10 9 8 7 6 5 4 3 2 1 , , , , , , , , , , , U =
{ } 11 9 7 4 3 1 , , , , , A =

{ } 12 11 9 7 5 2 1 , , , , , , B =
Comprobar las leyes de DMorgan:

Solucin.
{ } 12 11 9 7 5 4 3 2 1 , , , , , , , , B A = U
{ } 11 9 7 1 , , , B A = I

{ } 12 10 8 6 5 2 , , , , , ' A =
{ } 10 8 6 4 3 , , , , ' B =
( ) { } ( ) 1 10 8 6 _ , , ' B A = U
{ } ( ) 2 10 8 6 _ , , ' B ' A = I
Como ( ) ( ) ( ) ' B ' A ' B A I U = = 2 1
( ) { } ( ) 3 12 10 8 6 5 4 3 2 _ , , , , , , , ' B A = I
{ } ( ) 4 12 10 8 6 5 4 3 2 _ , , , , , , , ' B ' A = U
Como ( ) ( ) ( ) ' B ' A ' B A U I = = 4 3


I.6 PRODUCTO CARTESIANO DE DOS CONJUNTOS Y SU GRFICA

Uno de los principios bsicos para hacer un anlisis matemtico es el concepto de parejas ordenadas:
dos objetos, personas, smbolos o cosas mencionados en un orden definido por su posicin, es decir,
primero uno y luego el otro. Si este orden cambiara, es decir, primero el otro y luego el uno, se tendr
como resultado una nueva pareja ordenada y diferente a la inicialmente considerada.

La simbologa matemtica que se utiliza para representar una pareja ordenada es escribir dentro de un
parntesis, la primera componente separada por una coma de la segunda componente, por ejemplo:
( ) y , x es la pareja ordenada, en donde x es la primera componente y y es la segunda componente.

El producto cartesiano de dos conjuntos A y B es el conjunto de todos los posibles pares ordenados
que se forman eligiendo como primera componente a un elemento que pertenezca a A , y como segunda
componente a un elemento que pertenezca a B .

El producto cartesiano se denota de la siguiente forma: B A y se lee A cruz B .

( ) { A x y , x B A = y } B y

La definicin anterior expresa que el producto cartesiano de los conjuntos A y B , son la parejas
ordenadas ( ) y , x tal que x pertenece al conjunto A y y pertenece al conjunto B .
Pgina del Colegio de Matemticas de la ENP-UNAM Conjuntos Autor: Dr. Jos Manuel Becerra Espinosa
12

Ejemplo.
Obtener el producto cartesiano B A de los siguientes conjuntos:
{ } 3 2 1 , , A =

{ } 7 6 4 2 , , , B =
Solucin.
( ) ( ) ( ) ( ) ( ) ( ) ( ) ( ) ( ) ( ) ( ) ( ) { } 7 3 6 3 4 3 2 3 7 2 6 2 4 2 2 2 7 1 6 1 4 1 2 1 , , , , , , , , , , , , , , , , , , , , , , , B A =

El nmero de parejas ordenadas que resultan de un producto cartesiano se obtiene multiplicando sus
cardinalidades. En el ejemplo anterior,
( ) 3 = A
y ( ) 4 = B , el nmero de parejas ordenadas es:
( )( ) 12 4 3 = .

El producto cartesiano no es conmutativo. Esto significa que A B B A , a menos que B A = .

Ejemplo.
Obtener el producto cartesiano A B dados los mismos conjuntos anteriores:
{ } 3 2 1 , , A =

{ } 7 6 4 2 , , , B =
Solucin.
( ) ( ) ( ) ( ) ( ) ( ) ( ) ( ) ( ) ( ) ( ) ( ) { } 3 7 2 7 1 7 3 6 2 6 1 6 3 4 2 4 1 4 3 2 2 2 1 2 , , , , , , , , , , , , , , , , , , , , , , , A B =
A B B A

Ejemplo.
Dados los siguientes conjuntos:
{ } sanda , uva , mango P =
{ } limn , tuna , ciruela , pia , meln Q =
obtener los productos cartesianos Q P y P Q .
Solucin.
( ) ( ) ( ) ( ) ( ) { , limn , mango , tuna , mango , ciruela , mango , pia , mango , meln , mango Q P =
( ) ( ) ( ) ( ) ( ), limn , uva , tuna , uva , ciruela , uva , pia , uva , meln , uva
( ) ( ) ( ) ( ) ( )} limn , sanda , tuna , sanda , ciruela , sanda , pia , sanda , meln , sanda
( ) ( ) ( ) { , sanda , meln , uva , meln , mango , meln P Q =
( ) ( ) ( ), sanda , pia , uva , pia , mango , pia
( ) ( ) ( ), sanda , ciruela , uva , ciruela , mango , ciruela
( ) ( ) ( ), sanda , tuna , uva , tuna , mango , tuna
( ) ( ) ( )} sanda limn, , uva limn, , mango limn,

Un sistema de dos ejes coordenados o plano cartesiano, se define como el conjunto de todas las parejas
ordenadas de nmeros reales, que corresponden en s al producto cartesiano R x R.

Un sistema de ejes coordenados se construye haciendo que dos lneas rectas se corten
perpendicularmente en un punto llamado origen, quedando el plano dividido en cuatro regiones llamadas
cuadrantes. Al eje horizontal se le conoce como eje x y al eje vertical como eje y .

Esto se representa de la siguiente forma:
Pgina del Colegio de Matemticas de la ENP-UNAM Conjuntos Autor: Dr. Jos Manuel Becerra Espinosa
13

x 1 4 3 2 5 -1 -2 -3 -4 -5
Cuadrante I
(+, +)
1
2
3
4
5
y
-1
-2
-3
-4
-5
Cuadrante IV
(+, -)
Cuadrante III
(-, -)
Cuadrante II
(-, +)


Dado que el conjunto R x R son todas las parejas ordenadas ( ) y , x de un plano cartesiano, se tiene que:

R
2
=R x R
( ) { = x y , x
R y y R }

En una pareja ordenada ( ) y , x , a x se le da el nombre de abscisa y a y , el nombre de ordenada. Estos
valores sirven para localizar un punto en el plano cartesiano, y se les llama coordenadas de un punto,
que se escribe como ( ) y , x P .

A cada pareja ordenada de este producto cartesiano le corresponde uno y slo un punto sobre el plano
cartesiano, y a cada punto del plano cartesiano le corresponde una y slo una pareja ordenada. A esto se
le llama correspondencia biunvoca.

Ejemplo.
Ubicar en el plano cartesiano los siguientes puntos:
( ) 4 2
1
, P
, ( ) 5 4
2
, P , ( ) 2 3
3
, P , ( ) 4 3
4
, P

x
1 4 3 2 5 -1 -2 -3 -4 -5
P
3
(-3,-2)
1
2
4
5
y
-1
-2
-3
-4
-5
3
P
2
(-4,5)
P
1
(2,4)
P
4
(3,-4)

Pgina del Colegio de Matemticas de la ENP-UNAM Conjuntos Autor: Dr. Jos Manuel Becerra Espinosa
14


Ejemplo.
Dados los conjuntos { } 4 3 2 , , A = y { } 3 2 1 , , B = , obtener la grfica del producto cartesiano B A

Solucin.
El conjunto solucin a este producto cartesiano son nueve puntos discretos formado por las parejas
ordenadas: ( ) ( ) ( ) ( ) ( ) ( ) ( ) ( ) ( ) { } 3 4 2 4 1 4 3 3 2 3 1 3 3 2 2 2 1 2 , , , , , , , , , , , , , , , , , B A = . Grficamente esto es:

x
1 4 3 2 5 -1 -2 -3 -4 -5
1
2
4
5
y
-1
-2
-3
-4
-5
3


Ejemplo.
Sean los conjuntos { } R = x , x x A 4 2 y { } R = y , x y B 5 2 , graficar el producto
cartesiano B A

El conjunto solucin a este producto cartesiano es una superficie plana de forma rectangular limitada
tanto en x como en y . Grficamente esto es:

x
1 4 3 2 5 -1 -2 -3 -4 -5
1
2
4
5
y
-1
-2
-3
-4
-5
3



Pgina del Colegio de Matemticas de la ENP-UNAM Sistemas de numeracin Autor: Dr. Jos Manuel Becerra Espinosa
1

SISTEMAS DE NUMERACIN

UNIDAD II


II.1 BREVE RESEA HISTRICA

Efectuar mediciones y conteos son las principales actividades matemticas que el hombre realiza desde
la antigedad. Sin embargo, en la actualidad se conoce muy poco acerca de los orgenes de los
nmeros, lo que si se sabe es que la necesidad de contar nace desde las pocas prehistricas de la
humanidad.

Los sistemas de recuento ms primitivos se basaban en el cinco, el diez o el veinte, que estn
relacionados con los cinco dedos que el humano tiene en cada mano, o los diez dedos si se toman
ambas, o los veinte si se consideran las manos y los pies.

Sin embargo, esta forma de contar era limitada. Imagnese, por ejemplo, que el hombre tena que contar
cuntas semillas sembraba en un da. Lo primero que hacia era relacionar cada semilla que sembraba
con una piedra y si le sobraban piedras le faltaban semillas y si le faltaban piedras le sobraban semillas,
es decir, utilizaba conjuntos equivalentes que se relacionaban.

Como puede advertirse, este mtodo era bastante complejo, as que lo que nuestros antepasados
hicieron, fue empezar a asociar smbolos que eran grabados en piedras o en la tierra, con los conceptos
que queran medir o contar, poco despus se comenzaron a asociar palabras y sonidos repitindolos en
el mismo orden. De esta forma la humanidad dio un paso gigantesco y comenz a contar cada vez ms
objetos.

Lo que se realiza en la actualidad, es la asociacin de conjuntos de forma biunvoca, es decir, se asocian
palabras con los nmeros, as como las cosas que se desean contar.

Ejemplo.
Para saber cuntas manzanas hay en una caja, se busca el conjunto de los nmeros naturales
equivalente al de manzanas de la caja. La cantidad de manzanas que hay en la caja es el nmero
cardinal del conjunto asociado (recurdese que la cardinalidad de un conjunto se defini como el nmero
de elementos que posee).

Los nmeros cardinales se utilizan para contar objetos de un conjunto. Esto es:
, , , , 4 3 2 1
, etc.

El proceso de asociar conjuntos de nmeros naturales sucesivos con los objetos de un conjunto
cualquiera es contar. Contar los elementos de un conjunto es independiente del orden que tomen dichos
objetos.

Cuando se toma en cuenta el orden de los objetos utilizamos lo que se conoce como nmeros ordinales.
Esto es: primero, segundo, tercero, etc.


II.2 SISTEMAS DE NUMERACIN

Los egipcios representan una de las civilizaciones ms antiguas y desarrolladas del mundo. Gracias a la
existencia de los papiros de Rhind y de sus mltiples jeroglficos es que se sabe algo acerca de su
aritmtica. Aunque emplearon el sistema duodecimal en la subdivisin del ao (en doce meses,
correspondientes a sus doce dioses principales) y del da (en doce horas de claridad y doce de tinieblas),
su numeracin era decimal y contaba con signos jeroglficos para las cifras del uno al diez y para cien,
mil, diez mil, cien mil y un milln.
Pgina del Colegio de Matemticas de la ENP-UNAM Sistemas de numeracin Autor: Dr. Jos Manuel Becerra Espinosa
2

Los babilonios, al igual que los egipcios, desarrollaron su propio sistema de numeracin, ellos escriban
sobre tablillas de arcilla, en donde utilizaban la escritura cuneiforme y no tenan ningn smbolo para
representar el cero. Utilizaban un sistema de numeracin de valor posicional a travs de dos smbolos
bsicos en forma de cua. Una en forma vertical para las unidades y otra en forma horizontal para las
decenas.

Los mayas inventaron un sistema de numeracin en donde aparece por primera vez el cero, adems de
que su base era el veinte, ya que se cree, que tal vez sea por el hecho de contemplar los dedos de pies y
manos. Esta civilizacin represent cada cantidad por medio de smbolos que segn la posicin que
ocupaban adquira cierto valor, es decir el sistema maya as como el decimal es un sistema de
posiciones. El smbolo del cero en cualquier posicin indica ausencia de cantidad.

Los hindes representaron con nueve smbolos diferentes, uno por cada nmero del uno al nueve. stos
han cambiado con el tiempo, pero llegaron a Europa en su forma actual en el siglo XVI.

Por su parte, los griegos y los hebreos, utilizaron nueve smbolos diferentes para estos nmeros. En cada
caso, los smbolos eran las primeras nueve letras de sus alfabetos.

El Imperio Romano desarroll un sistema de numeracin que se us en Europa hasta el siglo XVII. En la
actualidad es muy conocido y se usa para indicar los tomos de una obra, los captulos de un libro, el
nombre del siglo, el nombre de una poca, para las fechas, para los personajes de mismo nombre y las
horas en las cartulas de algunos relojes.

Las cifras estn representadas con letras que tienen determinado valor:

Letra Valor
I
1
V 5
X 10
L 50
C 100
D 500
M 000 1,


En esta numeracin la letra no depende de la posicin que sta tenga para que sea escrita. Para escribir
con nmeros romanos hay que tener en cuenta lo siguiente:

a) Los valores de las cifras iguales se suman

Ejemplos.
III = 3 1 1 1 = + +
XX = 20 10 10 = +
CCC= 300 100 100 100 = + +

b) Ninguna cifra puede repetirse ms de tres veces seguidas

Ejemplo.
La expresin XXXXX = 50 10 10 10 10 10 = + + + + es invlida. La forma correcta de representar a este
nmero es: L = 50



Pgina del Colegio de Matemticas de la ENP-UNAM Sistemas de numeracin Autor: Dr. Jos Manuel Becerra Espinosa
3

c) No pueden repetirse los nmeros V, L ni D

Ejemplos.
La expresin VV = 10 5 5 = + no es vlida. La forma correcta de representar a este nmero es: X
La expresin LL = 100 50 50 = + no es correcta. Se debe de representar como: C
La expresin DD = 000 1 500 500 , = + es invlida. La forma adecuada de representar a este nmero es:
M

d) Si se coloca a la derecha una cifra de menor valor, se suman los valores de las dos

Ejemplos.
LX = 60 10 50 = +
CL = 150 50 100 = +
MCLV = 155 1 5 50 100 000 1 , , = + + +

e) Todas las cifras colocadas a la izquierda de otra que sea mayor, se le tiene que restar el valor de la
menor a la mayor

Ejemplos.
XC = 90 10 100 =
CD = 400 100 500 =
CM = 900 100 000 1 = ,

f) Toda cifra colocada entre dos cifras iguales se resta su valor de la ltima

Ejemplos.
XIX = ( ) 19 1 10 10 = +
CXC = ( ) 190 10 100 100 = +
MMCM = ( ) 900 2 100 000 1 000 1 000 1 , , , , = + +

g) Si se coloca una lnea horizontal encima de una cifra, la hace mil veces mayor

Ejemplos.
IX = ( ) 000 9 000 1 9 , , =
XVII
= ( ) 000 17 000 1 17 , , =
DVIII
= ( ) 000 508 000 1 508 , , =


II.3 SISTEMA DECIMAL

La numeracin que se utiliza en la actualidad fue heredada por los rabes, por lo que sus caracteres los
llamamos arbigos. En un principio hubo dos clases de nmeros arbigos los del Imperio de Oriente y de
Occidente de Europa. En Mxico se emplean los occidentales, que fueron llevados por los Moros a
Espaa, los nmeros orientales se usan en Turqua, Egipto, Arabia y los pases vecinos.

De acuerdo a lo expuesto anteriormente, la numeracin egipcia y la romana, empleaban la base 10 pero
no usaban el principio de posicin. Otras numeraciones como la maya y la babilonia, usaban el principio de
posiciones pero no usaban la base diez. En el sistema decimal se usan los dos principios, es decir se utiliza
la base 10 , adems de que las cifras tienen su valor segn la posicin que stas ocupen.
Pgina del Colegio de Matemticas de la ENP-UNAM Sistemas de numeracin Autor: Dr. Jos Manuel Becerra Espinosa
4

Al decir que un sistema es de base diez, significa que slo hace uso de diez smbolos o guarismos
nicamente, es decir, los smbolos de base 10 son: 8 7 6 5 4 3 2 1 0 , , , , , , , , y 9.

Los dgitos pueden tener dos valores: un valor absoluto que es el que indica el nmero de unidades que
lo forman y un valor relativo que es el que adquieren segn la posicin que ocupan.

Ejemplo.
El valor absoluto de los dgitos que forman 496 es: 6 9 4 , , . Por su parte, el valor relativo es 90 400, y
6 .

Las cifras que intervienen en un nmero se dividen en perodos de seis cifras cada uno de la siguiente
forma:


Tercer periodo
Billones
Segundo periodo
Millones
Primer periodo
Unidades
Segundo
grupo
Miles
Primer
grupo
Unidades
Segundo
grupo
Miles
Primer
grupo
Unidades
Segundo
grupo
Miles
Primer
grupo
Unidades
T
e
r
c
e
r

g
r
u
p
o

C
e
n
t
e
n
a
s

S
e
g
u
n
d
o

g
r
u
p
o

D
e
c
e
n
a
s

P
r
i
m
e
r

g
r
u
p
o

U
n
i
d
a
d
e
s

T
e
r
c
e
r

g
r
u
p
o

C
e
n
t
e
n
a
s

S
e
g
u
n
d
o

g
r
u
p
o

D
e
c
e
n
a
s

P
r
i
m
e
r

g
r
u
p
o

U
n
i
d
a
d
e
s

T
e
r
c
e
r

g
r
u
p
o

C
e
n
t
e
n
a
s

S
e
g
u
n
d
o

g
r
u
p
o

D
e
c
e
n
a
s

P
r
i
m
e
r

g
r
u
p
o

U
n
i
d
a
d
e
s

T
e
r
c
e
r

g
r
u
p
o

C
e
n
t
e
n
a
s

S
e
g
u
n
d
o

g
r
u
p
o

D
e
c
e
n
a
s

P
r
i
m
e
r

g
r
u
p
o

U
n
i
d
a
d
e
s

T
e
r
c
e
r

g
r
u
p
o

C
e
n
t
e
n
a
s

S
e
g
u
n
d
o

g
r
u
p
o

D
e
c
e
n
a
s

P
r
i
m
e
r

g
r
u
p
o

U
n
i
d
a
d
e
s

T
e
r
c
e
r

g
r
u
p
o

C
e
n
t
e
n
a
s

S
e
g
u
n
d
o

g
r
u
p
o

D
e
c
e
n
a
s

P
r
i
m
e
r

g
r
u
p
o

U
n
i
d
a
d
e
s



El perodo de la derecha son las unidades, el siguiente son los millones, el siguiente es el de los billones,
etc.

Cada perodo se puede dividir en dos grupos de tres cifras cada uno: las unidades y los millares, a su vez
cada grupo se divide en unidades, decenas y centenas.

Como el sistema de base 10 , consta de diez dgitos o guarismos, si se desea contar utilizando la base
diez, se debe hacer de la siguiente manera:

, , , , , , , , , , , , , , , , , , , , , , , , , , , , 1001 1000 999 111 110 109 101 100 99 31 30 29 21 20 19 11 10 9 8 7 6 5 4 3 2 1 etc.

Es posible tambin escribir un nmero en notacin desarrollada, esto es, que cualquier cantidad se
puede escribir cmo la suma de los dgitos del nmero por la base diez elevada al correspondiente
exponente.

En general, un nmero N se puede expresar de la siguiente manera:

m
m
n
n
n
n
n
n
x x x x x x x x N

+ + + + + + + + + = 10 10 10 10 10 10 10 10
2
2
1
1
0
0
1
1
2
2
1
1


en donde:

x es cada dgito componente del nmero
n es el nmero de dgitos a la izquierda del punto decimal menos uno
m es el nmero de dgitos a la derecha del punto decimal
Pgina del Colegio de Matemticas de la ENP-UNAM Sistemas de numeracin Autor: Dr. Jos Manuel Becerra Espinosa
5

Ejemplo.
Representar el nmero 567 498 32 . , en notacin desarrollada.

Solucin.
( ) ( ) ( ) ( ) ( ) ( ) ( ) ( )
3 2 1 0 1 2 3 4
10 7 10 6 10 5 10 8 10 9 10 4 10 2 10 3 567 498 32

+ + + + + + + = . ,
( ) ( ) ( ) ( ) ( )
000 1
7
100
6
10
5
1 8 10 9 100 4 000 1 2 000 10 3 567 498 32
,
, , . , + + + + + + + =

Comprobando:
007 0 06 0 5 0 8 90 400 000 2 000 30 567 498 32 . . . , , . , + + + + + + + =


II.4 SISTEMAS DE DIFERENTES BASES

Existen diversos tipos de sistemas de numeracin dependiendo de la base. En general, en un sistema de
base n , se utilizan n smbolos. De acuerdo con esto, los sistemas ms utilizados adems del decimal
son:

Sistema Binario.

Es un sistema de base 2 , en el que slo se tienen dos smbolos o guarismos que son: 0 y 1. Es de
gran importancia debido a que es el lenguaje que manejan las computadoras. Su notacin es
( )
2
N


Ejemplo de nmero binario:
( )
2
1001111001

Sistema Octal.

Es un sistema de base 8 , tal como su nombre lo indica. Cuenta ocho smbolos: 6 5 4 3 2 1 0 , , , , , , y 7 . Su
notacin es
( )
8
N


Ejemplo de nmero octal:
( )
8
352706

Sistema Hexadecimal

Es un sistema de base 16 . Est formado por diecisis elementos: E , D , C , B , A , , , , , , , , , , 9 8 7 6 5 4 3 2 1 0 y F .
Su notacin es
( )
16
N
y las letras tienen los siguientes valores:
15 14 13 12 11 10 = = = = = = F , E , D , C , B , A

Ejemplo de nmero hexadecimal:
( )
16
6 4 7 B CE A


Se dice que los sistemas anteriores son de posicin, debido a que el valor que se le asigna a una
cantidad depende de la posicin relativa de sus smbolos.

Los sistemas numricos de posicin quedan representados por la siguiente expresin:

m
m
n
n
n
n
n
n a
a x a x a x a x a x a x a x a x N

+ + + + + + + + + =
2
2
1
1
0
0
1
1
2
2
1
1

Pgina del Colegio de Matemticas de la ENP-UNAM Sistemas de numeracin Autor: Dr. Jos Manuel Becerra Espinosa
6

en donde:

a es la base a la cual pertenece el nmero
x es cada dgito componente del nmero
n es el nmero de dgitos a la izquierda del punto menos uno
m es el nmero de dgitos a la derecha del punto

Los ndices positivos corresponden a la parte entera mientras que los negativos corresponden a la parte
fraccionaria del nmero representado. El punto debe considerarse antes del primer subndice negativo.

Ejemplo.
Representar en notacin desarrollada a los siguientes nmeros:

1) ( )
7
203 461.
Solucin.
( ) ( ) ( ) ( ) ( ) ( ) ( )
3 2 1 0 1 2
7
7 3 7 0 7 2 7 1 7 6 7 4 203 461

+ + + + + = .

2) ( )
13
2 4 6 5 C . B A
Solucin.
( ) ( ) ( ) ( ) ( ) ( ) ( ) ( )
3 2 1 0 1 2 3
13
13 2 13 12 13 4 13 11 13 6 13 10 13 5 2 4 6 5

+ + + + + + = C . B A

3) ( )
5
412 2403.
Solucin.
( ) ( ) ( ) ( ) ( ) ( ) ( ) ( )
3 2 1 0 1 2 3
5
5 2 5 1 5 4 5 3 5 0 5 4 5 2 412 2403

+ + + + + + = .


II.5 SISTEMA BASE 2

II.5.1 NMEROS BINARIOS

En una computadora el sistema de numeracin es en base 2 , utilizando slo dos smbolos: el 0 y el 1
hecho propiciado por ser precisamente dos los estados estables en los dispositivos digitales que
componen un procesador.

Esto significa que todo lo que sucede dentro de una computadora puede representarse con alguna
combinacin de estados on y off, en donde se interpreta el 1 como prendido y el 0 como apagado.

Una secuencia de estados en la computadora on-off-off-on-off se puede escribir matemticamente como
10010 , nmero que contiene cierta informacin.

A los nmeros de base 2 se llaman nmeros binarios. En esta base, cualquier nmero puede
representarse como:

( )
m
m
n
n
n
n
n
n
x x x x x x x x N

+ + + + + + + + + = 2 2 2 2 2 2 2 2
2
2
1
1
0
0
1
1
2
2
1
1 2


en donde:

x es cada dgito (cero o uno) del nmero
n es el nmero de dgitos a la izquierda del punto binario menos uno
m es el nmero de dgitos a la derecha del punto binario
Pgina del Colegio de Matemticas de la ENP-UNAM Sistemas de numeracin Autor: Dr. Jos Manuel Becerra Espinosa
7

Ejemplo.
El nmero
( )
2
011 10010.
representado como potencias de dos es:
( ) ( ) ( ) ( ) ( ) ( ) ( ) ( ) ( )
3 2 1 0 1 2 3 4
2
2 1 2 1 2 0 2 0 2 1 2 0 2 0 2 1 011 10010

+ + + + + + + = .

II.5.2 CONVERSIN DE NMEROS BASE DOS A BASE DIEZ

Para convertir un nmero base dos en decimal, basta con aplicar la expresin que calcula ( )
2
N
considerando que se simplifica cuando se tiene un cero.

Ejemplo.
Transformar el nmero
( )
2
101 1010.
a base decimal.

Solucin.
( ) ( ) ( ) ( ) ( ) ( ) ( ) ( )
3 2 1 0 1 2 3
2
2 1 2 0 2 1 2 0 2 1 2 0 2 1 101 1010

+ + + + + + = .
( ) ( ) ( ) ( ) ( ) ( ) ( ) 125 0 5 0 2 8 125 0 1 25 0 0 5 0 1 1 0 2 1 4 0 8 1 . . . . . + + + = + + + + + + =
625 10. =

Ejemplo.
Convertir el nmero
( )
2
011 10010.
a base decimal.
Solucin.
( ) ( ) ( ) ( ) ( ) ( ) ( ) ( ) ( )
3 2 1 0 1 2 3 4
2
2 1 2 1 2 0 2 0 2 1 2 0 2 0 2 1 011 10010

+ + + + + + + = .
( ) ( ) ( ) ( ) ( ) ( ) ( ) ( ) 125 0 25 0 2 16 125 0 1 25 0 1 5 0 0 1 0 2 1 4 0 8 0 16 1 . . . . . + + + = + + + + + + + =
375 18. =

Considerando las potencias de dos, se puede establecer la siguiente tabla de conversin de nmeros
expresados en sistema decimal a su respectiva forma binaria:

PARTE ENTERA PARTE FRACCIONARIA
Sistema
decimal
Sistema
binario
Sistema
decimal
Sistema
binario
0 (0000)2 0 (0.0000)2
1 (0001)2 0.0625 (0.0001)2
2 (0010)2 0.1250 (0.0010)2
3 (0011)2 0.1875 (0.0011)2
4 (0100)2 0.2500 (0.0100)2
5 (0101)2 0.3125 (0.0101)2
6 (0110)2 0.3750 (0.0110)2
7 (0111)2 0.4375 (0.0111)2
8 (1000)2 0.5000 (0.1000)2
9 (1001)2 0.5625 (0.1001)2
10 (1010)2 0.6250 (0.1010)2
11 (1011)2 0.6875 (0.1011)2
12 (1100)2 0.7500 (0.1100)2
13 (1101)2 0.8125 (0.1101)2
14 (1110)2 0.8750 (0.1110)2
15 (1111)2 0.9375 (0.1111)2

II.5.3 CONVERSIN DE NMEROS BASE DIEZ A BASE DOS

Para convertir un nmero decimal natural en binario, se divide reiteradamente por dos los cocientes
sucesivos y los residuos sern, en orden de abajo hacia arriba, las cifras binarias. El proceso se repite
con los cocientes obtenidos mientras no sea cero.

Pgina del Colegio de Matemticas de la ENP-UNAM Sistemas de numeracin Autor: Dr. Jos Manuel Becerra Espinosa
8

Leer el nmero de
abajo hacia arriba
Leer el nmero de
abajo hacia arriba
Leer el nmero de
arriba hacia abajo
Ejemplo.
Transformar a base dos los siguientes nmeros expresados en base diez.
1) 13
Solucin:
1
2
1
0
2
1
1
2
1
1
2
3
0
2
0
3
2
6
1
2
1
6
2
13
3
2
1
0
= + =
= + =
= + =
= + =
x ,
x ,
x ,
x ,

El nmero buscado es:
( )
2
1101

2) 37
Solucin.
1
2
1
0
2
1
0
2
0
1
2
2
0
2
0
2
2
4
1
2
1
4
2
9
0
2
0
9
2
18
1
2
1
18
2
37
5
4
3
2
1
0
= + =
= + =
= + =
= + =
= + =
= + =
x ,
x ,
x ,
x ,
x ,
x ,

El nmero buscado es:
( )
2
100101


Para convertir un nmero decimal fraccionario en binario, se multiplica reiteradamente por dos la parte
fraccionaria y los nmeros antes del punto sern, en orden de arriba hacia abajo, las cifras binarias. El
proceso se repite hasta que la parte fraccionaria sea cero o que la obtencin de componentes se
vuelva cclica.

Ejemplo.
Transformar el nmero 875 0. a binario.
Solucin:
( )
( )
( ) 1 0 1 2 5 0
1 5 0 1 2 75 0
1 75 0 1 2 875 0
3
2
1
= + =
= + =
= + =

x , .
x , . .
x , . .

El nmero buscado es:
( )
2
111 0.



Pgina del Colegio de Matemticas de la ENP-UNAM Sistemas de numeracin Autor: Dr. Jos Manuel Becerra Espinosa
9

Leer el nmero de
arriba hacia abajo
Ejemplo.
Convertir el nmero 60 0. a binario.
Solucin.
( )
( )
( )
( )
( ) 1 20 0 1 2 60 0
1 60 0 1 2 80 0
0 80 0 0 2 40 0
0 40 0 0 2 20 0
1 20 0 1 2 60 0
5
4
3
2
1
= + =
= + =
= + =
= + =
= + =

x , . .
x , . .
x , . .
x , . .
x , . .

Como se puede observar, no se llega a tener una fraccin cero, pero la obtencin de las componentes es
cclica ya que la fraccin inicial es igual a la ltima fraccin obtenida. A medida de que se tomen ms
cifras de periodos sucesivos, la precisin es mayor. Esto significa que el nmero buscado es: ( )
2
1001 0.


II.6 CONVERSIONES DE NMEROS EN DISTINTAS BASES

II.6.1 CONVERSIONES DE NMEROS EXPRESADOS EN CUALQUIER BASE A BASE DIEZ

Para transformar un nmero en base diez a otra base, se aplica la expresin:

m
m
n
n
n
n
n
n a
a x a x a x a x a x a x a x a x N

+ + + + + + + + + =
2
2
1
1
0
0
1
1
2
2
1
1


en donde:
a es la base a la cual pertenece el nmero
x es cada dgito componente del nmero
n es el nmero de dgitos a la izquierda del punto menos uno
m es el nmero de dgitos a la derecha del punto

Ejemplos.
Transformar los siguientes nmeros a base diez:
1)
( )
5
1 3042.

Solucin.
( ) ( ) ( ) ( ) ( ) ( )
1 0 1 2 3
5
5 1 5 2 5 4 5 0 5 3 1 3042

+ + + + = .
( ) ( ) ( ) ( ) ( ) 2 397 2 0 2 20 375 2 0 1 1 2 5 4 25 0 125 3 . . . = + + + = + + + + =
2)
( )
3
12 2101.

Solucin.
( ) ( ) ( ) ( ) ( ) ( ) ( )
2 1 0 1 2 3
3
3 2 3 1 3 1 3 0 3 1 3 2 12 2101

+ + + + + = .
efectuando las operaciones y considerando la parte decimal peridica
1
:
( ) ( ) ( ) ( ) ( ) ( ) ( ) 22 0 33 0 1 9 54 11 0 2 33 0 1 1 1 3 0 9 1 27 2 12 2101
3
. . . . . + + + + = + + + + + = 55 64. =
3) ( )
11
30 694 5 A . A
Solucin.
( ) ( ) ( ) ( ) ( ) ( ) ( ) ( )
2 1 0 1 2 3 4
11
11 0 11 3 11 4 11 9 11 6 11 11 5 30 694 5

+ + + + + + = A . A
efectuando las operaciones y considerando la parte decimal peridica:
( ) ( ) ( ) ( ) ( ) ( ) ( ) ( ) 0083 0 0 09 0 3 1 4 11 9 121 6 1331 10 14641 5 30 694 5
11
. . . A + + + + + + =
27 87344 2727 0 4 99 726 13310 73205 . . = + + + + + =

1
Un nmero decimal que tiene una barra significa que las cifras que se repiten en grupos. Por ejemplo: L 345345345 0 345 0 . . =
Pgina del Colegio de Matemticas de la ENP-UNAM Sistemas de numeracin Autor: Dr. Jos Manuel Becerra Espinosa
10

Leer el nmero de
abajo hacia arriba
Leer el nmero de
abajo hacia arriba
Leer el nmero de
abajo hacia arriba
II.6.2 CONVERSIONES DE NMEROS ENTEROS EXPRESADOS EN BASE DIEZ A CUALQUIER
BASE

Para convertir un nmero decimal entero a una base a , se divide reiteradamente por a los cocientes
sucesivos y los residuos sern, en orden de abajo hacia arriba, las cifras que componen al nmero. El
proceso se repite con los cocientes obtenidos mientras no sea cero.

Ejemplos.
Transformar los siguientes nmeros decimales a la base pedida:

1) 44 a base seis.
Solucin.
1
6
1
0
6
1
1
6
1
1
6
7
2
6
2
7
6
44
2
1
0
= + =
= + =
= + =
x ,
x ,
x ,

El nmero buscado es:
( )
6
112


2) 562 a base octal.
Solucin.
1
8
1
0
8
1
0
8
0
1
8
8
6
8
6
8
8
70
2
8
2
70
8
562
3
2
1
0
= + =
= + =
= + =
= + =
x ,
x ,
x ,
x ,

El nmero buscado es:
( )
8
1062


3) 7829 a base quince.
Solucin.
2
15
2
0
15
2
4
15
4
2
15
34
11
15
11
34
15
521
14
15
14
521
15
7829
3
2
1
0
= + =
= + =
= = + =
= = + =
x ,
x ,
B x ,
E x ,

El nmero buscado es:
( )
15
24BE




Pgina del Colegio de Matemticas de la ENP-UNAM Sistemas de numeracin Autor: Dr. Jos Manuel Becerra Espinosa
11

Leer el nmero de
arriba hacia abajo
Leer el nmero de
arriba hacia abajo
II.6.3 CONVERSIONES DE NMEROS FRACCIONARIOS EXPRESADOS EN BASE DIEZ A CUALQUIER BASE

Para convertir un nmero fraccionario expresado en base diez a una base a , se multiplica
reiteradamente por a la parte fraccionaria y los nmeros antes del punto sern, en orden de arriba
hacia abajo, las cifras que componen al nmero. El proceso se repite hasta que la parte fraccionaria
sea cero o que la obtencin de componentes se vuelva cclica.

Ejemplo.
Transformar los siguientes nmeros decimales fraccionarios a la base pedida:

1) 20 0. a base cinco
Solucin.
( ) 1 00 0 1 5 20 0
1
= + =

x , . .

El nmero buscado es:
( )
5
1 0.


2) 60 0. a base nueve
Solucin.
( )
( )
( ) 5 40 0 5 9 60 0
3 60 0 3 9 40 0
5 40 0 5 9 60 0
3
2
1
= + =
= + =
= + =

x , . .
x , . .
x , . .

Como se puede observar, no se llega a tener una fraccin cero, pero la obtencin de las componentes es
cclica ya que la fraccin inicial es igual a la ltima fraccin obtenida. A medida de que se tomen ms
cifras de periodos sucesivos, la precisin es mayor. Por ejemplo, si se consideran tres cifras el resultado
aproximado con cinco decimales es:
( ) ( ) ( ) ( ) 59943 0 00685 0 03703 0 55555 0 9 5 9 3 9 5 535 0
3 2 1
9
. . . . . = + + = + + =


Ahora, si se consideran cinco cifras el resultado aproximado con cuatro decimales tiene mayor precisin:
( ) ( ) ( ) ( ) ( ) ( )
5 4 3 2 1
9
9 5 9 3 9 5 9 3 9 5 53535 0

+ + + + = .
59989 0 00001 0 00045 0 00685 0 03703 0 55555 0 . . . . . . = + + + + =
Esto significa que el nmero buscado es: ( )
9
53 0.

3) 18 0. a base hexadecimal
Solucin.
( )
( )
( )
( )
( )
( ) A x , . .
x , . .
x , . .
x , . .
E x , . .
x , . .
= = + =
= + =
= + =
= + =
= = + =
= + =

10 88 0 10 16 68 0
7 680 0 7 16 48 0
4 48 0 4 16 28 0
1 28 0 1 16 08 0
14 08 0 14 16 88 0
2 88 0 2 16 18 0
6
5
4
3
2
1

Esto significa que el nmero buscado es: ( )
16
147 2 0 A E .

II.6.4 CONVERSIONES DE NMEROS ENTEROS EXPRESADOS EN CUALQUIER BASE

Para convertir un nmero escrito en base a a base b , lo que se tiene que hacer primero es transformar
el nmero de base a a base diez por medio de:

m
m
n
n
n
n
n
n a
a x a x a x a x a x a x a x a x N

+ + + + + + + + + =
2
2
1
1
0
0
1
1
2
2
1
1

Pgina del Colegio de Matemticas de la ENP-UNAM Sistemas de numeracin Autor: Dr. Jos Manuel Becerra Espinosa
12

Leer el nmero de
abajo hacia arriba
Leer el nmero de
abajo hacia arriba
y posteriormente convertirlo a la base b por medio de divisiones sucesivas.

Ejemplos.
Transformar los siguientes nmeros a las bases pedidas.

1)
( )
12
9 3 A B
a base siete
Solucin.
Convirtiendo el nmero a base diez:
( ) ( ) ( ) ( ) ( )
0 1 2 3
12
12 10 12 9 12 11 12 3 9 3 + + + = A B
( ) ( ) ( ) ( ) 6886 10 108 1584 5184 1 10 12 9 144 11 1728 3 = + + + = + + + =
ahora, se convierte este nmero a base siete:
2
7
2
0
7
2
6
7
6
2
7
20
0
7
0
20
7
140
3
7
3
140
7
983
5
7
5
983
7
6886
4
3
2
1
0
= + =
= + =
= + =
= + =
= + =
x ,
x ,
x ,
x ,
x ,

El nmero buscado es:
( )
7
26035


2)
( )
6
514
a base tres
Solucin.
Convirtiendo el nmero a base diez:
( ) ( ) ( ) ( )
0 1 2
6
6 4 6 1 6 5 514 + + =
( ) ( ) ( ) 190 4 6 180 1 4 6 1 36 5 = + + = + + =
ahora, se convierte este nmero a base tres:
2
3
2
0
3
2
1
3
1
2
3
7
0
3
0
7
3
21
0
3
0
21
3
63
1
3
1
63
3
190
4
3
2
1
0
= + =
= + =
= + =
= + =
= + =
x ,
x ,
x ,
x ,
x ,

El nmero buscado es:
( )
3
21001


3)
( )
4
21 1203.
a base dos
Solucin.
Convirtiendo el nmero a base diez:
Pgina del Colegio de Matemticas de la ENP-UNAM Sistemas de numeracin Autor: Dr. Jos Manuel Becerra Espinosa
13

Leer el nmero de
abajo hacia arriba
Leer el nmero de
arriba hacia abajo
( ) ( ) ( ) ( ) ( ) ( ) ( )
2 1 0 1 2 3
4
4 1 4 2 4 3 4 0 4 2 4 1 21 1203

+ + + + + = .
considerando cuatro cifras significativas:
( ) ( ) ( ) ( ) ( ) ( ) ( ) 0625 0 1 25 0 2 1 3 4 0 16 2 64 1 21 1203
4
. . . + + + + + =
5625 99 0625 0 5 0 3 32 64 . . . = + + + + =
Al ser un nmero con parte entera y parte decimal, se procede en dos partes:
primero, se convierte la parte entera a base dos:
1
2
1
0
2
1
1
2
1
1
2
3
0
2
0
3
2
6
0
2
0
6
2
12
0
2
0
12
2
24
1
2
1
24
2
49
1
2
1
49
2
99
6
5
4
3
2
1
0
= + =
= + =
= + =
= + =
= + =
= + =
= + =
x ,
x ,
x ,
x ,
x ,
x ,
x ,

segundo, se convierte la parte fraccionaria a base dos:
( )
( )
( )
( ) 1 000 0 1 2 500 0
0 500 0 0 2 250 0
0 250 0 0 2 125 0
1 125 0 1 2 5625 0
4
3
2
1
= + =
= + =
= + =
= + =

x , . .
x , . .
x , . .
x , . .

Finalmente, el nmero buscado es: ( )
2
1001 1100011.

II.6.5 CONVERSIONES DE NMEROS BINARIOS A BASE OCTAL Y VICEVERSA

Cuando se desea transformar un nmero binario a base ocho o viceversa, se aplica la siguiente tabla de
conversin:

OCTAL BINARIO
0 000
1 001
2 010
3 011
4 100
5 101
6 110
7 111

y se toman grupos de tres cifras de la siguiente forma:

De derecha a izquierda cuando es una parte entera
De izquierda a derecha cuando se trata de una fraccin.


Pgina del Colegio de Matemticas de la ENP-UNAM Sistemas de numeracin Autor: Dr. Jos Manuel Becerra Espinosa
14

Ejemplo.
Convertir el nmero
( )
2
100 101011111.
a base ocho.
Solucin.
De acuerdo a la tabla, se cumple que: {{{{
4 7 3 5
100 111 011 101 .
, as que el nmero es:
( )
8
4 537.


Ejemplo.
Convertir el nmero
( )
8
41 6352.
a base dos.
Solucin.
De acuerdo a la tabla, se cumple que:
{{{{{{
001 100 010 101 011 110
1 4 2 5 3 6 .
, as que el nmero es:
( )
2
100001 10 1100111010 .


II.6.6 CONVERSIONES DE NMEROS BINARIOS A BASE HEXADECIMAL Y VICEVERSA

Para transformar un nmero binario a base hexadecimal o viceversa, se aplica la siguiente tabla de
conversin:

OCTAL BINARIO
0 0000
1 0001
2 0010
3 0011
4 0100
5 0101
6 0110
7 0111
8 1000
9 1001
A 1010
B 1011
C 1100
D 1101
E 1110
F 1111

y se toman grupos de cuatro cifras de la siguiente forma:

De derecha a izquierda cuando es una parte entera
De izquierda a derecha cuando se trata de una fraccin.

Ejemplo.
Convertir el nmero ( )
2
1010 01 1110011001 . a base diecisis.
Solucin.
De acuerdo a la tabla, se cumple que: {{{{
A E
.1010 0101 0110 1110
5 6
, as que el nmero es:
( )
16
65 A . E


Ejemplo.
Convertir el nmero
( )
16
7 3 9 F . D C
a base dos.
Solucin.
De acuerdo a la tabla, se cumple que:
{{{{{{
0111 1111 0011 1101 1001 1100
7 3 9 F . D C
, as que el nmero es:

( )
2
11110111 010011 1100100111 .

Pgina del Colegio de Matemticas de la ENP-UNAM Sistemas de numeracin Autor: Dr. Jos Manuel Becerra Espinosa
15

II.7 OPERACIONES EN DISTINTAS BASES

Para efectuar operaciones con nmeros en base a , es necesario recordar que al tener a dgitos, lo que
cambia es la posicin y la disposicin que se tenga.

Las operaciones aritmticas con nmeros en cualquier base cumplen reglas similares al sistema decimal:

II.7.1 SUMA

Dos nmeros se pueden sumar de una manera muy similar al que se hace en la base diez, slo que
cuando se presenta la operacin del dgito ms grande ms uno se escribe 0 y se tiene un acarreo de 1
(se lleva uno).

Ejemplos.

1) ( ) ( )
2 2
1011 1001 +
Solucin
Sumando los dgitos correspondientes de izquierda a derecha:
1
0
2 2 2 1 1 = +
el resultado es 0 y se lleva 1
1
0
2 2 2 1 0 1 = + +
el resultado es 0 y se lleva 1
1 0 0 1 = + +
1
0
2 2 2 1 1 = +
el resultado es 0 y se lleva 1
( )
( )
( )
2
2
2
10100
1011
1001
+

2) ( ) ( )
3 3
221 1210 +
Solucin
Sumando los dgitos correspondientes de izquierda a derecha:
1 1 0 = +
1
0
3 3 3 2 1 = +
el resultado es 0 y se lleva 1
1
2
5 3 5 2 2 1 = + +
el resultado es 2 y se lleva 1
2 1 1 = +
( )
( )
( )
3
3
3
2201
221
1210
+
Pgina del Colegio de Matemticas de la ENP-UNAM Sistemas de numeracin Autor: Dr. Jos Manuel Becerra Espinosa
16

3) ( ) ( )
5 5
20 34 +
Solucin
Sumando los dgitos correspondientes de izquierda a derecha:
4 0 4 = +
1
0
5 5 5 2 3 = +
el resultado es 0 y se lleva 1
( )
( )
( )
3
3
3
104
20
34
+

4) ( ) ( )
8 8
605 436 +
Solucin
Sumando los dgitos correspondientes de izquierda a derecha:
1
3
11 8 11 5 6 = +
el resultado es 3 y se lleva 1
4 0 3 1 = + +
1
2
10 8 10 6 4 = +
el resultado es 2 y se lleva 1
( )
( )
( )
8
8
8
1243
605
436
+


II.7.2 RESTA

Para restar dos nmeros en cualquier base se procede de forma similar que en el sistema decimal. Al
realizar las restas parciales entre dos dgitos de idntica posicin, si el sustraendo excede al minuendo,
se le quita uno al digito que est a la izquierda en el minuendo, ya que la unidad que est a la izquierda
tiene tantas unidades como elementos tiene la base.

En el caso de que el minuendo sea menor que el sustraendo el resultado ser negativo. Para fines
prcticos, en estos casos se resta la cantidad ms pequea a la ms grande y se le agrega el signo
negativo.

Ejemplos.
1) ( ) ( )
2 2
1001 1101
Solucin.

( )
( )
( )
2
2
2
0100
1001
1101


Pgina del Colegio de Matemticas de la ENP-UNAM Sistemas de numeracin Autor: Dr. Jos Manuel Becerra Espinosa
17

2) ( ) ( )
3 3
201 2102
Solucin.
( )
( )
( )
3
3
3
1201
201
2102

En la tercera columna 2 para 11 es 2 y se lleva 1. En la cuarta columna 1 1 0 = + para 2 es 1.

3) ( ) ( )
5 5
4123 2431
Solucin.
El sustraendo es mayor que el minuendo, as que el resultado es negativo.
Restando la cantidad menor a la mayor se tiene:

( )
( )
( )
5
5
5
1142
2431
4123

En la segunda columna 3 para 12 es 4 y se lleva 1. En la tercera columna 10 1 4 = + para 11 es 1
y se lleva 1. En la cuarta columna 3 1 2 = + para 4 es 1.
( ) ( ) ( )
5 5 5
1142 4123 2431 =

4) ( ) ( )
8 8
5214 4523
Solucin.
El sustraendo es mayor que el minuendo, as que el resultado es negativo.
Restando la cantidad menor a la mayor se tiene:
( )
( )
( )
8
8
8
471
4523
5214

En la segunda columna 2 para 11 es 7 y se lleva 1. En la tercera columna 6 1 5 = + para 12 es 4
y se lleva 1. En la cuarta columna 5 1 4 = + para 5 es 0 .
( ) ( ) ( )
8 8 8
471 5214 4523 =


II.7.3 MULTIPLICACIN

Cuando se desea multiplicar dos nmeros en base a se lleva a cabo conforme las tablas de multiplicacin
correspondientes a dicha base, considerando que cuando el resultado es mayor que a , entonces se debe
dividir ese nmero por la base. El residuo es el primer dgito resultante y el cociente es el acarreo que debe
sumarse a la siguiente operacin. Por ejemplo, si se quiere multiplicar ( ) ( )
7 7
5 6 se tiene:
( ) ( )
4
2
30 7 30 5 6 =

( )
( )
( )
7
7
7
42
5
6


Ejemplos.

1) ( ) ( )
2 2
11 101
Pgina del Colegio de Matemticas de la ENP-UNAM Sistemas de numeracin Autor: Dr. Jos Manuel Becerra Espinosa
18

Solucin.
( )
( )
( )
2
2
2
1111
101
101
11
101


1 1 1
0 0 1
1 1 1
=
=
=

1 1 1
0 0 1
1 1 1
=
=
=


2) ( ) ( )
3 3
12 110
Solucin.
( )
( )
( )
3
3
3
2020
110
220
12
110


2 1 2
2 1 2
0 0 2
=
=
=

1 1 1
1 1 1
0 0 1
=
=
=


3) ( ) ( )
5 5
20 14
Solucin.
( )
( )
( )
5
5
5
330
33
00
20
14


0 1 0
0 4 0
=
=

3 1 2 2 1 2
1
3
8 5 8 4 2
= + =
=
,


4) ( ) ( )
8 8
423 213
Solucin.
( )
( )
( )
8
8
8
112521
1054
426
641
423
213


6 2 3
4 1 3 3 1 3
1
1
9 8 9 3 3
=
= + =
=
,
4 2 2
2 1 2
6 3 2
=
=
=

1
0
8 8 8 2 4
5 1 4 4 1 4
1
4
12 8 12 3 4
=
= + =
=
,



II.7.4 DIVISIN

Cuando se desea dividir dos nmeros en cualquier base se procede de forma anloga a la que se hace
en la base diez. La divisin consiste en identificar cuntas veces un nmero (el divisor) est contenido en
otro nmero (el dividendo). Para fines prcticos, conviene escribir una tabla con los productos del divisor
para visualizar de forma ms fcil las operaciones. Al resultado entero de la divisin se le denomina
cociente y si la divisin no es exacta, es decir, el divisor no est contenido un nmero exacto de veces en
el dividendo, la operacin tendr un residuo.
Pgina del Colegio de Matemticas de la ENP-UNAM Sistemas de numeracin Autor: Dr. Jos Manuel Becerra Espinosa
19

Ejemplos.

1) ( ) ( )
8 8
7 43
Solucin.
Construyendo la tabla de multiplicacin para el divisor:
( ) ( ) ( )
( ) ( ) ( )
( ) ( ) ( )
( ) ( ) ( )
( ) ( ) ( )
( ) ( ) ( )
( ) ( ) ( )
8 8 8
8 8 8
8 8 8
8 8 8
8 8 8
8 8 8
8 8 8
61 7 7
52 6 7
43 5 7
34 4 7
25 3 7
16 2 7
7 1 7
=
=
=
=
=
=
=


El divisor cabe cinco veces en el dividendo y para obtener el residuo, se resta del dividendo el producto
del cociente por el divisor para obtener el residuo, que en este caso es cero:

( )
( )
( )
8
8
8
5
0
43
43 7



2) ( ) ( )
4 4
13 3211
Solucin.
Construyendo la tabla de multiplicacin para el divisor:
( ) ( ) ( )
( ) ( ) ( )
( ) ( ) ( )
4 4 4
4 4 4
4 4 4
111 3 13
32 2 13
13 1 13
=
=
=


El divisor cabe dos veces en las primeras dos cifras del dividendo, por lo que, la primera cifra del cociente es
2 . A continuacin se multiplica ( )
4
13 por ( )
4
2 , esto es: ( ) ( ) ( )
4 4 4
32 2 13 = y se efecta la resta de este
producto del dividiendo y se obtiene ( )
4
0 que es el residuo. Despus se baja el 1 y se aprecia que ( )
4
01 no
cabe ninguna vez, por lo que la segunda cifra del cociente es 0 . Posteriormente se baja el 1 siguiente y se
aprecia que ( )
4
11 no cabe ninguna vez, por lo que la tercera cifra del cociente es 0 . Se, multiplica ( )
4
13 por
( )
4
0 , que es ( )
4
0 y finalmente se efecta la resta de este producto del dividiendo y se obtiene ( )
4
11 que es el
residuo:

( ) ( )
( )
( )
4
4
4 4
200
11
00
011
32
3211 13


Pgina del Colegio de Matemticas de la ENP-UNAM Sistemas de numeracin Autor: Dr. Jos Manuel Becerra Espinosa
20

3) ( ) ( )
6 6
25 2543
Solucin.
( ) ( ) ( )
( ) ( ) ( )
( ) ( ) ( )
( ) ( ) ( )
( ) ( ) ( )
6 6 6
6 6 6
6 6 6
6 6 6
6 6 6
221 5 25
152 4 25
123 3 25
54 2 25
25 1 25
=
=
=
=
=


El ( )
6
25 cabe una vez en las primeras dos cifras del dividendo, por lo que, la primera cifra del cociente
es 1. A continuacin se multiplica ( )
6
1 por ( )
6
25 , que es ( )
6
25 , y se efecta la resta de este producto
del dividiendo y se obtiene ( )
6
0 que es el residuo. Despus se baja el 4 y se aprecia que ( )
6
04 no cabe
ninguna vez, por lo que la segunda cifra del cociente es 0 . Posteriormente se baja el 3 y se aprecia que
( )
6
43 cabe una vez en el divisor, por lo que, la tercera cifra del cociente es 1. Finalmente, se efecta la
resta de este producto del dividiendo y se obtiene ( )
6
14 que es el residuo:

( ) ( )
( )
( )
6
6
6 6
101
14
25
043
25
2543 25



4) ( ) ( )
3 3
211 2201222
Solucin.
( ) ( ) ( )
( ) ( ) ( )
3 3 3
3 3 3
1122 2 211
211 1 211
=
=


El ( )
3
211 cabe una vez en las tres primeras cifras del dividendo, por lo que, la primera cifra del cociente
es 1 y para obtener el residuo se multiplica ( )
3
1 por ( )
3
211 , que es ( )
3
211 . Ahora, se efecta la resta de
este producto del dividiendo y se obtiene ( )
3
2 que es el residuo. Despus se baja el 1 y se aprecia que
( )
3
21 no cabe ninguna vez en el divisor, por lo que la segunda cifra del cociente es 0 . A continuacin se
baja el 2 siguiente y se aprecia que ( )
3
212 cabe una vez en el divisor, por lo que, la tercera cifra del
cociente es 1. Nuevamente, se efecta la resta del producto de cociente por el divisor del dividiendo y se
obtiene ( )
3
1 que es el residuo, despus se baja el 2 siguiente y se observa que ( )
3
12 no cabe ninguna
vez en ( )
3
211 , por lo que la cuarta cifra del cociente es 0 . Se baja el ltimo 2 y se observa que ( )
3
122
no cabe ninguna vez en ( )
3
211 , por lo que la quinta cifra del cociente es 0 y se obtiene es el residuo es
( )
3
122 :

Pgina del Colegio de Matemticas de la ENP-UNAM Sistemas de numeracin Autor: Dr. Jos Manuel Becerra Espinosa
21

( ) ( )
( )
( )
3
3
3 3
10100
122
000
122
211
212
211
2201222 211




Pgina del Colegio de Matemticas de la ENP-UNAM El campo de los nmeros reales Autor: Dr. Jos Manuel Becerra Espinosa
1

EL CAMPO DE LOS NMEROS REALES

UNIDAD III


III.1 NMEROS NATURALES

Los nmeros naturales son aquellos que sirven para designar la cantidad de elementos que posee un
cierto conjunto
1
. Se representan como N.

N { } = , , , , , , , , 7 6 5 4 3 2 1 0

Los nmeros naturales son infinitos, pues para cada uno de ellos hay otro distinto que le sucede y que no
le precede.

Se habla del orden en estos nmeros a travs de su propiedad de tricotoma afirmando que dados n y
m dos nmeros naturales, entonces se tiene exactamente una de las tres posibilidades:

m n
m n
m n
>
=
<


Grficamente, este conjunto se puede representar mediante una recta numrica en donde los nmeros
son los puntos:








Una operacin en N es una manera de asociar a cada par de nmeros naturales, otro nmero natural
bien determinado. Las operaciones que se definen en este conjunto son la suma y la multiplicacin.

Sean a , b y c tres nmeros naturales cualesquiera. Las propiedades bsicas de las operaciones
definidas en N son:

1. Cerradura:
+b a N
b a N

2. Asociatividad:
( ) ( ) c b a c b a + + = + +
( ) ( ) c b a c b a =

1
Existen autores que definen al conjunto de los nmeros naturales como aquellos que sirven para contar, por lo que inician en el
uno. Si incluyen al cero lo definen como conjunto de nmeros naturales ampliados o como nmeros completos.
N
5 6 9 8 7 10 4 3 2 1 0 11
N
5 6 9 8 7 10 4 3 2 1 0 11
Pgina del Colegio de Matemticas de la ENP-UNAM El campo de los nmeros reales Autor: Dr. Jos Manuel Becerra Espinosa
2

3. Conmutatividad:
a b b a + = +
a b b a =

4. Elementos neutros
Para la suma es el cero ya que: a a = + 0
Para el producto es el uno ya que: a a = 1

5. Distributividad
La propiedad distributiva del producto sobre la suma es: ( ) c a b a c b a + = +

Ejemplo.
Dados los nmeros 2 , 3 y 5, comprobar las propiedades de la suma y del producto.

Solucin.
Cerradura:
= + 5 3 2 N
= 10 5 2 N
Asociatividad:
( ) ( ) 10 5 3 2 5 3 2 = + + = + +
( ) ( ) 30 5 3 2 5 3 2 = =
Conmutatividad:
5 2 3 3 2 = + = +
6 2 3 3 2 = =
Los elementos neutros:
Para la suma es el cero ya que: 2 0 2 = +
Para el producto es el uno ya que: 2 1 2 =
Distributividad
del producto sobre la suma es: ( ) 16 5 2 3 2 5 3 2 = + = +

Un nmero es mltiplo de otro si se obtiene multiplicando este ltimo por un nmero natural. Por ejemplo
el nmero 10 es mltiplo del 5 ya que ( )( ) 5 2 10 = .

Las propiedades de los mltiplos son:

El cero es mltiplo de cualquier nmero
Un nmero siempre es mltiplo de si mismo
La suma de mltiplos de un nmero tambin es un mltiplo de este nmero
El producto de mltiplos de un nmero tambin es mltiplo de este nmero
Si un nmero es mltiplo de otro y este lo es de un tercero, el primero es mltiplo del tercero.

Ejemplos.
El nmero 0 es mltiplo del 6 ya que ( )( ) 6 0 0 =
El nmero 7 es mltiplo del 7 ya que ( )( ) 7 1 7 =
El nmero 18 es mltiplo del 3 ya que ( )( ) 6 3 18 = y el nmero 12 tambin es mltiplo del 3 ya que
( )( ) 4 3 12 = , por lo tanto, el nmero 12 18 30 + = es mltiplo del 3 ya que ( )( ) 10 3 30 =
El nmero 6 es mltiplo del 2 ya que ( )( ) 3 2 6 = y el nmero 8 tambin es mltiplo del 2 ya que
( )( ) 4 2 8 = , por lo tanto, el nmero ( )( ) 8 6 48 = es mltiplo del 2 ya que ( )( ) 24 2 48 =
Pgina del Colegio de Matemticas de la ENP-UNAM El campo de los nmeros reales Autor: Dr. Jos Manuel Becerra Espinosa
3

El nmero 20 es mltiplo del 10 ya que ( )( ) 2 10 20 = , pero a su vez el nmero 10 es mltiplo del 5 ya
que ( )( ) 2 5 10 = , por lo tanto, el nmero 20 es mltiplo del 5 ya que ( )( ) 4 5 20 =

Un nmero natural es divisor de otro si cuando se divide el primero entre el segundo el residuo es cero,
es decir, si la divisin es exacta. Por ejemplo el nmero 2 es divisor del 6 ya que
3
2
6
=
.

Las propiedades de los divisores son:

El nmero uno es divisor de cualquier nmero
Un nmero siempre es divisor de si mismo
Si un nmero es divisor de otro, y ste lo es de un tercero, el primero es divisor del tercero.

Ejemplos.
El nmero 1 es divisor del 5 ya que
5
1
5
=

El nmero 11 es divisor del 11 ya que
1
11
11
=

El nmero 6 es divisor del 12 ya que
2
6
12
=
y el nmero 3 es divisor del 6 ya que
2
3
6
=
, por lo
tanto, el nmero 3 es divisor del 12 ya que
4
3
12
=


Cuando los nmeros son grandes hay reglas que permiten reconocer directamente que un nmero es
divisible por otro. Estas reglas se conocen como criterios de divisibilidad y los ms comunes son:

Un nmero es divisible por 2 si termina en cero o en cifra par.
Un nmero es divisible por 3 si la suma de sus cifras absolutas es mltiplo de tres.
Un nmero es divisible por 4 si en las dos ltimas cifras, hay dos ceros o un nmero mltiplo de cuatro.
Un nmero es divisible por 5 cuando acaba en cero o en cinco.
Un nmero es divisible por 6 si es divisible por dos y por tres.
Un nmero es divisible por 7 si su cifra ms significativa menos dos veces su siguiente cifra ms
cuatro veces su siguiente cifra y as sucesivamente es cero o mltiplo de siete.
Un nmero es divisible por 8 cuando sus tres ltimas cifras son ceros o son mltiplo de ocho.
Un nmero es divisible por 9 cuando la suma de sus cifras es mltiplo de nueve.
Un nmero es divisible por 10 si termina en cero.
Un nmero es divisible por 11 cuando la diferencia entre la suma de las cifras pares y las impares es
mltiplo de once o cero.
Un nmero es divisible por 12 cuando divisible por tres y por cuatro.

Ejemplo.
Aplicando los criterios anteriores, determinar la divisibilidad del nmero 720

Solucin:
a) Al terminar en cifra par, 720 es divisible por 2
b) 9 0 2 7 = + + que es mltiplo de 3, as que es divisible por 3
c) las dos ltimas cifras son 20 que es mltiplo de 4 , as que es divisible por 4
d) al terminar en cero, 720 es divisible por 5
e) al ser divisible por 2 y por 3, es divisible por 6
Pgina del Colegio de Matemticas de la ENP-UNAM El campo de los nmeros reales Autor: Dr. Jos Manuel Becerra Espinosa
4

f) 9 0 2 7 = + + que es mltiplo de 9 , as que es divisible por 9
g) al terminar en cero, 720 es divisible por 10

Los criterios de divisibilidad permiten encontrar con rapidez divisores de un nmero. Algunos nmeros
como el siete, trece o el diecinueve solo tienen dos divisores: la unidad y el mismo. Estos nmeros se
llaman nmeros primos y se denotan como P.

P { } = , , , , , , , , , , , , , , 43 41 37 31 29 23 19 17 13 11 7 5 3 2

Los nmeros que no son primos se llaman nmeros compuestos.

Descomponer un nmero en factores primos es expresarlo como producto de nmeros primos. En la prctica,
para descomponer un nmero en factores primos se divide sucesivamente por los nmeros primos comenzando
por el primer nmero primo hasta que se encuentre un cociente que sea igual a uno.
Ejemplo.
Descomponer el nmero 180 en factores primos.

Solucin.
5 1 5 5
3 5 3 15
3 15 3 45
2 45 2 90
2 90 2 180
=
=
=
=
=

Por lo que ( )( )( )( )( ) ( )( )( ) 5 3 2 5 3 3 2 2 180
2 2
= =

Para calcular todos los divisores de un nmero, se realiza la descomposicin factorial del nmero,
despus se determinan los divisores de cada uno de los factores y finalmente se construye un esquema
con todos los productos posibles.

Ejemplo.
Calcular todos los divisores del nmero 60 .

Solucin.
5 1 5 5
3 5 3 15
2 15 2 30
2 30 2 60
=
=
=
=

Por lo que ( )( )( )( ) ( )( )( ) ( )( )( ) 5 4 3 5 3 2 5 3 2 2 60
2
= = =
Los divisores del 3 son: 3 1,
Los divisores del 4 son: 4 2 1 , ,
Los divisores del 5 son: 5 1,
Efectuando todas las combinaciones posibles de productos, se tiene:
( )( )( )
( )( )( ) 5 5 1 1
1 1 1 1
=
=

Pgina del Colegio de Matemticas de la ENP-UNAM El campo de los nmeros reales Autor: Dr. Jos Manuel Becerra Espinosa
5

( )( )( )
( )( )( )
( )( )( )
( )( )( )
( )( )( )
( )( )( )
( )( )( )
( )( )( )
( )( )( )
( )( )( ) 60 5 4 3
12 1 4 3
30 5 2 3
6 1 2 3
15 5 1 3
3 1 1 3
20 5 4 1
4 1 4 1
10 5 2 1
2 1 2 1
=
=
=
=
=
=
=
=
=
=

Por lo tanto, los divisores del 60 , puestos en orden, son: 60 30 20 15 12 10 6 5 4 3 2 1 , , , , , , , , , , ,

El mnimo comn mltiplo de dos o ms nmeros, es el nmero ms pequeo posible, que es mltiplo de
esos nmeros.

Por ejemplo, el mnimo comn mltiplo de los nmeros 100 , 200 , 300 , es 600 , puesto que es el
nmero ms pequeo que es mltiplo de 100 , 200 y 300 .

Para calcular el mnimo comn mltiplo de dos o ms nmeros se descomponen los nmeros en factores
primos:
( )( )
2 2
5 2 100 =
( )( )
2 3
5 2 200 =
( )( )( )
2 2
5 3 2 300 =
y se toman los factores comunes y no comunes con su mayor exponente, en este caso:
( )( )( ) 600 5 3 2
2 3
=

La sustraccin es una operacin que no es cerrada en N, pues la diferencia de dos nmeros naturales
puede no ser un nmero natural (no lo es cuando el sustraendo es mayor que el minuendo). Por eso se
necesita definir otro conjunto de nmeros en el que se puede restar un nmero de otro, cualesquiera que
sean stos.


III.2 NMEROS ENTEROS

Los nmeros enteros, representados por Z son aquellos que surgen de la resta de dos nmeros
naturales
2
.

Z { = = b , a , b a x x N }

Este conjunto es una extensin de los nmeros naturales ya que incluye a sus opuestos, es decir
aparecen los nmeros negativos.

Z { } = , , , , , , , , , , , , 5 4 3 2 1 0 1 2 3 4 5


2
Se utiliza esta letra porque es la letra inicial de la palabra de origen alemn Zahlen, que significa nmero.
Pgina del Colegio de Matemticas de la ENP-UNAM El campo de los nmeros reales Autor: Dr. Jos Manuel Becerra Espinosa
6

Ejemplo.
Cuando en pocas muy fras la temperatura est por debajo de cero, implcitamente se habla de un
nmero entero, tal es el caso de -1C.

La razn principal para introducir los nmeros negativos sobre los nmeros naturales es la posibilidad de
resolver ecuaciones del tipo: b x a + = , para la incgnita x .

Se habla del orden en estos nmeros a travs de su propiedad de tricotoma afirmando que dados n y
m dos nmeros enteros, entonces se tiene exactamente una de las tres posibilidades:

m n
m n
m n
>
=
<


Esto significa que es un conjunto completamente ordenado sin cota superior o inferior y grficamente,
tambin se puede representar mediante una recta numrica en donde los nmeros son los puntos:








Una operacin en Z es una manera de asociar a cada par de nmeros enteros, otro nmero entero bien
determinado. Las operaciones que se definen en este conjunto son la suma y la multiplicacin (la resta se
considera como la suma de nmeros de diferente signo).

Sean a , b y c tres nmeros enteros cualesquiera. Las propiedades bsicas para la suma y el producto
en Z son:

1. Cerradura:
+b a Z
b a Z
2. Asociatividad:
( ) ( ) c b a c b a + + = + +
( ) ( ) c b a c b a =
3. Conmutatividad:
a b b a + = +
a b b a =
4. Elementos neutros
Para la suma es el cero ya que: a a = + 0
Para el producto es el uno ya que: a a = 1
5. Inverso aditivo:
Para la suma existe a tal que ( ) 0 = + a a
6. Distributividad
La propiedad distributiva del producto sobre la suma es: ( ) c a b a c b a + = +


Z
-1 0 3 2 1 4 -2 -3 -4 -5 5
Z
-1 0 3 2 1 4 -2 -3 -4 -5 5
Pgina del Colegio de Matemticas de la ENP-UNAM El campo de los nmeros reales Autor: Dr. Jos Manuel Becerra Espinosa
7

Ntese como no existe un inverso multiplicativo. Adems, la divisin no es una operacin cerrada en Z,
pues el cociente de dos nmeros naturales puede no ser un nmero entero (no lo es cuando el dividendo
no es mltiplo del divisor). Por esa razn, es necesario el establecimiento de otro sistema numrico en el
que se pueda dividir dos nmeros.


III.3 NMEROS RACIONALES

Nmero racional es el que se puede expresar como cociente de dos nmeros enteros con divisor
diferente de cero, es decir, en forma de fraccin. Se representan por Q.

Q
{ = = b , a ,
b
a
x x
Z } 0 b ,

Los nmeros racionales no enteros se llaman fraccionarios en donde a es el numerador y b el
denominador
3
. Ntese como en esta definicin, el denominador nunca puede ser cero porque la divisin
por cero no est definida.

En el conjunto de los nmeros enteros cada nmero tiene un siguiente (el siguiente al 3 es el 4 , el
siguiente al 6 es el 5 , etc.), no pasa lo mismo con los racionales, pues entre cada dos nmeros
racionales existe al menos otro nmero racional (propiedad de densidad).

Los nmeros racionales pueden ser ubicados tambin en la recta numrica mediante puntos,
independientemente de que no presentan una secuencia determinada, por ejemplo:








Al expresar un nmero racional, no entero, puede tener alguna de las siguientes representaciones:


Fracciones

Nmeros racionales
Exactos
Nmeros decimales
Peridicos


Si la fraccin es irreducible y en la descomposicin factorial del denominador slo se encuentran los
factores 2 y 5, entonces la fraccin es igual a un nmero decimal exacto, pero si en el denominador hay
algn factor distinto de 2 o 5 la expresin decimal es peridica, por ejemplo:

3
Los nmeros enteros son racionales, pues se pueden expresar como cociente de ellos mismos por la unidad y en otros cocientes
equivalentes como:
L = = = = =
4
8
3
6
2
4
1
2
2

Q
5
9
2
1

3
7

0
8
7
Q
5
9
2
1

3
7

0
8
7
Pgina del Colegio de Matemticas de la ENP-UNAM El campo de los nmeros reales Autor: Dr. Jos Manuel Becerra Espinosa
8

5 4
2
9
. =
, se obtiene un decimal exacto
4 2
5
12
. =
, se obtiene un decimal exacto
=
3 2 1 3 2 1 3 2 1
571428 571428 571428 1
7
11
.
, se obtiene un expresin decimal peridica.

Dos fracciones son equivalentes cuando tienen el mismo valor decimal. Las fracciones equivalentes
representan la misma parte de una cantidad.

Para convertir una fraccin a un nmero decimal peridico, basta con efectuar la divisin.

Ejemplo.
Convertir el nmero
11
13
a decimal.

Solucin.
90
20
90
20
13 11
1818 1 L .

= 18181818 1
11
13
.


Para convertir un nmero decimal peridico a fraccin, se emplea el siguiente procedimiento:

Se iguala a x el nmero decimal.
Se identifican las posiciones en donde inicia y termina el primer ciclo de periodicidad
Se multiplica por
n
10 a la expresin, donde n es el nmero de posiciones despus del punto
decimal en donde termina el ciclo.
Se multiplica por
m
10 a la expresin, donde m es el nmero de posiciones despus del punto
decimal en donde inicia el ciclo.
Se resta la segunda expresin a la primera
Se despeja x y se simplifica la fraccin de ser posible.

Ejemplos.
1) Convertir el nmero 72222 3. a fraccionario

Solucin.
= 72222 3. x
La periodicidad termina despus de dos cifras despus del punto decimal, as que se multiplica por
100 10
2
= . Por su parte la periodicidad inicia despus de una cifra a la derecha del punto, as que se
multiplica por 10 10
1
= . Esto es:
= 222 372 100 . x
Pgina del Colegio de Matemticas de la ENP-UNAM El campo de los nmeros reales Autor: Dr. Jos Manuel Becerra Espinosa
9

= 2222 37 10 . x
se resta la segunda expresin a la primera para eliminar la parte decimal:
335 90
2222 37 10
222 372 100
=
=
=
x
. x
. x

despejando x y simplificando se tiene:
18
67
90
335
= = x
= 72222 3
18
67
.

2) Convertir el nmero 346346346 2. a fraccionario

Solucin.
= 346346346 2. x
La periodicidad termina despus de tres cifras a la derecha del punto decimal, as que se multiplica por
000 1 10
3
, = . Por su parte la periodicidad inicia inmediatamente despus del punto, as que se multiplica
por 1 10
0
= . Esto es:
= 346346 346 2 000 1 . , x ,
= 346346346 2 1 . x
se resta la segunda expresin a la primera para eliminar la parte decimal:
344 2 999
346346346 2 1
346346 346 2 000 1
, x
. x
. , x ,
=
=
=

despejando x se tiene:
999
344 2,
x =

= 346346346 2
999
344 2
.
,


Simplificar una fraccin es sustituirla por la fraccin equivalente cuyo denominador es el menor posible.

Ejemplo.
Simplificar
90
48


Solucin.
3 1 3 3
2 3 2 6
2 6 2 12
2 12 2 24
2 24 2 48
=
=
=
=
=

( )( )( )( )( ) 3 2 2 2 2 48 =
Pgina del Colegio de Matemticas de la ENP-UNAM El campo de los nmeros reales Autor: Dr. Jos Manuel Becerra Espinosa
10

5 1 5 5
3 5 3 15
3 15 3 45
2 45 2 90
=
=
=
=

( )( )( )( ) 5 3 3 2 90 =
( )( )( )( )( )
( )( )( )( )
( )( )( )
( )( ) 15
8
5 3
2 2 2
5 3 3 2
3 2 2 2 2
90
48
= = =


En las fracciones se cumple que:
b
a
b
a
b
a

=
, aunque por convencin se utiliza
b
a



El orden en Q establece que un nmero es menor que otro, si est colocado a la izquierda de l en la
recta numrica; y es mayor, cuando est a su derecha. Para comparar dos fracciones se considera que:

bc ad
d
c
b
a
> >


Ejemplo.
Dadas las fracciones
5
8

y
4
9

, determinar cul es ms grande.



Solucin.
( )( ) 32 4 8 = y ( )( ) 45 9 5 = , como 45 32 > , se cumple que
4
9
5
8
>


Una operacin en Q es una manera de asociar a cada par de nmeros racionales, otro nmero racional
bien determinado. Las operaciones que se definen en este conjunto son la suma y la multiplicacin (la
resta se considera como la suma de nmeros de diferente signo y la divisin como la multiplicacin de un
nmero por el recproco de otro, siempre cuando el segundo no sea cero).

bd
bc
bd
ad
bd
bc ad
d
c
b
a
=

= , donde bd es el mnimo comn mltiplo de b y d


bd
ac
d
c
b
a
=
las fracciones antes de multiplicarse deben simplificarse
c
d
b
a
d
c
b
a
=

Ejemplos.
9
37
9
16 21
9
16
3
7
=
+
= +
35
6
105
18
105
10 28
21
2
15
4
= =
+
= +
8
7
2
1
4
7
6
3
8
14
=
|

\
|

\
|
=
|

\
|

\
|

Pgina del Colegio de Matemticas de la ENP-UNAM El campo de los nmeros reales Autor: Dr. Jos Manuel Becerra Espinosa
11

( )
2
9
9
2
1
3
27
4
2
4
27
3
2
4
27
9
6
27
4
9
6
= =
|

\
|
=
|

\
|
=
|

\
|
=
|

\
|


Sean a , b y c tres nmeros racionales cualesquiera. Las propiedades bsicas para la suma y el
producto en Q son:

1. Cerradura:
+b a Q
b a Q
2. Asociatividad:
( ) ( ) c b a c b a + + = + +
( ) ( ) c b a c b a =
3. Conmutatividad:
a b b a + = +
a b b a =
4. Elementos neutros
Para la suma es el cero ya que: a a = + 0
Para el producto es el uno ya que: a a = 1
5. Inversos:
Para la suma existe a , llamado opuesto o simtrico tal que ( ) 0 = + a a
Para el producto existe
0
1
a ,
a
, llamado inverso multiplicativo o recproco tal que 1
1
=
|

\
|

a
a
6. Distributividad
La propiedad distributiva del producto sobre la suma es: ( ) c a b a c b a + = +

Ejemplo.
Dados los nmeros
2
1
,
7
4

y
5
3
, comprobar las propiedades de las operaciones de la suma y el
producto en Q.

Solucin.
Cerradura:
=

= |

\
|
+
14
1
14
8 7
7
4
2
1
Q
=

= |

\
|

7
2
14
4
7
4
2
1
Q
Asociatividad:
5
3
7
4
2
1
5
3
7
4
2
1
+
(

\
|
+ =
|

\
|
+ +
5
3
14
8
14
7
35
21
35
20
2
1
+
(

\
|
+ =
|

\
|
+ +
5
3
14
1
35
1
2
1
+ = +
70
42
70
5
70
2
70
35
+ = +
70
37
70
37
=
5
3
7
4
2
1
5
3
7
4
2
1

|
|

\
|
|

\
|
=
|

\
|

35
6
35
6
70
12
70
12
5
3
14
4
35
12
2
1
= =
|

\
|
=
|

\
|


Pgina del Colegio de Matemticas de la ENP-UNAM El campo de los nmeros reales Autor: Dr. Jos Manuel Becerra Espinosa
12

Conmutatividad:
2
1
7
4
7
4
2
1
+ =
|

\
|
+
14
1
14
1
14
7
14
8
14
8
14
7
= + =
|

\
|
+
|

\
|

\
|
=
|

\
|

|

\
|
2
1
7
4
7
4
2
1
7
2
7
2
14
4
14
4
= =
Elementos neutros:
Para la suma es el cero ya que:
2
1
0
2
1
= +

Para el producto es el uno ya que:
2
1
1
2
1
=

Inversos:
Para el
2
1
su opuesto o simtrico es
2
1

ya que
0
2
1
2
1
= |

\
|
+

Para el
2
1
su inverso multiplicativo o recproco es
2
2
1
1
=
ya que
( ) 1 2
2
1
=

Distributividad del producto sobre la suma es:
|

\
|
+
|

\
|
=
|

\
|
+
5
3
2
1
7
4
2
1
5
3
7
4
2
1
10
3
14
4
35
21
35
20
2
1
+ =
|

\
|
+
70
1
70
1
70
21 20
70
1
10
3
7
2
35
1
2
1
=
+
= = + =
|

\
|

En general, dado un nmero racional de la forma
b
a
, su recproco viene dado por
a
b
.

Ejemplos.
El recproco de
4
3
es
3
4

El recproco de
5
1

es 5
El recproco de 8 es
8
1


Una razn es el resultado de comparar dos cantidades del mismo tipo, la primera de ellas llamada
antecedente ( ) a y la segunda llamada consecuente ( ) b . Estas cantidades se presentan en forma
fraccionaria, de la siguiente manera:

b
a
o b a o b a : con 0 b

Por ejemplo, un recipiente A tiene una capacidad de 6 litros y otro B tiene una capacidad de 3 litros.
Si se compara la capacidad de A con la de B , la razn es de
3
6
, es decir 2 , esto significa que A tiene
el doble de capacidad de B . Grficamente esto se puede ver en la siguiente figura:

Pgina del Colegio de Matemticas de la ENP-UNAM El campo de los nmeros reales Autor: Dr. Jos Manuel Becerra Espinosa
13

6 litros
A Recipiente
B Recipiente
3 litros
A Capacidad de B = 2 Veces la Capacidad de


Una proporcin es una proposicin que establece que dos razones son iguales. Esto es:

bc ad
d
c
b
a
= =
con 0 0 d , b

y se lee: a lo es a b como c lo es a d .

Por ejemplo, las razones
3
2
y
9
6
son equivalentes ya que se cumple que ( ) ( ) 18 6 3 9 2 = = , por lo tanto,
forman una proporcin. Grficamente esto es:

6 partes de 9 2 partes de 3
9
6
3
2
=


Las magnitudes proporcionales pueden ser de dos clases:

a) Magnitudes Directamente Proporcionales: Dadas dos cantidades el aumento de una corresponde al
aumento de la otra o la disminucin de una corresponde a la disminucin de la otra.
Pgina del Colegio de Matemticas de la ENP-UNAM El campo de los nmeros reales Autor: Dr. Jos Manuel Becerra Espinosa
14

Propiedad: si a y b son dos cantidades entonces:

a te tan cons
b
a
= y b son directamente proporcionales

Ejemplos.
1) Si se tiene la razn
5
4
y se quiere formar una proporcin directa se puede multiplicar por un mismo
nmero tanto al antecedente como al consecuente, si ese nmero es 3 se obtiene
15
12
. Ntese como
ambas cantidades aumentan.

2) Dada la razn
24
20
y se quiere formar una proporcin directa se puede dividir por un mismo nmero
tanto al antecedente como al consecuente, si ese nmero es 4 se obtiene
6
5
. Ntese como ambas
cantidades disminuyen.

b) Magnitudes Inversamente Proporcionales: Dadas dos cantidades el aumento de una corresponde a la
disminucin de la otra o la disminucin de una corresponde al aumento de la otra.

Propiedad: si a y b son dos cantidades entonces:

a te tan cons b a = y b son inversamente proporcionales

Ejemplos.
1) Si se tiene la razn
15
2
y se quiere formar una proporcin inversa se puede multiplicar por un nmero
al antecedente y dividir por ese mismo nmero al consecuente, si ese nmero es 5 se obtiene
3
10
.
Ntese como una cantidad aumenta y la otra disminuye.

2) Dada la razn
3
4
y se quiere formar una proporcin inversa se puede dividir por un nmero al
antecedente y multiplicar por ese mismo nmero al consecuente, si ese nmero es 2 se obtiene
6
2
.
Ntese como una cantidad disminuye y otra aumenta.

Una regla de tres directa se forma con la igualdad de dos razones directamente proporcionales, en donde
se conoce las dos cantidades de una razn y slo una cantidad de otra razn. Si a lo es a b , como c lo
es una cantidad desconocida x , se representa como:

)
`

x c
b a

donde x viene dado por:
a
b c
x

=



Pgina del Colegio de Matemticas de la ENP-UNAM El campo de los nmeros reales Autor: Dr. Jos Manuel Becerra Espinosa
15

Ejemplo.
Si tres discos cuestan 150 pesos, cunto costarn siete discos?

Solucin.
El problema expresado en forma de regla de tres directa es:
)
`

x 7
150 3
, entonces, los siete discos
costaran
( )
350
3
150 7
= = x
pesos.

Una regla de tres inversa se forma con la igualdad de dos razones inversamente proporcionales, en
donde se conoce las dos cantidades de una razn y slo una cantidad de otra razn. Si a lo es a b ,
como c lo es una cantidad desconocida x , se representa como:

)
`

x c
b a

donde x viene dado por:
c
b a
x

=


Ejemplo.
Si cuatro albailes hacen una casa en noventa das, cunto tiempo tardaran seis albailes?

Solucin.
El problema expresado en forma de regla de tres inversa es:
)
`

x 6
90 4
, entonces, los seis
trabajadores tardaran
( )
60
6
90 4
= = x
das.

Como se ha dicho, en el conjunto de los nmeros racionales se pueden efectuar las cuatro operaciones
bsicas, no obstante, no se puede resolver de forma general un problema donde intervenga la radicacin
de un nmero entero, por lo tanto, es necesario definir un nuevo sistema numrico.


III.4 NMEROS IRRACIONALES

Con los nmeros racionales se pueden representar casi todas las cantidades que se encuentran en la
vida cotidiana. Sin embargo, hay otra clase de nmeros, que se escriben con una infinidad de decimales
pero que no tienen un perodo, es decir, no tienen cifras que se repitan en el mismo orden. Los nmeros
de esta clase reciben el nombre de irracionales y, a diferencia de los racionales, no pueden expresarse
en forma de fraccin, sino slo en forma decimal. Se denotan por Q.

En general, cualquier raz inexacta de un nmero racional o alguna combinacin algebraica que la
involucre (y que exista) es un nmero irracional. Esto significa que este conjunto tambin es infinito.

Ejemplos de nmeros irracionales.

= 7320508075 1 3 .
= 0423711763 3 793
6
.
Pgina del Colegio de Matemticas de la ENP-UNAM El campo de los nmeros reales Autor: Dr. Jos Manuel Becerra Espinosa
16

=
+
6180339887 1
2
5 1
.
(este nmero es llamado ureo
4
)

Ntese como estos nmeros tienen una infinidad de cifras y no tienen periodicidad. Para todo fin prctico,
cuando se trabaja con nmeros irracionales se efectan aproximaciones, o bien, se utilizan algunos
smbolos especiales.

Ejemplo.

El nmero es un irracional que representa las veces que cabe el dimetro de una circunferencia en su
permetro P . Es decir, si se tuvieran las medidas exactas del permetro P de una circunferencia y de su
dimetro, D , viene dado por
D
P
. Si se quisiera efectuar la divisin nunca se terminara ya que se podran
obtener tantas cifras decimales como se quisiera, pero nunca se llegara a un residuo igual a cero, ni se
encontraran cifras que formen un perodo. Por lo tanto, no se puede escribir exactamente en cifras decimales:

= 897932 1415926535 3.

Los puntos suspensivos indican que las cifras son infinitas. En la prctica, sin embargo, cuando se
requiere calcular permetros o reas de circunferencias, volmenes de esferas o para hacer cualquier
otro clculo, en el que aparezca , se usa la aproximacin 1416 3. = .

Ejemplo.
2 es otro nmero irracional, ya que es la medida de la hipotenusa de un tringulo rectngulo cuyos
catetos miden una unidad de longitud. Normalmente se aproxima a 4142 1. , aunque su valor es de:
= 4142135623 1 2 .

Un nmero irracional tiene un nmero ilimitado de cifras, por tanto, es imposible escribir su valor exacto.
Para manejar estos nmeros se utilizan aproximaciones de los mismos. Aumentando el nmero de cifras,
el error va disminuyendo, de modo que puede ser tan pequeo como se quiera. Algunos nmeros
irracionales se pueden representar en la recta numrica mediante procedimientos geomtricos utilizando
regla y comps (por ejemplo las races cuadradas no exactas, como el caso de 2 ya expuesto). Para
muchos nmeros irracionales no se puede aplicar este mtodo, la representacin de estos nmeros se
hace por aproximacin. Por ejemplo, algunos nmeros irracionales en la recta seran:







Una operacin en Q es una manera de asociar a cada par de nmeros irracionales, otro nmero irracional
bien determinado. Las operaciones que se definen en este conjunto son la suma, la resta, la multiplicacin,
el cociente y la extraccin de races (exceptuando la radicacin de nmeros negativos de ndice par).


4
La seccin urea es la divisin armnica de un segmento en media y extrema razn. Es decir, que el segmento menor es al
segmento mayor, como este es a la totalidad. De esta manera se establece una relacin de tamaos con la misma proporcionalidad
entre el todo dividido en mayor y menor. Matemticamente es el resultado de la expresin:
1
1 x
x
x
=

.
Q
8
0
2
4
10
Q
8
0
2
4
10
Pgina del Colegio de Matemticas de la ENP-UNAM El campo de los nmeros reales Autor: Dr. Jos Manuel Becerra Espinosa
17

Sean a , b y c tres nmeros irracionales cualesquiera. Las propiedades bsicas para la suma en Q son:

1. Asociatividad:
( ) ( ) c b a c b a + + = + +
2. Conmutatividad:
a b b a + = +
3. Inverso:
Para la suma existe a , llamado opuesto o simtrico tal que ( ) 0 = + a a

Las operaciones de suma y producto no son cerradas en Q y no tiene elemento neutro.


III.5 NMEROS REALES

El conjunto de los nmeros reales surge de la unin de los nmeros racionales y de los irracionales. Se
denotan como R. Este conjunto comprende a todos los sistemas numricos anteriores.

R=QU Q

Se habla del orden en los nmeros reales a travs de la propiedad de tricotoma afirmando que dados n
y m dos nmeros reales, entonces se tiene exactamente una de las tres posibilidades:

m n
m n
m n
>
=
<


Al igual que en los conjuntos N, Z, Q y Q, los nmeros reales se pueden representar en una recta, slo
que en este caso no hay puntos discretos, sino se trata de una recta continua:







La recta R sobre la cual se representa a los nmeros racionales e irracionales se llama recta real. A cada
punto de esta recta se le asocia un nico nmero real llamado coordenada o abscisa del punto y,
recprocamente, a cada punto de esa recta se le asocia un nico nmero para que sea su coordenada. Si
esta doble asignacin se hace de manera que puntos distintos tengan coordenadas distintas y cada
nmero sea coordenada de algn punto, se ha obtenido una correspondencia biunvoca entre la recta y el
conjunto de los nmeros reales. Esta asignacin se denomina sistema de coordenadas unidimensional.

En general, dado un punto P cualquiera en la recta, al nmero real a se le llama coordenada o abscisa
de P y se denota por ( ) a P , que se lee: punto P de coordenada a .

Ejemplo.
Ubicar de forma aproximada los siguientes nmeros en la recta real:
75 5 2 0
2
11
2 . , , , , ,

Solucin.
R
-1 0 3 2 1 4 -2 -3 -4 -5 -6 5 6
R
-1 0 3 2 1 4 -2 -3 -4 -5 -6 5 6
Pgina del Colegio de Matemticas de la ENP-UNAM El campo de los nmeros reales Autor: Dr. Jos Manuel Becerra Espinosa
18

En forma de coordenadas, los nmeros toman la forma: ( ) 2
1
P ,
|

\
|

2
11
2
P
, ( ) 0
3
P , ( )
4
P , ( ) 2
5
P ,
( ) 75 5
6
. P que en la recta real estn localizados as
5
:







Una operacin en R es una manera de asociar a cada par de nmeros reales, otro nmero real bien
determinado. Las operaciones que se definen en este conjunto son la suma, la multiplicacin (la resta se
considera como la suma de nmeros de diferente signo y la divisin como la multiplicacin de un nmero
por el recproco de otro, siempre cuando el segundo no sea cero), la radicacin de nmeros positivos y la
radicacin de ndice impar de nmeros negativos. Es decir, las operaciones que se definen en este
conjunto son todas excepto dos:

La divisin por cero
La extraccin de races de ndice par de nmeros negativos.

Sean a , b y c tres nmeros reales cualesquiera. Las propiedades bsicas para la suma y el producto en R son:

1. Cerradura:
+b a R
b a R
2. Asociatividad:
( ) ( ) c b a c b a + + = + +
( ) ( ) c b a c b a =
3. Conmutatividad:
a b b a + = +
a b b a =
4. Elementos neutros
Para la suma es el cero ya que: a a = + 0
Para el producto es el uno ya que: a a = 1
5. Inversos:
Para la suma existe a , llamado opuesto o simtrico tal que ( ) 0 = + a a
Para el producto existe
0
1
a ,
a
, llamado inverso multiplicativo o recproco tal que 1
1
=
|

\
|

a
a
6. Distributividad
La propiedad distributiva del producto sobre la suma es: ( ) c a b a c b a + = +


5
El punto ( ) 2
5
P se ubic de forma aproximada a su valor de 4142 1. . Sin embargo, tambin se puede obtener aplicando el
Teorema de Pitgoras trazando un tringulo cuya base o cateto adyacente es 1 y cuya altura o cateto opuesto es 1 y desde el
origen se traz con un comps un arco de circunferencia en sentido inverso a las manecillas del reloj (por ser negativo). El punto en
que cruza la recta es su representacin en la recta numrica.
R -1 0 3 2 1 4 -2 -3 -4 -5 -6 5 6
( ) 2
1
P |

\
|

2
11
2
P ( ) 0
3
P ( )
4
P ( ) 2
5
P ( ) 75 5
6
. P
R -1 0 3 2 1 4 -2 -3 -4 -5 -6 5 6
R -1 0 3 2 1 4 -2 -3 -4 -5 -6 5 6
( ) 2
1
P |

\
|

2
11
2
P ( ) 0
3
P ( )
4
P ( ) 2
5
P ( ) 75 5
6
. P
Pgina del Colegio de Matemticas de la ENP-UNAM El campo de los nmeros reales Autor: Dr. Jos Manuel Becerra Espinosa
19

Sea la suma de dos nmeros reales: b a + , al dividir por b , se obtiene
1 + = + =
+
b
a
b
b
b
a
b
b a

Sea el producto de dos nmeros reales: b a , al dividir por b , se obtiene
a a
b
b
a
b
b a
= = =

1


Ntese la diferencia de los resultados, esto obedece a que al dividir implcitamente se aplic el inverso
multiplicativo de b . En el segundo caso, la fraccin se convierte en un nmero entero y en el primer caso
la fraccin persiste. Esto significa que
a
b
b a

+
.

Ejemplo.
Dados los nmeros 4 = a y 5 = b , comprobar que
a
b
b a

+


Solucin.
4 8 1 1 8 0
5
5
5
4
5
5 4
= + = + =
+
. .


Los nmeros reales son el conjunto con el que se trabajar en este libro, sin embargo no son los ms
completos, porque si se necesita extraer la raz de un nmero negativo con ndice par, los nmeros
reales no son suficientes, por lo que es necesario definir otro sistema numrico que permita tal operacin.
Este nuevo sistema se definir en el subtema VI.4.


III.6 VALOR ABSOLUTO

El valor absoluto de un nmero real representa la magnitud de dicho nmero. Esta magnitud es la
distancia que existe, sobre la recta numrica, del nmero dado al cero. El valor absoluto se indica
escribiendo el nmero entre barras verticales.

La definicin formal del valor absoluto es:

<

=
0
0
x si x
x si x
x



Por ejemplo, la magnitud de 5 es 5 5 5 = y la magnitud de 5 es 5 5 5 = . Esto se
aprecia en la siguiente figura:




R
-1 0 3 2 1 4 -2 -3 -4 -5 -6 5 6
5 unidades de
magnitud
5 unidades de
magnitud
- 5 = 5 5 = 5 y
R
-1 0 3 2 1 4 -2 -3 -4 -5 -6 5 6
5 unidades de
magnitud
5 unidades de
magnitud
- 5 = 5 5 = 5 y
Pgina del Colegio de Matemticas de la ENP-UNAM El campo de los nmeros reales Autor: Dr. Jos Manuel Becerra Espinosa
20

Ejemplos.
La magnitud de 3 es 3, 3 3 =
La magnitud de
7
4

es
7
4
,
7
4
7
4
=
.

La magnitud de 0 es 0 , 0 0 =
La magnitud de 2 es 2 ,
2 2 =
.

Lo anterior significa que si x es positivo o cero, x es su propio valor absoluto. Si x es negativo,
entonces su opuesto x es el valor absoluto.


III.7 INTERVALOS

A un segmento de la recta numrica que representa al subconjunto de los nmeros reales se le denomina
intervalo.

Si a y b son dos nmeros reales tales que b a < , entonces los intervalos son subconjuntos de R que
formalmente se definen como:

Cerrados. Cuando sus extremos s pertenecen al subconjunto. Se representan como [ ] b , a y
comprende todos los nmeros reales x que cumplen con: [ ] { = x , b x a x b , a R }.






Abiertos. Cuando sus extremos no pertenecen al subconjunto. Se representan como ( ) b , a y
comprende todos los nmeros reales x que cumplen con: ( ) { < < = x , b x a x b , a R }.






Semiabiertos por la izquierda. Cuando el primer extremo no pertenece al subconjunto y el segundo
extremo si. Se representa como ( ] b , a y comprende todos los nmeros reales x que cumplen con:
( ] { < = x , b x a x b , a R }.



R
b x a
a
b
R
b x a
a
b
R
b x a < <
a
b
R
b x a < <
a
b
Pgina del Colegio de Matemticas de la ENP-UNAM El campo de los nmeros reales Autor: Dr. Jos Manuel Becerra Espinosa
21




Semiabiertos por la derecha. Cuando el primer extremo pertenece al subconjunto y el segundo
extremo no. Se representa como [ ) b , a y comprende todos los nmeros reales x que cumplen con:
[ ) { < = x , b x a x b , a R }.






Infinitos. El infinito, ya sea positivo o negativo, no es un extremo determinado, as que siempre ser
abierto. Existen cinco casos:

Cerrados a la izquierda: [ ) { = x , a x x , a R }.






Cerrados a la derecha: ( ] { = x , b x x b , R }.






Abiertos a la izquierda: ( ) { > = x , a x x , a R }.







R
b x a <
a
b
R
b x a <
a
b
R
b x a <
a
b
R
b x a <
a
b
R
a x
a
R
a x
a
R
b x
b
R
b x
b
R
a x >
a
R
a x >
a
Pgina del Colegio de Matemticas de la ENP-UNAM El campo de los nmeros reales Autor: Dr. Jos Manuel Becerra Espinosa
22

Abiertos a la derecha: ( ) { < = x , b x x b , R }.






Abierto completamente: es aquel que contiene a todos los nmeros reales:
( ) { < = x , x x , R }.






Ejemplos.
Graficar los siguientes intervalos:

1) [ ] 5 1,




2) ( ) 2 3,




3) [ ) , 0




4) ( ) 4 ,


R
b x <
b
R
b x <
b
R
< < x
R
< < x
R -1 0 3 2 1 4 -2 -3 -4 -5 -6 5 6
5 1 x
R -1 0 3 2 1 4 -2 -3 -4 -5 -6 5 6
R -1 0 3 2 1 4 -2 -3 -4 -5 -6 5 6
5 1 x
R -1 0 3 2 1 4 -2 -3 -4 -5 -6 5 6
2 3 < < x
R -1 0 3 2 1 4 -2 -3 -4 -5 -6 5 6
R -1 0 3 2 1 4 -2 -3 -4 -5 -6 5 6
2 3 < < x
R -1 0 3 2 1 4 -2 -3 -4 -5 -6 5 6
0 x
R -1 0 3 2 1 4 -2 -3 -4 -5 -6 5 6
R -1 0 3 2 1 4 -2 -3 -4 -5 -6 5 6
0 x
R -1 0 3 2 1 4 -2 -3 -4 -5 -6 5 6
4 < x
R -1 0 3 2 1 4 -2 -3 -4 -5 -6 5 6
4 < x
Pgina del Colegio de Matemticas de la ENP-UNAM El campo de los nmeros reales Autor: Dr. Jos Manuel Becerra Espinosa
23

III.8 LEYES DE EXPONENTES

Sea un nmero real x . Si se multiplica por s mismo se obtiene x x . Si a este resultado se multiplica
nuevamente por x resulta x x x . De manera sucesiva, si x se multiplica por si misma n veces, se
obtiene:
43 42 1
veces n
x x x x

Para simplificar este tipo de expresiones se acostumbra utilizar una notacin abreviada, tal que:

5
4
3
2
x x x x x x
x x x x x
x x x x
x x x
=
=
=
=


y en general:
n
veces n
x x x x x =
43 42 1

Donde x es llamada base y el nmero n escrito arriba y a su derecha, es llamado exponente. El
exponente indica el nmero de veces que la base se toma como factor.

Primera ley de los exponentes

Sea un nmero real x diferente de cero y dos nmeros naturales n y m tambin diferentes de cero.
Entonces, se cumple que:

m n m n
x x x
+
=

Al multiplicar potencias con la misma base, se mantiene la base y se suman los exponentes.

Ejemplos.

1) ( )( )
5 2 3 2 3
x x x x = =
+

2) ( )( )
8 6 2
20 5 4 a a a =
3) ( )( )( )
13 7 2 4
10 5 2 k k k k =
4)
( )
4 3 2 3
6
4
3
8 b a b a ab = |

\
|

5)
10 9 10 9 4 6 5 3
5
1
240
48
12
1
4
8
5
6
q p q p q q p q p = = |

\
|
|

\
|
|

\
|



Segunda ley de los exponentes

Sea un nmero real x diferente de cero y dos nmeros naturales n y m tambin diferentes de cero.
Entonces, se cumple que:

Pgina del Colegio de Matemticas de la ENP-UNAM El campo de los nmeros reales Autor: Dr. Jos Manuel Becerra Espinosa
24

m n
m
n
x
x
x

=


Al dividir potencias con la misma base, se mantiene la base y se restan los exponentes.

Ejemplos.

1)
3 4 7
4
7
x x
x
x
= =


2)
5
3
8
2
5
10
a
a
a
=


3)
2 2
5
3 7
4
7
28
m k
m k
m k
=


4)
2
4
6
3
8
4
1
3
2
a
a
a
=

5)
6 4
2 2
7 6 3
3
2
48
32
z xy
z y x
z y x
=




Tercera ley de los exponentes

Sea un nmero real x diferente de cero. Si en la ley anterior, se hace que m n = , se tiene que:
0
x x
x
x
n n
n
n
= =

.

Pero al dividir una expresin por si misma el resultado es la unidad, as que se cumple que:

1
0
= x

Cualquier base diferente de cero elevada a la potencia cero es uno.

1)
1
0 2 2
2
2
= = =

x x
x
x

2) ( ) 5 1 5 5
0
= = a
3)
( ) 1
0
= xyz

4)
3
9
27
3
3
=
a
a

5)
1
0 13 13
13
13
7 6
6 4 3
= = =

x x
x
x
x x
x x x


Pgina del Colegio de Matemticas de la ENP-UNAM El campo de los nmeros reales Autor: Dr. Jos Manuel Becerra Espinosa
25

Cuarta ley de los exponentes

Sea un nmero real x diferente de cero y dos nmeros naturales n y m tambin diferentes de cero.
Entonces, se cumple que:

( )
m n
m
n
x x

=

Al elevar una potencia a otra potencia, se mantiene la base y se multiplican los exponentes.

Ejemplos.

1) ( )
( ) 6 2 3
2
3
x x x = =
2) ( )
( ) 12 4 3
4
3
a a a =
3) ( )
( ) 15 3 5
3
5
e e e = =


Quinta ley de los exponentes

Sean dos nmeros reales x y y diferentes de cero y un nmero natural n tambin diferente de cero.
Entonces, se cumple que:

( )
n n n
y x xy =

El producto de uno o ms factores que se elevan todos a la vez a un exponente es igual a un producto de
cada factor elevado al exponente.

Ejemplos.

1)
( )
10 10 5
5
2
32 2 2 a a a = =
2) ( ) ( )
12 12 3
3
4
27 3 3 k k k = =
3) ( )
12 4 12 4 4
4
3
625 5 5 b a b a ab = =
4) ( )
6 2 6 2 2
2
2
16 4 4 y x y x xy = =
5) ( )
18 12 30 18 12 30 6
6
3 2 5
000 000 1 10 10 p n m , ' p n m p n m = =


Sexta ley de los exponentes

Sean dos nmeros reales x y y diferentes de cero y un nmero natural n tambin diferente de cero.
Entonces, se cumple que:

0 =
|
|

\
|
y ,
y
x
y
x
n
n
n


El cociente de uno o ms factores que se elevan todos a la vez a un exponente es igual al cociente de
cada factor elevado al exponente.
Pgina del Colegio de Matemticas de la ENP-UNAM El campo de los nmeros reales Autor: Dr. Jos Manuel Becerra Espinosa
26

Ejemplos.

1)
2
2
2
y
x
y
x
=
|
|

\
|

2)
( )
( )
3 3
3 3
3
3 3
d c
b a
cd
ab
cd
ab
= =
|

\
|

3)
( ) ( )
81
625
3
5
3
5
3
5
12
4
4
3 4
4
4
3
4
3
p p p p
= = =
|
|

\
|

4)
( )
( )
8
12
4
2
4
3
4
4
2
3
4
2
3
16 2 2
4
8
m
k
m
k
m
k
m
k
= =
|
|

\
|
=
|
|

\
|

5)
( ) ( ) ( )
( ) ( ) ( )
12 24
30 18
12
2
6
4 6
6
5
6
3 6
6
2 4
5 3
729
096 4
3
4
3
4
z w
y x ,
z w
y x
z w
y x
=

=
|
|

\
|



Sptima ley de los exponentes

Sea un nmero real x diferente de cero. Si n es un nmero entero diferente de cero, por las leyes
anteriores se cumple que:

1
0
= = = =
n n n n
n
n
x x x x
x
x


Pero el recproco del nmero real
n
x se defini como
n
x
1
, ya que cumple con
1
1
=
n
n
x
x
.
Comparando las expresiones, se llega a:

n
n
x
x
1
=



Elevar una expresin a una potencia entera negativa, equivale a formar una fraccin con numerador uno
y cuyo denominador es la misma expresin pero con la potencia positiva.

Ejemplos.

1)
x
x
1
1
=


2)
3
3
6
6
a
a =


3)
5 4
5 4
10 7
5 3
8
8
3
24
q p
q p
q p
q p
= =



4)
c a
b
c b a
bc a
c b a
6
2
1 2 6
5 11
4 3 5
2
3
2
3
18
27
= =


5)
( )
12 12 4
12 4
4
3
16
1 1
2
1
2 2
x x
x x = = =


Pgina del Colegio de Matemticas de la ENP-UNAM El campo de los nmeros reales Autor: Dr. Jos Manuel Becerra Espinosa
27

III.9 NOTACIN CIENTFICA

En las ciencias es frecuente encontrar cantidades muy grandes o muy pequeas que contienen una gran
cantidad de ceros.

Ejemplos.

1) La distancia media de la Tierra al Sol es de 00 1496000000 metros.
2) Un mililitro es la milsima parte de una millonsima de metro cbico, es decir:
3
000000001 0 1 m . ml = .

A fin de evitar escribir tanta cifra, estos nmeros se pueden compactar mediante la notacin cientfica.

La notacin cientfica es un nmero escrito bajo la forma
n
A 10 , donde
10 1 < A
y n es un entero.
Para transformar un nmero a notacin cientfica, existen dos casos:

Cuando el nmero es mayor o igual a diez, el punto decimal se recorre n posiciones a la
izquierda y el exponente es positivo

Cuando el nmero es menor de uno, el punto decimal se recorre n posiciones a la derecha y el
exponente es negativo

En los ejemplos planteados, la distancia que separa a la Tierra del Sol se puede expresar como
11
10 496 1 . metros, por su parte, un mililitro equivale a
9
10 1

metros cbicos.

Ejemplos.
Expresar los siguientes nmeros en notacin cientfica:

1) 7540000000
Solucin.
Se recorre el punto decimal nueve posiciones a la izquierda:
9
10 54 7 7540000000 = .

2) 7926 0000000000 0.
Solucin.
Moviendo el punto decimal once posiciones a la derecha:
11
10 926 7 7926 0000000000 0

= . .

Para transformar un nmero expresado en notacin cientfica a notacin normal, existen dos casos:

Cuando 1 > n , se le agregan ceros hasta que el punto decimal recorra n posiciones a la derecha.
Cuando 1 < n , se le agregan ceros hasta que el punto decimal recorra n posiciones a la izquierda.

Ejemplos.
Convertir los siguientes nmeros expresados en notacin cientfica a notacin normal:

1)
6
10 7 2 .
Solucin.
Se recorre el punto decimal seis posiciones a la derecha y se agregan ceros:
2700000 10 7 2
6
= .

2)
9
10 18 3

.
Solucin.
Pgina del Colegio de Matemticas de la ENP-UNAM El campo de los nmeros reales Autor: Dr. Jos Manuel Becerra Espinosa
28

Moviendo el punto decimal nueve posiciones a la izquierda y agregando ceros:



III.10 LOGARITMOS

Sea la expresin: , con 0 > a y 1 a .

Se denomina logaritmo base del nmero al exponente b al que hay que elevar la base para
obtener dicho nmero. Es decir:



que se lee como "el logaritmo base del nmero es y como se puede apreciar, un logaritmo
representa un exponente.

La constante a es un nmero real positivo distinto de uno, y se denomina base del logaritmo. La
potencia
b
a para cualquier valor real de solo tiene sentido si 0 > a .

Ejemplos.

1) 25 5
2
= 2 25
5
= log
2) 81 3
4
= 4 81
3
= log
3) 512 8
3
= 3 512
8
= log
4)
64
1
2
1
6
= |

\
|

6
64
1
2
1
= log

5)
1024
1
4
5
=


5
1024
1
4
= log


Logaritmos Decimales:

Se llaman logaritmos decimales a los logaritmos que tienen por base el nmero diez. Al ser muy
habituales es frecuente no escribir la base:



Logaritmos Naturales:

Se llaman logaritmos naturales (tambin llamados neperianos) a los logaritmos que tienen por base el
nmero irracional = 59 7182818284 2. e , y se denotan como ln o por L :

x L x ln x log
e
= =

Ejemplos.
653212 1 45 45
10
. log log =
123963 5 168 168 . ln log
e
=
8 0000000031 0 10 18 3
9
. . =

x a
b
=
a x
b x log
a
=
a x b
b
x log x log =
10
Pgina del Colegio de Matemticas de la ENP-UNAM El campo de los nmeros reales Autor: Dr. Jos Manuel Becerra Espinosa
29

Para potencias enteras de diez, los logaritmos decimales cumplen con:
2 01 0 01 0 10
2
= =

. log .
1 1 0 1 0 10
1
= =

. log .
0 1 1 10
0
= = log
1 10 10 10
1
= = log
2 100 100 10
2
= = log
3 000 1 000 1 10
3
= = , log ,
4 000 10 000 10 10
4
= = , log ,

Los logaritmos decimales de los nmeros comprendidos entre otros dos, cuyos logaritmos decimales son
nmeros enteros, son nmeros decimales. Todo nmero decimal se compone de parte entera y parte
decimal. La parte entera recibe el nombre de caracterstica y la parte decimal, mantisa.

La parte entera del logaritmo o caracterstica depende del intervalo en el que se defina el nmero y la
parte decimal o mantisa del valor de las cifras significativas del nmero.

Por ejemplo, para = 653212 1 45 . log , la caracterstica es y la mantisa es 653212 0. .

La mantisa siempre es positiva, pero la caracterstica puede ser cero si el nmero est comprendido
entre y 10 , es positiva, s el nmero es mayor que o negativa si el nmero es menor que 1. Las
potencias de slo tienen caracterstica, su mantisa es 0 . En el logaritmo de un nmero menor que
1 la caracterstica es negativa, pero la mantisa es positiva. Por ejemplo 698970 0 1 5 0 . . log + y
no puede escribirse como 698970 1. , pues esto indica que tanto la caracterstica como la mantisa
son negativas. El modo correcto de escribirlo, indicando que slo la caracterstica es negativa, es
698970 1. .

Ejemplos.

1) Para 795184 2 624 . log , la caracterstica es 2
2) Para 845098 0 7 . log , la caracterstica es 0
3) Para 462398 2 029 0 . . log , la caracterstica es 2

Las propiedades de los logaritmos son las siguientes:

1)
2) 1 = a log
a

3)
( ) v log u log v u log
a a a
+ =
4)
v log u log
v
u
log
a a a
= |

\
|

5) u log n u log
a
n
a
=
6) u log
n
u log
a
n
a
1
=

1
1 10
10
0 1 =
a
log
Pgina del Colegio de Matemticas de la ENP-UNAM El campo de los nmeros reales Autor: Dr. Jos Manuel Becerra Espinosa
30

Ejemplos.
Comprobar las propiedades de los logaritmos.

1) 0 1 10
0
= = log log
2) 1 10 = log
3)
( ) 5 000 100 000 1 100 = = , log , log
que equivale a calcular: 5 3 2 000 1 100 = + = + , log log
4)
4 000 10
100
000 000 1
= = |

\
|
, log
, '
log

que equivale a calcular: 4 2 6 100 000 000 1 = = log , ' log
5) 2 100 10
2
= = log log
que equivale a calcular:
( ) 2 1 2 10 2 = = log
6) 2 100 000 10 = = log , log
que equivale a calcular:
( ) 2 4
2
1
000 10
2
1
= = , log


Ejemplo.
Aplicando las propiedades de los logaritmos, simplificar la siguiente expresin:
( )( )
4
6
2
3 5
(

c
b a
log

Solucin.
( )( ) ( )( )
( )( ) [ ] ( ) c log b log a log c log c a log
c
b a
log
c
b a
log 2 3 5 4 2 3 5 4
2
3 5
4
2
3 5
6 6 6 6 6 6
4
6
+ = = =
(



Ejemplo.
Sabiendo que 2 100 = log y que 6020 0 4 . log , aplicando las propiedades de los logaritmos y sin
usar la calculadora, determinar los valores aproximados de: 400 log , 25 log , 16 log , 2 log .

Solucin.
( )( ) 6020 2 6020 0 2 4 100 4 100 400 . . log log log log + + = =
398 1 0620 0 2 4 100
4
100
25 . . log log log log = =
( ) 204 1 0620 0 2 4 2 4 16
2
. . log log log = =
3010 0
2
6020 0
4
2
1
4 2 .
.
log log log = =

Un antilogaritmo es el nmero que corresponde a un logaritmo dado. Consiste en el problema inverso al
clculo del logaritmo de un nmero. Esto es:

x a x y log anti y x log
y
a a
= = =

es decir, consiste en elevar la base al nmero que resulta.

Pgina del Colegio de Matemticas de la ENP-UNAM El campo de los nmeros reales Autor: Dr. Jos Manuel Becerra Espinosa
31

Ejemplo.
527 4 10 527 4 655810 3 655810 3 527 4
655810 3
10 10
, , . log anti . , log
.


Cambio de Base:

Dada una base conocida b , para calcular un logaritmo de un nmero x en cualquier base a , se aplica
la siguiente expresin:
a log
x log
x log
b
b
a
=
.

Por conveniencia, la base elegida para b generalmente es la diez, as que la expresin queda como:

a log
x log
x log
a
10
10
=

Ejemplo.
Calcular: 570
3
log

Solucin: se identifican las variables: 10 570 3 = = = b , x , a
776048 5
477121 0
755874 2
3
570
570
3
.
.
.
log
log
log =
Comprobacin: 570 3
776048 5

.



Pgina del Colegio de Matemticas de la ENP-UNAM Operaciones con monomios y polinomios Autor: Dr. Jos Manuel Becerra Espinosa
1

OPERACIONES CON MONOMIOS Y
POLINOMIOS

UNIDAD IV


IV.1 OPERACIONES CON MONOMIOS

Una variable es un elemento de una frmula, proposicin o algoritmo que puede adquirir o ser sustituido
por un valor cualquiera.

Un coeficiente es un factor multiplicativo que pertenece a una variable.

Una constante es un valor fijo, aunque a veces no determinado.

Expresiones algebraicas son todas aquellas que combinan constantes y variables mediante operaciones.

Ejemplos.

1)
4 3 2
9 z y x , el coeficiente es 9 y las variables son
4 3 2
z y x
2)
4
8 5
7
2
3
4
d
c
b a +
, los coeficientes son
3
4

y
7
2
; las variables son
8 5
b a y
4
d
c


Un trmino algebraico es cada sumando de una expresin algebraica.

Los trminos poseen grados de dos tipos:

Grado absoluto. Es la suma de los exponentes de las literales que forman al trmino.

Grado relativo. Es aquel exponente que tiene una literal especfica.

Ejemplos.

1) En el trmino
4 3 2
5 z y x , el grado absoluto es 9 y el grado relativo de la literal x es 2 .
2) En el trmino
6 5
7 bc a , el grado absoluto es 12 y el grado relativo de la literal b es 1.

Se define como monomios a las expresiones algebraicas que constan de un solo trmino.

Ejemplos.

1) c b a
2 4
5
2)
( )
4
3 3
11
2
y x

3)
7
5 a

El valor numrico de un monomio es el nmero que se obtiene al sustituir las literales por valores
especficos, despus de efectuar las operaciones indicadas.


Pgina del Colegio de Matemticas de la ENP-UNAM Operaciones con monomios y polinomios Autor: Dr. Jos Manuel Becerra Espinosa
2

Ejemplos.

1) Si en el monomio b a
2
4 , las literales toman los valores 2 = a y 3 = b , su valor numrico es:
( )( ) 48 3 2 4
2
=

2) Si en el monomio
2 3
3
4
yz x
, las literales toman los valores 1 = x , 9 = y y
2
1
= z , su valor
numrico es:
( ) 3
2
1
9 1
3
4
2
3
=




Trminos semejantes. Son aquellos que tienen la parte literal igual. Dos o ms trminos son semejantes
cuando tienen la misma parte literal, es decir, las mismas literales elevadas a los mismos exponentes.

Ejemplos.

1)
2
3x y
2
7x son trminos semejantes
2)
3 4 2
2
5
np m k
y
4 3 2
12 m p nk son trminos semejantes
3) b a
2
2 y
2
6ab no son trminos semejantes


Suma de monomios

Para sumar monomios tienen que ser semejantes. El resultado es un monomio semejante a ellos que
tiene por coeficiente la suma de los coeficientes de cada monomio.

Ejemplos.
Sumar los siguientes monomios:

1)
4 4 4 4 4
19 4 8 2 5 x x x x x = + + +
2) c b a b ca c b a c b a
5 2 5 2 5 2 5 2
10 2 7 = + +
3)
3 3 3 3
12
7
4
5
2
1
3
4
yz yz yz yz =

+ +



Resta de monomios

Para restar monomios tambin es necesario que sean semejantes. El resultado es un monomio
semejante a ellos que tiene por coeficiente la resta de los coeficientes de cada monomio.

Ejemplos.

1)
2 2 2 2 2
4 2 4 11 x x x x x =
2)
3 4 4 3 3 4 3 4
7 12 10 15 m k k m m k m k =
3) c ab acb c ab c ab
2 2 2 2
10
11
2
2
1
5
2
=



Pgina del Colegio de Matemticas de la ENP-UNAM Operaciones con monomios y polinomios Autor: Dr. Jos Manuel Becerra Espinosa
3

Multiplicacin de monomios

Para multiplicar monomios no es necesario que sean semejantes. Una vez que se aplican las leyes de los
exponentes que se requieran, se multiplican los coeficientes y se suman los exponentes de cada literal.

Ejemplos.

1) ( )( )
8 5 3
10 5 2 x x x =
2) ( )( )( )
6 3 8 4 5 5 2 2 3 2
84 7 3 4 h g f e h f hg e g f e =
3) ( ) ( ) ( ) ( )( )( )
25 20 16 12 3 6 6 2
4
4 3
3
2
2
3
000 2 16 125 2 5 z y , z y z y z y z y z y yz = =


Divisin de monomios

Para dividir dos monomios, tampoco es necesario que sean semejantes. Una vez que se aplican las
leyes de los exponentes que se requieran, se dividen los coeficientes y se restan los exponentes de cada
literal.

Ejemplos.

1)
3
2
5
2
6
12
a
a
a
=
2)
z x
z y x
z y x
2
5 2
2 5 4
4
16
64
=

3)
4
3 3
3 4 3
7 2
4 3 5
6
6
8
48
m
n k
n m k
n m k
n m k
= =




IV.2 OPERACIONES CON POLINOMIOS

Un polinomio en x de grado n es una expresin del tipo:

( )
n
n o
x a x a x a x a x a a x P + + + + + + = L
4
4
3
3
2
2 1


donde n N y
n o
a , , a , a , a L
2 1
son coeficientes reales y se lee como P de x .

El grado de un polinomio con respecto a una literal es el mayor exponente de sus trminos.

Ejemplos.

1)
3 2
8 6 2 5 x x x + + el grado es 3
2)
2 4 3
10 1 2 8 2 x x x x + + el grado es 4
3)
2 3 5 2 4 3
5 7 8 12 7 14 xm m x m x m m x + + + + el grado con respecto a x es 5

Para ordenar un polinomio con respecto a una literal, se puede efectuar de manera descendente
(posicionndola de mayor a menor grado) o de forma ascendente (ubicndola de menor a mayor grado).

Ejemplos.
Pgina del Colegio de Matemticas de la ENP-UNAM Operaciones con monomios y polinomios Autor: Dr. Jos Manuel Becerra Espinosa
4

1) El polinomio x x x x 10 5 6 9 2
3 4 2
+ + ordenado de forma descendente es:
9 10 2 5 6
2 3 4
+ + x x x x

2) El polinomio
3 3 2 2
5 7 12 8 xy y x y x + + ordenado de forma ascendente con respecto a x es:
y x y x xy
3 2 2 3
7 8 5 12 + +

Completar un polinomio es aadir los trminos intermedios que falten poniendo de coeficiente 0 .

Ejemplo.

El polinomio
4 6 3
5 13 9 2 8 x x x x ordenado de forma descendente y completndolo es:
2 9 0 8 5 0 13
2 3 4 5 6
+ + + x x x x x x

Suma de polinomios

Para sumar polinomios se suprimen los signos de agrupacin precedidos del signo ( ) + , dejando el mismo
signo de cada uno de los trminos que se hallan dentro de l y se simplifican los trminos que sean
semejantes.

Ejemplos.

1) ( ) ( ) 4 9 11 2 4 7 3 5 11 2 4 7 3 5
2 2 2 2 2
= + + + = + + + x x x x x x x x x x
2) ( ) ( ) k k k k k k k k k k k k k k 4 2 12 5 8 7 3 6 4 2 12 5 8 7 3 6
5 3 2 4 2 5 3 2 4 2
+ + + + = + + + +
8 2 8 12 7 2
2 3 4 5
+ + + = k k k k k
3) ( ) ( ) ( ) 2 7 11 9 3 5 8 4 1 6
3 3 3 3 2 2 2 2 3 3
+ + + + + + ab b a b a ab b a b a b a ab
12 4 13 6 2 7 11 9 3 5 8 4 1 6
3 2 2 3 3 3 3 3 2 2 2 2 3 3
+ + = + + + + + + = ab b a b a ab b a b a ab b a b a b a ab
4)

+ +

+ + +

+ x x x x x x
5
6
1
4
3
2
11
5
8
4
2
5
3
4
6
7
2 2 2


3
17
15
58
20
97
5
6
1
4
3
2
11
5
8
4
2
5
3
4
6
7
2 2 2 2
+ + = + + + + + + = x x x x x x x x

Resta de polinomios

Para restar polinomios se suprimen los signos de agrupacin precedidos del signo (-), cambiando el signo
de cada uno de los trminos del sustraendo y se simplifican los trminos que sean semejantes.

Ejemplos.

1) ( ) ( ) 5 6 2 7 2 5 4 9 5 6 2 7 2 5 4 9
2 3 2 3 2 3 2 3
+ + + + = + + + x x x x x x x x x x x x
7 11 6 2
2 3
+ + = x x x
2) ( ) ( ) a a a a a a a a a 3 9 4 5 7 3 14 4 9 2 5
4 6 3 2 3 5 4
+ + + +
a a a a a a a a a 3 9 4 5 7 3 14 4 9 2 5
4 6 3 2 3 5 4
+ + + + + =
21 2 3 9 11 9 4
2 3 4 5 6
+ + + = a a a a a a
3) ( ) ( ) ( ) p k p k kp p k p k kp p k p k
2 2 3 4 2 2 3 4 2 3 2
3 6 15 5 2 4 4 8 4 10 5 + + + +
Pgina del Colegio de Matemticas de la ENP-UNAM Operaciones con monomios y polinomios Autor: Dr. Jos Manuel Becerra Espinosa
5

p k p k kp p k p k kp p k p k
2 2 3 4 2 2 3 4 2 3 2
3 6 15 5 2 4 4 8 4 10 5 + + + + + =
7 13 12 14
4 2 2 3
+ + = kp p k p k
4)

+ 7
3
1
4
11
2
9
5
12
3
4
7
8
6
5
3
2
2 2 2
x x x x x x

15
59
7
1
12
31
7
3
1
4
11
2
9
5
12
3
4
7
8
6
5
3
2
2 2 2 2
+ = + + + + x x x x x x x x

Producto de un monomio por un polinomio

Para multiplicar un monomio por un polinomio se multiplican todos los trminos del polinomio por el
monomio, es decir, es una suma de producto de monomios.

Ejemplos.

1) ( ) ( ) ( ) ( ) ( ) ( ) 8 2 2 2 7 2 3 2 5 2 8 2 7 3 5 2
2 2 2 2 3 2 4 2 2 3 4 2
x x x x x x x x x x x x x x + + = + +

2 3 4 5 6
16 4 14 6 10 x x x x x + + =
2) ( )( )
2 2 5 3 4 2 3
7 3 6 2 10 9 5 b a b a b a ab b a + + +

4 3 2 4 2 3 3 5 7 6 6 4
35 15 30 10 50 45 b a b a b a b a b a b a + + =
3)
( )
3 2 3 4 2 5 3 4 3 4 2
10 22 6 12 8 4
2
3
eh gh e h f g e h ef g f e + +



3 3 4 3 2 4 4 2 3 4 5 4 3 6 2 8 4 5 3 8 3
15 33 9 18 12 6 h g f e h g f e g f e h g f e g f e h g f e + + =
4)
5 4 3 5 4 3 3 2 2
5
2
15
15
6
15
15
2
3
1
5 3 a a a a a a a a a a + = + =

+

Multiplicacin de dos polinomios

Para multiplicar dos polinomios se aplica la propiedad distributiva del producto sobre la suma, esto es, se
multiplican todos los trminos del segundo polinomio por cada uno de los trminos del primero y se
reducen los trminos semejantes. La multiplicacin de polinomios es distributiva respecto a la adicin.

Ejemplos.

1) ( )( ) 12 42 24 10 35 20 6 21 12 2 7 4 6 5 3
2 2 3 2 3 4 2 2
+ + + + = + + x x x x x x x x x x x x
12 52 65 41 12
2 3 4
+ + = x x x x
2) ( )( )
4 2 2 3 3 2 2 4 2 2 2 2
36 144 48 192 16 64 4 16 9 12 4 b b a ab b a b a a b a b ab a + + = +

4 3 2 2 3 4
36 48 128 192 64 b ab b a b a a + + =
3) ( )( )
4 4 7 2 2 2 4 3 4 6 2 2 3 4 3 2
25 15 2 15 10 6 6 5 3 1 5 2 z y z y z y yz z y z y y z z y yz yz yz + = +
y z z y z y yz + + + +
2 3 3 2 5
6 5 5 30

Divisin de un polinomio por un monomio

Para dividir un polinomio por un monomio, se divide cada trmino del dividendo por el divisor, es decir, es
una suma de cociente de monomios.


Pgina del Colegio de Matemticas de la ENP-UNAM Operaciones con monomios y polinomios Autor: Dr. Jos Manuel Becerra Espinosa
6

Ejemplos.

1) x x x x
x
x
x
x
x
x
x
x
x
x x x x
7 4 2 5
12
84
12
48
12
24
12
60
12
84 48 24 60
2 3 4
2
3
2
4
2
5
2
6
2
3 4 5 6
+ = + =
+

2) 3 13 18 8
5
15 5 65 90 40
4 2 2 3 3
3
3 5 3 2 5 3 6 4 3
+ + + =

+
y y w y w y
y w
y w y w y w y w y w

3)
3 4 2
3 4 4 5 4 6 6 5 3 3 7 6 4 4 5
6
90 24 36 60 54
r q p
r q p r q p r q p r q p r q p


2 2 4 3 3 4 3
15 4 6 10 9 p r p pqr q p r p =

Cociente de dos polinomios

Para dividir dos polinomios se efecta el siguiente procedimiento:

Se ordenan los polinomios de forma descendente con respecto al grado de una misma variable.
Se divide el primer trmino del dividendo por el primer trmino del divisor y se obtiene el primer
trmino del cociente.
Se resta del dividendo el producto del primer trmino del cociente por el divisor y se obtiene el primer
residuo (esto implica cambiar todos los signos del producto efectuado y reducir trminos semejantes
con el dividendo).
Se bajan los trminos restantes del dividendo sumndolos al residuo anterior.
Se divide el primer trmino del residuo por el primer trmino del divisor, obteniendo as el segundo
trmino del cociente.
Se procede de forma anloga hasta obtener un residuo nulo o de grado inferior al del divisor.
Comprobar el resultado mediante el algoritmo: ( )( ) dividendo residuo divisor cociente = +

Ejemplos.

1) Dividir 9 5 25 6
2 3 4
+ x x x x por 3 + x

Solucin.
11 2 9
42
33 11
9 11
6 2
9 5 2
27 9
9 5 25 9
3
9 5 25 6 3
2 3
2
2
2 3
2 3
3 4
2 3 4
+
+
+

+
+
+

+ +
x x x
x
x
x x
x x
x x
x x x
x x
x x x x x


Comprobacin: ( )( ) 42 33 6 27 3 11 2 9 42 11 2 9 3
2 3 2 3 4 2 3
+ + + + = + + + x x x x x x x x x x x
9 5 25 6
2 3 4
+ = x x x x

Pgina del Colegio de Matemticas de la ENP-UNAM Operaciones con monomios y polinomios Autor: Dr. Jos Manuel Becerra Espinosa
7

2) Dividir 1 4 2 3 2
2 3 4
+ + + x x x x por 1
2
+ x x

Solucin.

1 5 2
0
1
1
5 5 5
1 4 4 5
2 2 2
1 4 2 3 2 1
2
2
2
2 3
2 3
2 3 4
2 3 4 2
+ +
+
+
+
+ +
+
+ + + +
x x
x x
x x
x x x
x x x
x x x
x x x x x x


Comprobacin: ( )( ) 0 1 5 2 5 2 5 2 0 1 5 2 1
2 2 3 2 3 4 2 2
+ + + + + + = + + + + x x x x x x x x x x x x
1 4 2 3 2
2 3 4
+ + + = x x x x


3) Dividir 8
3
+ x por 2 + x

Solucin.
Completando el polinomio y efectuando la divisin:

4 2
0
8 4
8 4
4 2
8 0 2
2
8 0 0 2
2
2
2
2 3
2 3
+

+
+
+ +

+ + +
x x
x
x
x x
x x
x x
x x x x


Comprobacin: ( )( ) 8 0 8 4 2 4 2 0 4 2 2
3 2 2 3 2
+ = + + + + = + + + x x x x x x x x x
8
3
+ = x


4) Dividir 9 14 22 30
2 3
+ k k k por 3 5 + k

Solucin.

Pgina del Colegio de Matemticas de la ENP-UNAM Operaciones con monomios y polinomios Autor: Dr. Jos Manuel Becerra Espinosa
8

2 8 6
3
6 10
9 10
24 40
9 14 40
18 30
9 14 22 30 3 5
2
2
2
2 3
2 3
+

+
+
+

+ +
k k
k
k
k k
k k
k k
k k k k

Comprobacin: ( )( ) 3 6 24 18 10 40 30 3 2 8 6 3 5
2 2 3 2
+ + + + = + + + k k k k k k k k
9 14 22 30
2 3
+ = k k x


5) Dividir
3 2 2 3
4 22 4 30 b ab b a a + + por b a 4 6

Solucin.
La divisin se ejecutar respecto a la variable a :

2 2
3 2
3 2
2 2
3 2 2
2 3
3 2 2 3
4 5
0
4 6
4 6
16 24
4 22 24
20 30
4 22 4 30 4 6
b ab a
b ab
b ab
ab b a
b ab b a
b a a
b ab b a a b a
+

+
+
+
+
+ +


Comprobacin: ( )( )
3 2 2 2 2 3 2 2
4 16 20 6 24 30 0 4 5 4 6 b ab b a ab b a a b ab a b a + + = + +

3 2 2 3
4 22 4 30 b ab b a a + + =


IV.3 VALOR Y GRFICA DE POLINOMIOS EN UNA SOLA VARIABLE

Dado un polinomio de la forma:

( )
n
n
n
n o
x a x a x a x a x a a x P + + + + + + =

1
1
3
3
2
2 1



Se conoce como valor de un polinomio ( )
n
n
n
n o
x a x a x a x a x a a x P + + + + + + =

1
1
3
3
2
2 1
para
c x = , al valor numrico que toma el polinomio cuando se sustituye la variable, x , por el nmero c y se
realizan las operaciones. Se denota como ( ) c P y se lee P de c .
Pgina del Colegio de Matemticas de la ENP-UNAM Operaciones con monomios y polinomios Autor: Dr. Jos Manuel Becerra Espinosa
9

Ejemplos.

1) Evaluar el polinomio ( ) 14 8 5
2
+ + = x x x P para 3 = x .
Solucin.

( ) ( ) ( ) 7 14 24 45 14 3 8 3 5 3
2
= + + = + + = P

2) Evaluar el polinomio ( ) 5 6 7 4
2 3 4
+ + + + = x x x x x P para 2 = x .
Solucin.

( ) ( ) ( ) ( ) ( ) 5 5 12 28 32 16 5 2 6 2 7 2 4 2 2
2 3 4
= + + = + + + + = P

3) Evaluar el polinomio ( ) 2 7 10 8
2 3
+ + = x x x x P para
4
1
= x
.
Solucin.
= + + = + + = +

2
4
7
8
5
8
1
2
4
7
16
10
64
8
2
4
1
7
4
1
10
4
1
8
4
1
2 3
P
1
8
8
8
16 14 5 1
= =
+ +
=

Como se defini en el subtema I.6, el plano cartesiano es un sistema formado por dos ejes numricos
reales perpendiculares donde su origen es el punto en que se cruzan. El eje horizontal ( ) x recibe el
nombre de eje de las abscisas y el eje vertical ( ) y recibe el nombre de eje de las ordenadas.

La grfica de un polinomio est formada por el conjunto de parejas coordenadas ( ) y , x que cumplen o
satisfacen la regla de correspondencia ( ) x P .

Los polinomios ( ) x P pueden evaluarse para todo x R y por ello se unen los puntos obtenidos para
obtener sus grficas.

Para fines prcticos, para valores diferentes de x se pueden obtener los valores de ( ) x P , generando
puntos de coordenadas ( ) [ ] x P , x que se localizan en el plano coordenado y que al unirse conforman su
grfica.

La variable x recibe el nombre de variable independiente y a ( ) x P se le conoce como variable
dependiente, es decir, que est en funcin de la variable x .

Ejemplo.
Tabular y graficar los siguientes polinomios en los intervalos pedidos:

1) ( ) 6
2
= x x x P en el intervalo
[ ] 6 5,
Solucin.
Tabulando con los valores enteros del intervalo:




Pgina del Colegio de Matemticas de la ENP-UNAM Operaciones con monomios y polinomios Autor: Dr. Jos Manuel Becerra Espinosa
10

x ( ) x P
5
( ) ( ) 24 6 5 25 6 5 5
2
= + =
4 ( ) ( ) 14 6 4 16 6 4 4
2
= + =
3
( ) ( ) 6 6 3 9 6 3 3
2
= + =
2
( ) ( ) 0 6 2 4 6 2 2
2
= + =
1
( ) ( ) 4 6 1 1 6 1 1
2
= + =
0
( ) 6 6 0 0 6 0 0
2
= =
1
6 6 1 1 6 1 1
2
= =
2
4 6 2 4 6 2 2
2
= =
3
0 6 3 9 6 3 3
2
= =
4
6 6 4 16 6 4 4
2
= =
5
14 6 5 25 6 5 5
2
= =
6
24 6 6 36 6 6 6
2
= =

x
15
-4 6
10
-5
-10
4
20
25
-6 2 -2 5 3 1 -3 -5 -1
5
y



2) ( ) 8 9 6
2 3
+ + = x x x x P en el intervalo
[ ] 5 1,
Solucin.
Tabulando con los valores enteros del intervalo:

x ( ) x P
1 ( ) ( ) ( ) 24 8 9 6 1 8 1 9 1 6 1
2 3
= + + + = + +
0
( ) ( ) 8 8 0 0 0 8 0 9 0 6 0
2 3
= + + = + +
1
( ) ( ) ( ) 4 8 9 6 1 8 1 9 1 6 1
2 3
= + + = + +
2 ( ) ( ) ( ) 6 8 18 24 8 8 2 9 2 6 2
2 3
= + + = + +
3
( ) ( ) ( ) 8 8 27 54 27 8 3 9 3 6 3
2 3
= + + = + +
4 ( ) ( ) ( ) 4 8 36 96 64 8 4 9 4 6 4
2 3
= + + = + +

5
( ) ( ) ( ) 12 8 45 150 125 8 5 9 5 6 5
2 3
= + + = + +

Pgina del Colegio de Matemticas de la ENP-UNAM Operaciones con monomios y polinomios Autor: Dr. Jos Manuel Becerra Espinosa
11

x
16
-8
y
2 5
-16
8
4
12
-4
-12
1 4
20
24
-1 3



3) ( ) 5 9
2 4
+ = x x x P en el intervalo
[ ] 3 3,

Solucin.
Tabulando con los valores enteros del intervalo:

x ( ) x P
3
( ) ( ) 5 5 81 81 5 3 9 3
2 4
= + = +
2 ( ) ( ) 15 5 36 16 5 2 9 2
2 4
= + = +

1 ( ) ( ) 3 5 9 1 5 1 9 1
2 4
= + = +
0
( ) ( ) 5 5 0 0 5 0 9 0
2 4
= + = +
1
( ) ( ) 3 5 9 1 5 1 9 1
2 4
= + = +
2 ( ) ( ) 15 5 36 16 5 2 9 2
2 4
= + = +

3
( ) ( ) 5 5 81 81 5 3 9 3
2 4
= + = +

x
10
-5
y
-3 -2 -1 1 2 3
-10
5
-15

Pgina del Colegio de Matemticas de la ENP-UNAM Operaciones con monomios y polinomios Autor: Dr. Jos Manuel Becerra Espinosa
12

4) ( ) 9
2
+ = x x P en el intervalo [ ] 4 4,
Solucin.
Tabulando en con los valores enteros del intervalo:

x ( ) x P
4 ( ) 7 9 16 9 4
2
= + = +

3
( ) 0 9 9 9 3
2
= + = +
2
( ) 5 9 4 9 2
2
= + = +
1
( ) 8 9 1 9 1
2
= + = +
0
( ) 9 9 0 9 0
2
= + = +
1
( ) 8 9 1 9 1
2
= + = +
2
( ) 5 9 4 9 2
2
= + = +
3
( ) 0 9 9 9 3
2
= + = +
4 ( ) 7 9 16 9 4
2
= + = +



x
6
-6
y
-2 3
-10
2
-2
4
-4
-8
-1 2
8
10
-3 1 -4 4




Pgina del Colegio de Matemticas de la ENP-UNAM Productos notables y factorizacin Autor: Dr. Jos Manuel Becerra Espinosa
1

PRODUCTOS NOTABLES Y FACTORIZACIN

UNIDAD V


V.1 PRODUCTOS NOTABLES

Tanto en la multiplicacin algebraica como en la aritmtica se sigue un algoritmo cuyos pasos conducen
al resultado. Sin embargo, existen productos algebraicos que responden a una regla cuya aplicacin
simplifica la obtencin del resultado. Estos productos reciben el nombre de productos notables.

Se llama producto notable al que puede ser obtenido sin efectuar la multiplicacin trmino a trmino. A
continuacin se describen los ms importantes.


V.1.1 CUADRADO DE UN BINOMIO

El producto de un binomio por s mismo recibe el nombre de cuadrado de un binomio.

El desarrollo del cuadrado del binomio b a + se puede obtener multiplicando trmino a trmino:

( ) ( )( )
2 2 2 2 2
2 b ab a b ba ab a b a b a b a + + = + + + = + + = +

El cuadrado de un binomio b a + es igual al cuadrado del primer trmino ms el doble del producto de
los trminos ms el cuadrado del segundo trmino.

Ahora, al elevar al cuadrado el binomio b a , tambin multiplicando trmino a trmino, se obtiene:

( ) ( )( )
2 2 2 2 2
2 b ab a b ba ab a b a b a b a + = + = =

El cuadrado de un binomio b a es igual al cuadrado del primer trmino menos el doble del producto
de los trminos ms el cuadrado del segundo trmino.

En las frmulas anteriores a y b pueden ser cualquier expresin algebraica y tener cualquier signo. Por
lo tanto, segunda la frmula es un caso particular de la primera ya que:
( ) ( ) [ ] ( )
2 2 2 2 2 2
2 2 b ab a b b a a b a b a + = + + = + =

Ejemplos.

1) ( ) ( )( ) 16 8 4 4 2 4
2 2 2 2
+ + = + + = + a a a a a
2) ( ) ( ) ( )( ) ( )
2 2 2 2 2
9 12 4 3 3 2 2 2 3 2 y xy x y y x x y x + + = + + = +
3) ( ) ( )( ) 25 10 5 5 2 5
2 2 2 2
+ = + + = b b b b b
4) ( ) ( ) ( )( ) ( )
2 2 2 2 2
64 96 36 8 8 6 2 6 8 6 m km k m m k k m k + = + + =
5)
2 2
2 2 2
16
25
3
5
9
4
4
5
4
5
3
2
2
3
2
4
5
3
2
b ab a b b a a b a + + =

+
6) ( ) ( ) ( )( ) ( )
6 3 2 4
2
3 3 2
2
2
2
3 2
81 126 49 9 9 7 2 7 9 7 q q p p q q p p q p + = + + =
7) ( ) ( ) ( )( ) 25 20 4 5 5 2 2 2 5 2
2 2 2 2
+ = + + = + k k k k k
Pgina del Colegio de Matemticas de la ENP-UNAM Productos notables y factorizacin Autor: Dr. Jos Manuel Becerra Espinosa
2

b
a
b
2
a ab
2
b
( )
2 2 2
2 b ab a b a + + = +
ab
a b
a
b
2
a ab
2
b
( )
2 2 2
2 b ab a b a + + = +
ab
a
8) ( ) ( ) ( )( ) ( )
2 4 8 2 4
2
4
2
4
49 140 100 7 7 10 2 10 7 10 + + = + + =

Representacin geomtrica de ( )
2
b a + :

Consiste en considerar el rea de un cuadrado de lados b a + y
las regiones que estas medidas generan en el cuadrado. Los
segmentos a y b horizontales y verticales dividen al cuadrado en
cuatro reas menores: dos cuadrados, uno de lado a y otro
menor de lado b , y dos rectngulos de largo a y ancho b . La
suma de las reas de estos cuadrados y rectngulos es igual al
rea total del cuadrado de lado b a + :



Representacin geomtrica de ( )
2
b a :

Consiste en considerar el rea de un cuadrado de lados a . Los
segmentos b a y b horizontales y verticales dividen al cuadrado
en cuatro reas menores: dos cuadrados, uno de lado b a y otro
menor de lado b , y dos rectngulos de largo b a y ancho b . La
suma de las reas de estos cuadrados y rectngulos es igual al
rea total del cuadrado de lado
2
a . Por lo tanto, el rea del
cuadrado de b a es igual al rea total menos el rea de los
rectngulos menos el rea del cuadrado menor, esto es:
( ) ( )
2 2 2 2 2 2 2 2
2 2 2 2 b ab a b b ab a b b b a a b a + = + = =



V.1.2 CUADRADO DE UN POLINOMIO

El producto de un trinomio por s mismo recibe el nombre de cuadrado de un trinomio.

El desarrollo del cuadrado del trinomio c b a + + se puede obtener de la siguiente forma:

( ) ( ) [ ] ( ) ( )
2 2 2 2 2 2 2
2 2 2 2 c bc ac b ab a c c b a b a c b a c b a + + + + + = + + + + = + + = + +

ordenando se tiene

( ) bc ac ab c b a c b a 2 2 2
2 2 2 2
+ + + + + = + +

Por su parte, el desarrollo del cuadrado del polinomio de cuatro trminos d c b a + + + se puede obtener
de la siguiente forma:

( ) ( ) ( ) [ ] ( ) ( )( ) ( )
2 2 2 2
2 d c d c b a b a d c b a d c b a + + + + + + = + + + = + + +

2 2 2 2
2 2 2 2 2 2 d cd c bd bc ad ac b ab a + + + + + + + + + =

ordenando se llega a:

( ) cd bd bc ad ac ab d c b a d c b a 2 2 2 2 2 2
2 2 2 2 2
+ + + + + + + + + = + + +
( )
2 2 2
2 b ab a b a + =
b
a
a
b a
b a
2
b
( )
2
b a
b ( )b b a
( )b b a
( )
2 2 2
2 b ab a b a + =
b
a
a
b a
b a
2
b
( )
2
b a
b ( )b b a
( )b b a
Pgina del Colegio de Matemticas de la ENP-UNAM Productos notables y factorizacin Autor: Dr. Jos Manuel Becerra Espinosa
3

En general, el cuadrado de un polinomio est dado por la suma de los cuadrados de cada uno de sus
trminos ms el doble producto algebraico de sus trminos, tomados de dos en dos.

Ejemplos.

1) ( ) ( ) ( ) ( )( ) ( )( ) ( )( ) c b c a b a c b a c b a 3 2 2 3 2 2 2 3 2 3 2
2 2 2 2
+ + + + + = + +
bc ac ab c b a 12 6 4 9 4
2 2 2
+ + + + + =
2) ( ) ( ) ( ) ( ) ( )( ) ( )( ) ( )( ) z y z x y x z y x z y x 6 8 2 6 5 2 8 5 2 6 8 5 6 8 5
2 2 2 2
+ + + + + =
yz xz xy z y x 96 60 80 36 64 25
2 2 2
+ + + =
3) +

+ g e f e g f e g f e
4
3
2
1
2
5
2
2
1
2
4
3
5
2
2
1
4
3
5
2
2
1
2 2 2 2

fg eg ef g f e g f
5
3
4
3
5
2
16
9
25
4
4
1
4
3
5
2
2
2 2 2
+ + + =


4) ( ) ( ) ( ) ( ) ( ) ( )( ) ( )( )+ + + + + + = + c a b a d c b a d c b a 9 4 2 7 4 2 5 9 7 4 5 9 7 4
2 2 2 2 2

( )( ) ( )( ) ( )( ) ( )( ) d c d b c b d a 5 9 2 5 7 2 9 7 2 5 4 2 + + +
cd bd bc ad ac ab d c b a 90 70 126 40 72 56 25 81 49 16
2 2 2 2
+ + + + + =
5) ( ) ( ) ( ) ( ) ( )( ) ( )( ) ( )( )+ + + + + + + = + + t p r p q p t r q p t r q p
3 2 3 3 2
2
2 2
2
3
2
2 3
2 2 10 2 2 6 2 2 10 6 2 10 6 2
( )( ) ( )( ) ( )( ) t r t q r q
2 2
10 2 6 2 10 6 2 + +
t r qt qr t p r p q p t r q p
2 2 3 2 3 3 2 4 2 6
20 12 120 4 40 24 100 36 4 + + + + + + =
6) ( ) + +

+
2
4 2
2 2
3 2
2
2
2
4 2 3 2 2
6
2
5
4
7
2
3
6
2
5
4
7
2
3
s q p n m k j h s q p n m k j h
( )+

s q p j h n j h m k j h
4 2 2 2 3 2 2
6
2
3
2
2
5
2
3
2
4
7
2
3
2
( ) ( ) s q p n s q p m k n m k
4 2 4 2 3 2 3 2
6
2
5
2 6
4
7
2
2
5
4
7
2


+ + + =
3 2 2 2 8 4 2 6 4 2 4
4
21
36
4
25
16
49
4
9
m jk h s q p n m k j h
s q np s q p m k n m k s q jp h jn h
4 2 4 2 3 2 3 2 4 2 2 2
30 21
4
35
18
2
15
+ +


V.1.3 PRODUCTO DE DOS BINOMIOS CONJUGADOS

Dos binomios son conjugados si difieren slo por el signo de uno de sus trminos.

Ejemplos.

1) ( ) b a 3 4 + y ( ) b a 3 4
2) ( ) j k 5 2 y ( ) j k 5 2 +

Al efectuar el producto de un binomio b a + por su conjugado b a , se tiene:

Pgina del Colegio de Matemticas de la ENP-UNAM Productos notables y factorizacin Autor: Dr. Jos Manuel Becerra Espinosa
4

( )( )
2 2 2 2
b a b ba ab a b a b a = + = +

esto significa que el producto de dos binomios conjugados es igual a la diferencia de los cuadrados de
sus trminos.

Esto es:

( )( )
2 2
b a b a b a = +

Ejemplos.

1) ( )( ) 9 3 3
2
= + k k k
2) ( )( )
2 2
4 9 2 3 2 3 y x y x y x = +
3) ( )( )
2 2
64 25 8 5 8 5 b a b a b a = +
4) ( )( )
6 4 3 2 3 2
49 16 7 4 7 4 z w z w z w = +
5)
2 2
25
9
4
1
5
3
2
1
5
3
2
1
y x y x y x =

+
6) ( )( )
2 2 2 2
16 36 4 6 4 6 n m k j mn jk mn jk = +
7) ( )( )
2 10 4 8 6 4 5 2 4 3 2 5 2 4 3 2
144 100 12 10 12 10 w u s v t r w u s v t r w u s v t r = +
8) ( )( )
2
1 1 1 = + +

La representacin del producto de dos binomios conjugados se efecta a partir de un cuadrado de lado
a y un cuadrado interior de lado b . El rea sombreada representa
2 2
b a y est dada por la suma de
los rectngulos ( )a b a y ( ) b a b , esto es, ( )( ) b a b a + :



b a
b
( )( )
2 2
b a b a b a = +
a
a
2
b
b a b



Pgina del Colegio de Matemticas de la ENP-UNAM Productos notables y factorizacin Autor: Dr. Jos Manuel Becerra Espinosa
5

V.1.4 PRODUCTO DE DOS BINOMIOS CON UN TRMINO COMN

Este producto notable corresponde a la multiplicacin de binomios cuyo trmino comn es x de la forma
( ) a x + por ( ) b x + . Al desarrollar el producto se tiene: ( )( ) ab xa xb x b x a x + + + = + +
2
, que se
puede agrupar como sigue:

( )( ) ( ) ab x b a x b x a x + + + = + +
2


Esto significa que el producto de binomios con un trmino comn es el cuadrado del trmino comn, ms
la suma de los trminos distintos multiplicada por el trmino comn y ms el producto de los trminos
distintos.

Ejemplos.

1) ( )( ) ( ) ( )( ) 6 5 3 2 3 2 3 2
2 2
+ + = + + + = + + x x x x x x
2) ( )( ) ( ) ( )( ) 4 3 4 1 4 1 4 1
2 2
+ = + + + = + a a a a a a
3) ( )( ) ( ) ( )( ) ( )( ) 15 16 4 3 5 2 3 5 2 3 2 5 2
2 2
+ + = + + + = + + b b b b b b
4) ( )( ) ( ) ( )( ) ( )( ) 42 39 9 7 6 3 7 6 3 7 3 6 3
2 2
+ = + + = z z z z z z
5) ( ) ( )( ) 5
2
21
16
49
1 5
4
7
1 5
4
7
1
4
7
5
4
7
2
2
+ = +

x x x x x x
6) ( )( ) ( ) ( )( ) ( )( ) 88 6 4 11 8 2 11 8 2 11 2 8 2
4 8 4
2
4 4 4
= + + = + e e e e e e
7) ( )( ) ( ) ( )( ) ( )( ) 28 15 25 7 4 5 7 4 5 7 5 4 5
2 3 4 6 2 3
2
2 3 2 3 2 3
+ = + + + = +
8) ( )( ) ( ) ( )( ) ( )( ) 60 17 12 5 12 5 12 5
2 2
+ = + + + = + + k k k k k k

Para representar el producto de dos binomios con un trmino comn se utiliza un cuadrado de lado x . A
uno de los lados se le agrega una cantidad a y a otro se le agrega una cantidad b , por lo que se forma
una superficie con cuatro regiones:



b x +
b
a
xb
xa ab
( )( ) ( ) ab x b a x b x a x + + + = + +
2
a x +
2
x
x
x

Pgina del Colegio de Matemticas de la ENP-UNAM Productos notables y factorizacin Autor: Dr. Jos Manuel Becerra Espinosa
6

El rea total que es ( )( ) b x a x + + , tambin est dada por la suma de cada una de las reas, es decir
ab xa xb x + + +
2
, que en forma simplificada es: ( ) ab x b a x + + +
2
.


V.1.5 CUBO DE UN BINOMIO

El desarrollo del cubo del binomio b a + se puede obtener multiplicando este binomio por su cuadrado:

( ) ( )( ) ( )( )
2 2 2 3
2 b ab a b a b a b a b a + + + = + + = +

3 2 2 2 2 3
2 2 b ab ba ab b a a + + + + + =

que simplificado es:

( )
3 2 2 3 3
3 3 b ab b a a b a + + + = +

Por su parte, el desarrollo del cubo del binomio b a , se obtiene de forma similar:

( ) ( )( ) ( )( )
2 2 2 3
2 b ab a b a b a b a b a + = =

3 2 2 2 2 3
2 2 b ab ba ab b a a + + =

que simplificado es:

( )
3 2 2 3 3
3 3 b ab b a a b a + =

En las frmulas anteriores a y b pueden ser cualquier expresin algebraica y tener cualquier signo. Por
lo tanto, segunda la frmula es un caso particular de la primera ya que:
( ) ( ) [ ] ( ) ( )
3 2 2 3 3 2 3 3 3
3 3 3 3 b ab b a a b b a b a a b a b a + + = + + + = + =

Considerando lo anterior, se aprecia que el desarrollo anterior presenta la siguiente estructura:

El cubo de la suma de dos trminos es igual al cubo del primer trmino ms el triple del cuadrado del
primer trmino por el segundo ms el triple del primer trmino por el cuadrado del segundo ms el cubo
del segundo trmino.

Ejemplos.

1) ( ) ( )( ) ( )( ) ( )( ) ( )( ) 8 12 6 8 4 3 2 3 2 2 3 2 3 2
2 3 2 3 3 2 2 3 3
+ + + = + + + = + + + = + a a a a a a a a a a
2) ( ) ( )( ) ( )( ) ( ) ( )( ) ( )( ) 125 25 3 5 3 5 5 3 5 3 5
2 3 3 2 2 3 3
+ + = + + + = k k k k k k k
125 75 15
2 3
+ = k k k
3) ( ) ( ) ( ) ( ) ( )( ) ( )( ) ( )( )
3 2 2 3 3 2 2 3 3
4 3 16 3 64 4 3 4 3 4 4 y y x y x x y y x y x x y x + + + = + + + = +

3 2 2 3
12 48 64 y xy y x x + + + =
4) ( ) ( ) ( ) ( ) ( )( ) ( )
3 2 2 3 3
7 7 6 3 7 6 3 6 7 6 d d c d c c d c + + + =
( )( ) ( )( )
3 2 2 3 3 2 2 3
343 882 756 216 343 49 6 3 7 36 3 216 d cd d c c d d c d c c + = + + =
5)
3 2 2 3 3
5
2
5
2
3
1
3
5
2
3
1
3
3
1
5
2
3
1

+ b b a b a a b a
Pgina del Colegio de Matemticas de la ENP-UNAM Productos notables y factorizacin Autor: Dr. Jos Manuel Becerra Espinosa
7

+ =
2 2 3
25
4
3
1
3
5
2
9
1
3
27
1
b a b a a

3 2 2 3 3 2 2 3 3
125
8
25
4
15
2
27
1
125
8
25
4
15
2
27
1
125
8
b ab b a a b ab b a a b + + + = + + + = +
6) ( ) ( ) ( ) ( ) ( )( ) ( )
3
2
2
2 3 2
2
3
3
3
3
2 3
8 8 4 3 8 4 3 4 8 4 y y x y x x y x + + + =
( )( ) ( )( )
6 4 3 2 6 9 6 4 3 2 6 9
512 768 384 64 512 64 4 3 8 16 3 64 y y x y x x y y x y x x + = + + =
7) ( ) ( ) ( ) ( ) ( )( ) ( )
3 2 2 3 3
10 10 3 3 10 3 3 3 10 3 + + + = + a a a a
( )( ) ( )( ) 1000 900 270 27 1000 100 3 3 10 9 3 27
2 3 2 3
+ + = + + + = a a a a a a
8) ( ) ( ) ( ) ( ) ( )( ) ( )
3 2 2 3 3
2 2 9 3 2 9 3 9 2 9 + + + = z z z z
( )( ) ( )( ) 8 108 486 729 8 4 9 3 2 81 3 729
2 3 2 3
= + + = z z z z z z


V.1.6 CUBO DE UN TRINOMIO

El desarrollo de un cubo de trinomio c b a + + se obtiene multiplicando este trinomio por su cuadrado:

( ) ( )( ) ( )( ) bc ac ab c b a c b a c b a c b a c b a 2 2 2
2 2 2 2 3
+ + + + + + + = + + + + = + +
c b abc ab bc b b a abc c a b a ac ab a
2 2 2 3 2 2 2 2 2 3
2 2 2 2 2 2 + + + + + + + + + + + =

2 2 3 2 2
2 2 2 bc ac abc c c b c a + + + + + +

simplificado queda como:

( ) abc bc c b ac c a ab b a c b a c b a 6 3 3 3 3 3 3
2 2 2 2 2 2 3 3 3 3
+ + + + + + + + + = + +

El resultado consta de diez trminos y presenta la siguiente estructura:

El cubo de un trinomio es igual a la suma de los cubos de cada uno de los trminos, ms el triple
producto del cuadrado de cada trmino por cada uno de los trminos restantes ms seis veces el
producto de los tres trminos.

Ejemplos.

1) ( ) ( ) ( ) ( ) ( ) ( ) ( )( ) ( ) ( ) c a b a b a c b a c b a 5 4 3 2 4 3 2 4 3 5 2 4 5 2 4
2 2 2 3 3 3 3
+ + + + + = + +
( )( ) ( ) ( ) ( )( ) ( )( )( ) c b a c b c b c a 5 2 4 6 5 2 3 5 2 3 5 4 3
2 2 2
+ + + +
( )( ) ( )( ) ( )( ) c a b a b a c b a 5 16 3 4 4 3 2 16 3 125 8 64
2 2 2 3 3 3
+ + + + + =
( )( ) ( )( ) ( )( ) ( )( )( ) c b a c b c b c a 5 2 4 6 25 2 3 5 4 3 25 4 3
2 2 2
+ + + +

2 2 2 2 3 3 3
300 240 48 96 125 8 64 ac c a ab b a c b a + + + + + + =
abc bc c b 240 150 60
2 2
+ + +

2) ( ) ( ) ( ) ( ) ( ) ( ) ( )( ) ( ) ( ) 1 3 3 6 3 3 6 3 3 1 6 3 1 6 3
2 2 2 3 3 3 3
+ + + + + = x y x y x y x y x
( )( ) ( ) ( ) ( )( ) ( )( )( ) 1 6 3 6 1 6 3 1 6 3 1 3 3
2 2 2
+ + + + y x y y x
( )( ) ( )( ) ( )( ) 1 9 3 36 3 3 6 9 3 1 216 27
2 2 2 3 3
+ + + = x y x y x y x
( )( ) ( )( ) ( )( ) ( )( )( ) 1 6 3 6 1 6 3 1 36 3 1 3 3
2
+ + + + y x y y x
Pgina del Colegio de Matemticas de la ENP-UNAM Productos notables y factorizacin Autor: Dr. Jos Manuel Becerra Espinosa
8

xy y y x x xy y x y x 108 18 108 9 27 324 162 1 216 27
2 2 2 2 3 3
+ + + =

3)
2 2 3 3 3 3
3
2
2
1
3
3
2
2
1
3
4
3
3
2
2
1
4
3
3
2
2
1

+ f e f e g f e g f e

2 2 2 2
4
3
3
2
3
4
3
3
2
3
4
3
2
1
3
4
3
2
1
3

+ g f g f g e g e

+ g f e
4
3
3
2
2
1
6

+ + =
2 2 3 3 3
9
4
2
1
3
3
2
4
1
3
64
27
27
8
8
1
f e f e g f e

+
2 2 2 2
16
9
3
2
3
4
3
9
4
3
16
9
2
1
3
4
3
4
1
3 g f g f g e g e

+ g f e
4
3
3
2
2
1
6

2 2 2 2 3 3 3
32
27
16
9
3
2
2
1
64
27
27
8
8
1
eg g e ef f e g f e + + + + =
efg fg g f
2
3
8
9
2 2
+

4) ( ) ( ) ( ) ( ) ( ) ( )
3
2
2
3
4
3
3
3
2
3
4 3 2
4 5 3 10 4 5 10 4 5 + + + = +
( )( ) ( ) ( ) ( )( )
2
4 2 4
2
2
2
3 2
10 5 3 10 5 3 4 5 3 + + +
( ) ( ) ( )( ) ( )( )( )
4 3 2
2
4 3 4
2
3
10 4 5 6 10 4 3 10 4 3 + + +
( )( ) ( )( )
6 2 3 4 12 9 6
16 5 3 4 25 3 1000 64 125 + + + =
( )( ) ( )( ) ( )( ) ( )( )
8 3 4 6 8 2 4 4
100 4 3 10 16 3 100 5 3 10 25 3 + + + +
( )( )( )
4 3 2
10 4 5 6 +

4 4 6 2 3 4 12 9 6
750 240 300 1000 64 125 + + =

4 3 2 8 3 4 6 8 2
200 1 200 1 480 500 1 , , , + +


V.1.7 SUMA Y RESTA DE CUBOS

Para obtener la suma de dos cubos de la forma
3 3
b a + se efecta el siguiente producto:

( )( )
2 2
b ab a b a + +

cuyo desarrollo es:
3 2 2 2 2 3
b ab b a ab b a a + + +

y simplificando se tiene:
3 3
b a +

Esto significa que:

Pgina del Colegio de Matemticas de la ENP-UNAM Productos notables y factorizacin Autor: Dr. Jos Manuel Becerra Espinosa
9

La suma de los cubos de dos trminos es igual al producto de la suma de los trminos, por un trinomio
formado por el cuadrado del primer trmino, menos el producto de los dos, ms el cuadrado del segundo.

Es decir:

( )( )
3 3 2 2
b a b ab a b a + = + +

Ejemplos.
Comprobar que los productos indicados representan la suma de dos cubos.

1) ( )( ) 1 1
2
+ + x x x
Solucin.
( )( )
3 3 3 2 2 3 2
1 1 1 1 1 + = + = + + + = + + x x x x x x x x x x
2) ( )( )
2 2
9 6 4 3 2 b ab a b a + +
Solucin:
( )( )
3 2 2 2 2 3 2 2
27 18 12 18 12 8 9 6 4 3 2 b ab b a ab b a a b ab a b a + + + = + +
( ) ( )
3 3 3 3
3 2 27 8 b a b a + = + =
3) ( )( )
12 6 2 4 6 2
25 20 16 5 4 j j k k j k + +
Solucin:
( )( )
18 12 2 4 6 12 2 6 4 6 12 6 2 4 6 2
125 100 80 100 80 64 25 20 16 5 4 j j k k j j k j k k j j k k j k + + + = + +
( ) ( )
3
6
3
2 18 6
5 4 125 64 j k j k + = + =

Similarmente, para obtener la diferencia de dos cubos de la forma
3 3
b a se efecta el siguiente
producto:

( )( )
2 2
b ab a b a + +

cuyo desarrollo es:
3 2 2 2 2 3
b ab b a ab b a a + +

y simplificando se tiene:
3 3
b a

Esto significa que:

La diferencia de los cubos de dos trminos es igual al producto de la diferencia de los trminos, por un
trinomio formado por el cuadrado del primer trmino, ms el producto de los dos, ms el cuadrado del
segundo.

Es decir:

( )( )
3 3 2 2
b a b ab a b a = + +

Ejemplos.
Comprobar que los productos indicados representan la diferencia de dos cubos

1) ( )( ) 4 2 2
2
+ + y y y
Solucin.
( )( )
3 3 3 2 2 3 2
2 8 8 4 2 4 2 4 2 2 = = + + = + + y y y y y y y y y y
Pgina del Colegio de Matemticas de la ENP-UNAM Productos notables y factorizacin Autor: Dr. Jos Manuel Becerra Espinosa
10

2) ( )( )
2 2
36 30 25 6 5 q pq p q p +
Solucin:
( )( )
3 2 2 2 2 3 2 2
216 180 150 180 150 125 36 30 25 6 5 q pq qp pq q p p q pq p q p + = +
( ) ( )
3 3 3 3
6 5 216 125 q p q p = =
3)

+ +


2 2
25
9
10
3
4
1
5
3
2
1
b ab a b a
Solucin:
3 2 2 2 2 3 2 2
125
27
50
9
20
3
50
9
20
3
8
1
25
9
10
3
4
1
5
3
2
1
b ab b a ab b a a b ab a b a + + =

+ +



3 3
3 3
5
3
2
1
125
27
8
1

= = b a b a


V.1.8 BINOMIO DE NEWTON

El teorema del binomio, tambin llamado binomio de Newton, expresa la ensima potencia de un
binomio como un polinomio. El desarrollo del binomio ( )
n
b a + posee singular importancia ya que
aparece con mucha frecuencia en Matemticas y posee diversas aplicaciones en otras reas del
conocimiento.

Si el binomio de la forma ( ) b a + se multiplica sucesivamente por si mismo se obtienen las siguientes
potencias:


( ) b a b a + = +
1

( ) ( )( )
2 2
2
2
2 b ab a b a b a b a
veces
+ + = + + = +
4 43 4 42 1

( ) ( )( )( )
3 2 2 3
3
3
3 3 b ab b a a b a b a b a b a
veces
+ + + = + + + = +
4 4 4 3 4 4 4 2 1

( ) ( ) ( )
4 3 2 2 3 4
4
4
4 6 4 b ab b a b a a b a b a b a
veces
+ + + + = + + = +
4 4 3 4 4 2 1

( ) ( ) ( )
5 4 3 2 2 3 4 5
5
5
5 10 10 5 b ab b a b a b a a b a b a b a
veces
+ + + + + = + + = +
4 4 3 4 4 2 1

( ) ( ) ( )
6 5 4 2 3 3 2 4 5 6
6
6
6 15 20 15 6 b ab b a b a b a b a a b a b a b a
veces
+ + + + + + = + + = +
4 4 3 4 4 2 1



De los desarrollos anteriores, se observa que:

El desarrollo de
n
b a ) ( + tiene 1 + n trminos
El exponente de a empieza con n en el primer trmino y va disminuyendo en uno con cada trmino,
hasta cero en el ltimo
El exponente de b empieza con cero en el primer trmino y va aumentando en uno con cada
trmino, hasta n en el ltimo
Pgina del Colegio de Matemticas de la ENP-UNAM Productos notables y factorizacin Autor: Dr. Jos Manuel Becerra Espinosa
11

Para cada trmino la suma de los exponentes de a y b es n
El coeficiente del primer trmino es uno y el del segundo es n
El coeficiente de un trmino cualquiera es igual al producto del coeficiente del trmino anterior por el
exponente de a dividido entre el nmero que indica el orden de ese trmino
Los trminos que equidistan de los extremos tienen coeficientes iguales.

Ejemplo.
( )
6 5 4 2 3 3 2 4 5 6 6
6 15 20 15 6 b ab b a b a b a b a a b a + + + + + + = +

( ) ( ) ( ) ( ) ( ) ( )
6
1 6
5
2 15
4
3 20
3
4 15
2
5 6
1
6 1 1

es


Aplicando las consideraciones expuestas en los incisos para el caso general se tiene:

( )
( ) ( )( )
( )( )
( )( )( )
4 4 3 3 2 2 1
4 3 2 1
3 2 ) 1 (
3 2 1
) 2 )( 1 (
2 1
) 1 (
1
b a
n n n n
b a
n n n
b a
n n
b a
n
a b a
n n n n n n

+

+

+ + = +

( )( )( )( )
( )( )( )( )
n n
b b a
n n n n n
+ +

+
5 5
5 4 3 2 1
4 3 2 1


Se define como factorial de un nmero natural n al producto de n por todos los nmeros que le
preceden hasta el uno. Se denota mediante ! n :

( )( )( ) ( )( ) n n n 1 4 3 2 1 ! =

Por definicin, el factorial de cero es uno: 1 ! 0

Ejemplos.
( )( ) 6 3 2 1 ! 3 = =
( )( )( )( ) 120 5 4 3 2 1 ! 5 = =
( )( )( )( )( )( )( ) 320 , 40 8 7 6 5 4 3 2 1 ! 8 = =
( )( )( ) ( )( ) 200 , 291 ' 178 , 87 14 13 4 3 2 1 ! 14 = =

Ahora, si se introduce la notacin factorial, la frmula del binomio puede escribirse as:

( )
( )( )
4 4 3 3 2 2 1
! 4
3 2 ) 1 (
! 3
) 2 )( 1 (
! 2
) 1 (
! 1
b a
n n n n
b a
n n n
b a
n n
b a
n
a b a
n n n n n n

+

+

+ + = +

( )( )( )( )
n n
b b a
n n n n n
+ +

+
5 5
! 5
4 3 2 1


Ejemplos.
1) Obtener el desarrollo de
4
) 4 3 ( y x
Solucin.
Haciendo x a 3 = , y b 4 = y 4 = n
Aplicando la frmula se tiene:
( ) ( ) ( ) ( )
( )
( ) ( )
( )( )
( )( ) ( )
4 3 2 2 3 4 4
4 4 3
! 3
2 3 4
4 3
! 2
3 4
4 3
! 1
4
3 4 3 y y x y x y x x y x + + + + =
( ) ( ) ( ) ( ) ( ) ( ) ( )( ) ( )
4 3 2 2 3 4 4
4 4 3
6
24
4 3
2
12
4 3
1
4
3 4 3 y y x y x y x x y x + + + + =
Pgina del Colegio de Matemticas de la ENP-UNAM Productos notables y factorizacin Autor: Dr. Jos Manuel Becerra Espinosa
12

( )( ) ( )( ) ( )( )
4 3 2 2 3 4
256 64 3
6
24
16 9
2
12
4 27
1
4
81 y y x y x y x x + + + + =

4 3 2 2 3 4
256 768 864 432 81 y xy y x y x x + + =

2) Hallar la expansin de
5
) 2 5 ( y x +
Solucin.
Haciendo 5x a = , y b 2 = y 5 = n
Aplicando la frmula se tiene:
( ) ( ) ( ) ( )
( )
( ) ( )
( )( )
( ) ( )
( )( )( )
( )( )
4 3 2 2 3 4 5 5
2 5
! 4
2 3 4 5
2 5
! 3
3 4 5
2 5
! 2
4 5
2 5
! 1
5
5 2 5 y x y x y x y x x y x + + + + = +
( )
5
2y +
( )( ) ( )( ) ( )( ) ( )( )
5 4 3 2 2 3 4 5
32 16 5 5 8 25 10 4 125 10 2 625 5 125 , 3 y y x y x y x y x x + + + + + =

5 4 3 2 2 3 4 5
32 400 000 , 2 000 , 5 250 , 6 125 , 3 y xy y x y x y x x + + + + + =

En el desarrollo binomial:

( )
n n n n n n n n
b ab
n
b a
) n ( n
b a
) n )( n ( n
b a
) n ( n
b a
n
a b a + +

+ +

+

+ + = +
1 2 2 3 3 2 2 1
! 1 ! 2
1
! 3
2 1
! 2
1
! 1


Analizando las caractersticas del desarrollo y si se decide llamar a un trmino cualquiera del desarrollo
como r-simo trmino, se observa que:

El exponente de b es: 1 r
El exponente de a es: ( ) 1 1 + = r n r n
El denominador del coeficiente es: ( )! 1 r
El numerador del coeficiente es: ( )( ) ( ) 2 2 1 + r n n n n

En consecuencia el r-simo trmino de la expansin de ( )
n
b a + es:

( )( ) ( )
( )
1 1
! 1
2 2 1
+

+
r
b
r n
a
r
r n n n n


Ejemplos.

1) Encontrar el tercer trmino del desarrollo
5
) 6 2 ( m k
Solucin.
3 5 6 2 = = = = r , n , m b , k a
Aplicando la expresin se tiene:
( )( )
( ) ( ) ( )( )
2 3 2 3 2 3
880 , 2 36 8 10 6 2
! 2
4 5
m k m k m k = =

2) Calcular el sexto trmino del desarrollo
7
) 4 ( y x +
Solucin.
6 7 4 = = = = r , n , y b , x a
Aplicando la expresin se tiene:
( )( )( )( )
( ) ( )
5 2 5 2 5 2
504 21 1024 21 4
! 5
3 4 5 6 7
y x , y x y x = =
Pgina del Colegio de Matemticas de la ENP-UNAM Productos notables y factorizacin Autor: Dr. Jos Manuel Becerra Espinosa
13

El tringulo de Pascal es un esquema que tiene como caracterstica que cada uno de los componentes
de sus filas representa los coeficientes del desarrollo binomial.

Se construye de la siguiente manera: Se empieza por el 1 de la cumbre. De una fila a la siguiente se
escriben los nmeros con un desfase de medio lugar o casilla para que cada casilla tenga dos nmeros
justo arriba, en la fila anterior. Cada extremo de la fila tiene un 1 y el valor que se escribe en una casilla
es la suma de los nmeros que estn encima.

Despus, se efecta una relacin entre los nmeros del tringulo de Pascal y la suma de las potencias de
a y b , de forma que los coeficientes se asignan en el mismo orden en que aparecen. Grficamente esto
es:


1 1
1 1 2
1 1
1 1
3 3
4 4 6
1 1 5 5 10 10
1 1 6 6 15 15 20
1 1 7 7 21 21 35 35
( )
1
b a +
( )
2
b a +
( )
3
b a +
( )
4
b a +
( )
5
b a +
( )
6
b a +
( )
7
b a +
1 ( )
0
b a +


Por ejemplo, para encontrar los coeficientes del desarrollo ( )
6
b a + , se le aplican los factores de la
sptima fila, tal y como se muestra en la siguiente figura:

1 6 15 20 15 6 1
6 5 4 2 3 3 2 4 5 6
6 15 20 15 6 b ab b a b a b a b a a + + + + + +
6 5 4 2 3 3 2 4 5 6
b ab b a b a b a b a a

Pgina del Colegio de Matemticas de la ENP-UNAM Productos notables y factorizacin Autor: Dr. Jos Manuel Becerra Espinosa
14

Ejemplos.

1) Aplicar el tringulo de Pascal para desarrollar ( )
4
2 3 y x
Solucin.
Ubicando los coeficientes respectivos se tiene:
( ) ( ) ( ) ( ) ( ) ( ) ( )( ) ( )
4 3 2 2 3 4 4
2 2 3 4 2 3 6 2 3 4 3 2 3 y y x y x y x x y x + + + + =
( )( ) ( )( ) ( )( )
4 3 2 2 3 4
16 8 3 4 4 9 6 2 27 4 81 y y x y x y x x + + + + =

4 3 2 2 3 4
16 96 216 216 81 y xy y x y x x + + =

2) Encontrar la expansin de ( )
6
3 4
4 5 b a + aplicando el tringulo de Pascal.
Solucin.
Ubicando los coeficientes respectivos se tiene:
( ) ( ) ( ) ( ) ( ) ( ) ( ) ( ) ( ) ( )
4
3
2
4
3
3
3
4
2
3
4
4 3
5
4
6
4
6
3 4
4 5 15 4 5 20 4 5 15 4 5 6 5 4 5 b a b a b a b a a b a + + + + = +
( )( ) ( )
6
3
5
3 4
4 4 5 6 b b a + +
( )( ) ( )( ) ( )( )
9 12 6 16 3 20 24
64 125 20 16 625 15 4 125 , 3 6 625 , 15 b a b a b a a + + + =
( )( ) ( )( )
18 15 4 12 8
096 , 4 024 , 1 5 6 256 25 15 b b a b a + + +

12 8 9 12 6 16 3 20 24
000 , 96 000 , 160 000 , 150 000 , 75 625 , 15 b a b a b a b a a + + + + =



V.2 FACTORIZACIN

Un factor es cada uno de los nmeros que se multiplican para formar un producto.

Ejemplo.
Sean los siguientes productos:

( )( ) 6 2 3 = , por lo que factores de son 3 y .
( )( ) 10 2 5 = , por lo que factores de son 5 y 2 .
( )( )( ) 30 2 3 5 = , por lo que factores de 30 son 5, 3 y 2 .

Ntese como el nmero 2 aparece como factor comn de 6 , 10 y 30 porque cada uno de estos
nmeros se divide exactamente entre dicho factor comn.

Cuando una expresin algebraica est contenida exactamente en todos y cada uno de los trminos de un
polinomio, se dice que es factor comn de ellos.

Ejemplos.

1) El trmino
2
3x es factor comn de y x
4
6 , de
3
9x y de
2 2
12 y x porque cada monomio puede
expresarse como el producto de
2
3x por otro trmino, es decir:
( )( ) y x x y x
2 2 4
2 3 6 =
( )( ) x x x 3 3 9
2 3
=
( )( )
2 2 2 2
4 3 12 y x y x =
18 15 4
096 , 4 720 , 30 b b a + +
6 2
10
Pgina del Colegio de Matemticas de la ENP-UNAM Productos notables y factorizacin Autor: Dr. Jos Manuel Becerra Espinosa
15

2) El trmino
2
4ab es factor comn de
3 2
28 b a , de
2 3
20 b a y de
3
8ab porque cada monomio puede
expresarse como el producto de
2
4ab por otro trmino, es decir:
( )( ) ab ab b a 7 4 28
2 3 2
=
( )( )
2 2 2 3
5 4 20 a ab b a =
( )( ) b ab ab 2 4 8
2 3
=

Factorizar es el proceso que permite descomponer en factores una expresin matemtica. Esto significa
que factorizar es convertir una expresin en el producto indicado de sus factores.

En toda expresin debe obtenerse la mxima factorizacin posible. Los tipos de factorizacin ms
utilizados se exponen a continuacin.


V.2.1 MONOMIO COMO FACTOR COMN

Para encontrar el factor comn de los trminos de un polinomio se busca el mximo comn divisor (MCD)
de los coeficientes de todos los trminos, y de las literales que aparezcan en todos los trminos, se
escogen las que tengan el menor exponente.

Ejemplos.
Factorizar los siguientes polinomios.

1)
El MCD de los coeficientes es , y las literales de menor exponente que aparecen en todos los trminos
son:
2
a y b , por lo que el factor comn es: b a
2
2
As que: ( ) b a b a b a b a 5 2 2 10 4
2 2 2 3
+ = +
2)
4 2 4 5 4 3 3 5
21 18 6 z y x z y x z y x +
El MCD de los coeficientes es 3, y las literales de menor exponente que aparecen en todos los trminos
son:
3
x ,
2
y y z , por lo que el factor comn es: z y x
2 3
3
As que: ( )
3 4 2 2 2 3 4 2 4 5 4 3 3 5
7 6 2 3 21 18 6 xz z y y x z y x z y x z y x z y x + = +
3)
4) ( ) q p p pq p + = + 4 3 3 12
2

5) ( ) x x x x x x x x x x 4 5 3 7 2 8 32 40 24 56 16
3 2 4 2 3 5 2 4 6
+ + = + +
6) ( )
6 2 2 5 3 3 2 3 2 9 2 5 4 8 3 6 5 3 4
13 2 9 10 7 7 91 14 63 70 49 m m k km m k k m k m k m k m k m k m k + + = + +
7)

( )
2 3 2 3 4 4 2
5
2
3
2
15
2
3
e f f e f e f e + = +
8) ( )
2 2 2 2 3 6 4 2 6 4 4 2 2 5 4 7 6 3
5 6 4 2 11 55 66 44 22 + = +
Ntese como no aparece en el factor comn la literal ya que no est en todos los trminos del
polinomio.


V.2.2 POLINOMIO COMO FACTOR COMN

En una expresin, cuando el mximo comn divisor (MCD) de todos los trminos es un polinomio
entonces se puede descomponer como el producto de este factor comn por un polinomio cuyo resultado
sea la expresin original, tal y como se muestra a continuacin.
2 2 3
10 4 b a b a +
2
( ) m k k km k + = +
2
Pgina del Colegio de Matemticas de la ENP-UNAM Productos notables y factorizacin Autor: Dr. Jos Manuel Becerra Espinosa
16

Ejemplos.
Factorizar las siguientes expresiones.

1) ( ) ( ) b a k b a + + + 5
El MCD de los todos los trminos es: ( ) b a +
As que: ( ) ( ) ( )( ) k b a b a k b a + + = + + + 5 5
2) ( ) ( ) ( ) n m s n m q n m r 3 11 3 8 3 6 +
El MCD de los todos los trminos es: ( ) n m 3
As que: ( ) ( ) ( ) ( )( ) s q r n m n m s n m q n m r 11 8 6 3 3 11 3 8 3 6 + = +
3) ( ) ( ) z y x p z y x z y x w 2 3 4 2 3 2 3 + + + +
Esta expresin puede rescribirse como:
( ) ( ) ( ) z y x p z y x z y x w 2 3 4 2 3 1 2 3 + + + +
por lo que el MCD de los todos los trminos es: ( ) z y x 2 3 +
As que: ( ) ( ) ( )( ) p w z y x z y x p z y x z y x w 4 1 2 3 2 3 4 2 3 2 3 + + = + + + +
4) ( ) ( )
3 2 2
4 3 3 4 a a a a +
Esta expresin puede rescribirse como:
( ) ( )
3 2 2
3 4 3 4 a a a a
El MCD de los todos los trminos es: ( )
2
3 4 a a
As que: ( ) ( ) ( ) ( ) [ ] ( ) ( ) ( ) ( ) a a a a a a a a a a a a a a = = = 1 3 4 3 3 3 3 4 3 4 3 4 3 4 3 4
2 2 2 3 2 2

5) ( ) ( )( ) 1 9 7 4 7 4 7 4 9
2 2
+ + = + + + z f e f e f e z
6) ( ) ( ) ( ) ( )( ) ( )( ) 1 5 2 2 2 10 2 2 2 2 10 2 4 2 10
3 3 3 3 3 3
+ = + = + = + u d c u d c d c d c u d c d c u
7) ( )( ) ( ) ( )( ) ( )( ) y c b x w w y c w w b x + + = + + + + 8 3 3 3 8 3
8) ( ) ( ) ( )( ) b c a a a a
b
a a
c
+ + = + + +
2 2 2 2
2
2 1
3
4
1
3
4
1
3
8



V.2.3 FACTORIZACIN POR AGRUPACIN DE TRMINOS

Existen polinomios cuyos trminos no contienen un mismo factor comn. En esos casos, se debe
factorizar por agrupacin, procedimiento que combina los dos mtodos anteriores.

Ejemplos.
Factorizar los siguientes polinomios:

1) bw aw bx ax + + +
Para los primeros dos trminos se toma como factor comn a x y para los otros dos a w:
( ) ( ) b a w b a x + + +
ahora, se factoriza el polinomio ( ) b a + :
( )( ) w x b a + +
( )( ) w x b a bw aw bx ax + + = + + +
2) y x ay ax 4 4 + + +
El factor comn para los primeros dos trminos es a y para los otros dos es 4 :
( ) ( ) y x y x a + + + 4
despus, se factoriza el polinomio ( ) y x + :
Pgina del Colegio de Matemticas de la ENP-UNAM Productos notables y factorizacin Autor: Dr. Jos Manuel Becerra Espinosa
17

( )( ) 4 + + a y x
( )( ) 4 4 4 + + = + + + a y x y x ay ax
3)
2
9 6 15 10 y xy py px +
Para los primeros dos trminos se toma como factor comn a p 5 y para los otros dos a y 3 :
( ) ( ) y x y y x p 3 2 3 3 2 5 +
ahora, se factoriza el polinomio ( ) y x 3 2 :
( )( ) y p y x 3 5 3 2 +
( )( ) y p y x y xy py px 3 5 3 2 9 6 15 10
2
+ = +
4) bd bc ad ac 3 6 4 8 +
El factor comn para los primeros dos trminos es a 4 y para los otros dos es b 3 :
( ) ( ) d c b d c a 2 3 2 4
despus, se factoriza el polinomio ( ) d c 2 :
( )( ) b a d c 3 4 2
( )( ) b a d c bd bc ad ac 3 4 2 3 6 4 8 = +
5) 3 10 3
2
+ + a a
Esta expresin puede rescribirse como: 3 9 3
2
+ + + a a a
El factor comn para los primeros dos trminos es a 3 :
( ) 3 3 3 + + + a a a
( )( ) 1 3 3 3 10 3
2
+ + = + + a a a a
6) ( ) ( ) ( )( ) ( )( ) z xy x x xz y x x x xz x y x z x xz y x y x 3 3 5 3 3 5 5 3 3 3 5 15 9 3 5
2 2 2 2 2 2 3 2 4
+ = + = + + = +

7) ( ) ( ) 2 3 2 3 2 3 2 3 3 2 2 3
2 2 2 2 2 2 2 2 2 2
+ = + = + ab y ab x y aby x abx aby x y abx
( )( )
2 2
2 3 y x ab + =
8) ( ) ( ) ( ) 1 2 1 2 1 2 1 2 2 2 1 2 2 2 + = + + = + + a a c a b a c ac b ab c ac b a ab
( )( ) 1 1 2 + = c b a
Otra forma de resolver este ejercicio es escribirlo como 1 2 2 2 + + c b c ac ab :
( ) ( ) ( ) ( )( ) 1 2 1 1 1 2 1 2 2 2 + = + + = + + a c b c b c b a c b a ac ab


V.2.4 FACTORIZACIN DE UN TRINOMIO CUADRADO PERFECTO

Una cantidad es cuadrado perfecto cuando es el producto de dos factores iguales, es decir, es el
cuadrado de otra cantidad. Por ejemplo,
2
9a es cuadrado perfecto, ya que es el cuadrado de a 3 .

Se conoce como trinomio cuadrado perfecto (TCP) al resultado que se obtiene de elevar al cuadrado un binomio:


( )
4 43 4 42 1
3 2 1
Perfecto
Cuadrado Trinomio
binomio
un de
Cuadrado
b ab a b a
2 2 2
2 + + = +



Para identificar si un trinomio es cuadrado perfecto, se debe cumplir que dos de sus trminos sean
cuadrados perfectos y que el otro trmino corresponda al doble producto de las races cuadradas de los
trminos cuadrticos.
Pgina del Colegio de Matemticas de la ENP-UNAM Productos notables y factorizacin Autor: Dr. Jos Manuel Becerra Espinosa
18

Ejemplos.
Determinar si los siguientes trinomios son cuadrados perfectos.

1)
2 2
25 40 16 y xy x + +
Primero se comprueba que dos trminos sean cuadrados perfectos:
x x 4 16
2
=
y y 5 25
2
=
el doble producto de las races cuadradas debe ser igual al otro trmino:
( )( ) xy y x 40 5 4 2 =
por lo tanto el trinomio, es un TCP.
2)
4 2 2
64 96 36 b ab a + +
Comprueba que dos trminos sean cuadrados perfectos:
a a 6 36
2
=
2 4
8 64 b b =
el doble producto de las races cuadradas debe ser igual al otro trmino:
( )( )
2 2
96 8 6 2 ab b a =
por lo tanto, el trinomio es un TCP.
3)
2 2
9 10 4 m km k + +
Primero se comprueba que dos trminos sean cuadrados perfectos:
k k 2 4
2
=
m m 3 9
2
=
el doble producto de las races cuadradas debe ser igual al otro trmino:
( )( ) km km m k 10 12 3 2 2 =
por lo tanto el trinomio no es un TCP.

Para factorizar un trinomio cuadrado perfecto se extrae la raz cuadrada de los trminos que son
cuadrados perfectos, se separan por el signo que tiene el trmino que no lo es y finalmente se eleva el
binomio al cuadrado.

Ejemplos.
Factorizar los siguientes TCP:

1) 49 14
2
+ + x x
Se extraen las races de los trminos cuadrados perfectos:
x x =
2

7 49 =
se separan por el signo del otro trmino ( ) + y el binomio se eleva al cuadrado: ( )
2
7 + x
( )
2 2
7 49 14 + = + + x x x
2)
2 2
4 4 b ab a +
Extrayendo las races de los trminos cuadrados perfectos:
a a =
2

b b 2 4
2
=
se separan por el signo del otro trmino ( ) y el binomio se eleva al cuadrado: ( )
2
2b a
( )
2 2 2
2 4 4 b a b ab a = +
Pgina del Colegio de Matemticas de la ENP-UNAM Productos notables y factorizacin Autor: Dr. Jos Manuel Becerra Espinosa
19

3)
4 2 3 6
100 180 81 q q p p +
Se extraen las races de los trminos cuadrados perfectos:
3 6
9 81 p p =
2 4
10 100 q q =
se separan por el signo del otro trmino ( ) y el binomio se eleva al cuadrado: ( )
2
2 3
10 9 q p
( )
2
2 3 4 2 3 6
10 9 100 180 81 q p q q p p = +
4) ( )
2 2 2
7 6 49 84 36 b a b ab a + = + +
5) ( )
2 2 2
5 3 25 30 9 y x y xy x = +
6) ( )
2
6 4 12 6 4 8
5 10 25 100 100 z w z z w w = +
7)
2
2 2
13
4
3
2
169
16
39
16
9
4

+ = + + f e f ef e
8) ( )
2
6 2 3 12 2 6 2 3 4 6 6 2 3 4 6 12 2
3 2 9 12 4 12 4 9 wt r n t w wt r n r n wt r n r n t w = + = +

Operacin: Completar un trinomio cuadrado perfecto

Ejemplos.
Completar los siguientes TCP:

1) 9 ___
2
+ + x
Se extraen las races de los trminos cuadrados perfectos:
x x =
2

3 9 =
se multiplican estos dos trminos y se duplica el resultado: ( )( ) x x 6 3 2 =
por lo tanto el TCP completo es: 9 6
2
+ + x x
2)
2 2
25 ___ 16 d c + +
Las races de los trminos cuadrados perfectos son:
c c 4 16
2
=
d d 5 25
2
=
se multiplican estos dos trminos y se duplica el resultado: ( )( ) cd d c 40 5 4 2 =
por lo tanto el TCP completo es:
2 2
25 40 16 d cd c + +
3)
6 4
49 ___ 144 +
Extrayendo las races de los trminos cuadrados perfectos:
2 4
12 144 =
3 6
7 49 =
se multiplican estos dos trminos y se duplica el resultado: ( )( )
3 2 3 2
168 7 12 2 =
por lo tanto el TCP completo es:
6 3 2 4
49 168 144 +
4) ___ 16
2
+ + x x
Se extrae la raz del trmino cuadrado perfecto: x x =
2

Pgina del Colegio de Matemticas de la ENP-UNAM Productos notables y factorizacin Autor: Dr. Jos Manuel Becerra Espinosa
20

se divide el otro trmino entre la raz obtenida: 16
16
=
x
x

este resultado se divide por dos 8
2
16
= y, finalmente, se eleva al cuadrado: 64 8
2
=
por lo tanto el TCP completo es: 64 16
2
+ + x x
5) ___ 48 36
2 2
+ + ab a
La raz del trmino cuadrado perfecto es: a a 6 36
2
=
se divide el otro trmino entre la raz obtenida:
2
2
8
6
48
b
a
ab
=
este resultado se divide por dos
2
2
4
2
8
b
b
= y, finalmente, se eleva al cuadrado: ( )
4
2
2
16 4 b b =
por lo tanto el TCP completo es:
4 2 2
16 48 36 b ab a + +
6) ___ 312 144
4 5 10
+ + h g g
Extrayendo la raz del trmino cuadrado perfecto:
5 10
12 144 g g =
se divide el otro trmino entre la raz obtenida:
4
5
4 5
26
12
312
h
g
h g
=
este resultado se divide por dos
4
4
13
2
26
h
h
= y, finalmente, se eleva al cuadrado: ( )
8
2
4
169 13 h h =
por lo tanto el TCP completo es:
8 4 5 10
169 312 144 h h g g + +


V.2.5 FACTORIZACIN DE UNA DIFERENCIA DE CUADRADOS

Una diferencia de cuadrados es el resultado del producto de dos binomios conjugados:

( )( ) b a b a b a + =
2 2


Esto implica que para factorizar una diferencia de cuadrados, se extraen las races cuadradas de los
trminos y se forma un binomio. Finalmente se expresa el producto de este binomio por su conjugado.

Ejemplos.
Factorizar las siguientes expresiones:

1) 4
2
x
Se extraen las races de los trminos:
x x =
2

2 4 =
se forma el binomio: ( ) 2 + x y se multiplica por su conjugado:
( )( ) 2 2 + x x
por lo que: ( )( ) 2 2 4
2
+ = x x x
2)
4 2
16 25 b a
Las races de los trminos son:
a a 5 25
2
=
Pgina del Colegio de Matemticas de la ENP-UNAM Productos notables y factorizacin Autor: Dr. Jos Manuel Becerra Espinosa
21

2 4
4 16 b b =
se forma el binomio: ( )
2
4 5 b a + y se multiplica por su conjugado:
( )( )
2 2
4 5 4 5 b a b a +
as que: ( )( )
2 2 4 2
4 5 4 5 16 25 b a b a b a + =
3) ( )( ) m k m k m k 8 10 8 10 64 100
2 2
+ =
4) ( )( )
4 3 4 3 8 6
3 12 3 12 9 144 r n r n r n + =
5) ( )( )
6 5 6 5 12 10
25 7 25 7 625 49 p t p t p t + =
6) ( )( )
9 6 8 7 9 6 8 7 18 12 16 14
16 20 16 20 256 400 h g f e h g f e h g f e + =
7)

+ = b a b a b a
11
6
2
1
11
6
2
1
121
36
4
1
2 2

8) ( ) ( ) ( ) ( ) ( ) ( ) [ ]
2 2 2 2 3 2 3
1 3 1 3 1 3 1 3 3 1 1 3 w z z w z w z z w z = = +
( ) ( ) [ ] ( ) [ ] ( )( )( ) w z w z z w z w z z + = + = 1 3 1 3 1 3 1 3 1 3 1 3


V.2.6 FACTORIZACIN DE UN TRINOMIO DE LA FORMA c bx x
n
n
+ +
2


Para factorizar un trinomio de la forma c bx x
n
n
+ +
2
, donde
n
x es un cuadrado perfecto y n natural par,
se expresa como producto de dos binomios cuyo primer trmino para ambos sea la raz cuadrada de
n
x ,
es decir,
2
n
x . Por su parte, los trminos no comunes de este producto de binomios deben cumplir con la
doble condicin de que su suma sea igual al coeficiente b y su producto igual al coeficiente c .

En general:

Si el trmino c es positivo entonces los dos nmeros buscados tienen el mismo signo. Si b es
positivo los nmeros son positivos. Si b es negativo los nmeros son negativos.
Si el trmino c es negativo entonces los nmeros buscados tienen signos contrarios y el signo del
nmero ms grande es el mismo que el del coeficiente b .

Ejemplos.
Factorizar los siguientes trinomios:

1) 10 7
2
+ + x x
La raz del primer trmino es: x x =
2

el trmino c es positivo y b tambin lo es, por lo que los dos nmeros buscados que sumados sean 7 y
multiplicados sea 10 son positivos. Estos nmeros son 5 y 2 .
Por lo tanto: ( )( ) 2 5 10 7
2
+ + = + + x x x x
2) 24 11
2
+ x x
La raz del primer trmino es: x x =
2

el trmino c es positivo y b es negativo, por lo que los dos nmeros buscados que sumados sean 11
y multiplicados sea 24 son negativos. Estos nmeros son 8 y 3 .
Por lo tanto: ( )( ) 3 8 24 11
2
= + x x x x
Pgina del Colegio de Matemticas de la ENP-UNAM Productos notables y factorizacin Autor: Dr. Jos Manuel Becerra Espinosa
22

3) 28 3
2 4
+ k k
La raz del primer trmino es:
2 4
k k =
el trmino c es negativo y b es positivo, por lo que los dos nmeros buscados que sumados sean 3 y
multiplicados sea 28 tienen signos contarios y el ms grande es positivo. Estos nmeros son 7 y 4 .
Por lo tanto: ( )( ) 4 7 28 3
2 2 2 4
+ = + k k k k
4) 15 2
3 6
z z
La raz del primer trmino es:
3 6
z z =
el trmino c es negativo y b tambin lo es, por lo que los dos nmeros buscados que sumados sean 2 y
multiplicados sea 15 tienen signos contarios y el ms grande es negativo. Estos nmeros son 5 y 3.
Por lo tanto: ( )( ) 3 5 15 2
3 3 3 6
+ = z z z z
5) ( )( ) 4 5 20 9
4 4 4 8
+ + = + + w w w w
6) ( )( ) 4 9 36 13
5 5 5 10
= + m m m m
7) ( )( ) 3 20 60 17
2 2 2 4
+ = + x x x x
8) ( )( ) 5 15 75 10
6 6 6 12
+ = n n n n
9) ( ) ( ) ( )( ) 2 3 4 3 8 3 2 3 8 6 9
2 2
+ = + = + x x x x x x
10) ( ) ( ) ( )( ) 1 2 5 2 5 2 4 2 5 8 4
3 3 3
2
3 3 6
+ = + = + a a a a a a


V.2.7 FACTORIZACIN DE UN TRINOMIO DE LA FORMA c bx ax + +
2


Para factorizar un trinomio de la forma c bx ax + +
2
, se efecta el siguiente procedimiento
1
:

Se multiplican todos los trminos por el coeficiente a
Se expresa el primer trmino en forma de cuadrado y para el segundo trmino se intercambia el
coeficiente a por b
Se factoriza aplicando el caso anterior
Se divide el resultado entre a de forma tal que no quede ningn cociente.

Ejemplos.
Factorizar los siguientes trinomios:

1) 2 7 6
2
+ + x x
Multiplicando los trminos del trinomio por 6 : ( ) ( ) ( ) 2 6 7 6 6 6
2
+ + x x
expresando el primer trmino en forma de cuadrado y para el segundo trmino se intercambia el
coeficiente 6 por el 7 : ( ) ( ) 12 6 7 6
2
+ + x x
aplicando el caso anterior de factorizacin se buscan dos nmeros que sumados sean 7 y multiplicados
sean 12 se tiene: ( )( ) 3 6 4 6 + + x x
se divide por 6 de forma que no queden cocientes:
( )( ) ( ) ( )
( )( ) 1 2 2 3
3
3 6
2
4 6
6
3 6 4 6
+ + =
+ +
=
+ +
x x
x x x x

por lo tanto: ( )( ) 1 2 2 3 2 7 6
2
+ + = + + x x x x


1
Generalmente a no es cuadrado perfecto y an sindolo no es de la forma c bx x
n
n
+ +
2
porque b no es entero.
Pgina del Colegio de Matemticas de la ENP-UNAM Productos notables y factorizacin Autor: Dr. Jos Manuel Becerra Espinosa
23

2) 2 3 2
2
+ x x
Multiplicando los trminos del trinomio por 2 : ( ) ( ) ( ) 2 2 3 2 2 2
2
+ x x
expresando el primer trmino en forma de cuadrado y para el segundo trmino se intercambia el
coeficiente 3 por el 2 : ( ) ( ) 4 2 3 2
2
+ x x
aplicando el caso anterior de factorizacin se buscan dos nmeros que sumados sean 3 y multiplicados
sean 4 se tiene: ( )( ) 1 2 4 2 + x x
se divide por 2 de forma que no queden cocientes:
( )( ) ( ) ( )
( )( ) 1 2 2
1
1 2
2
4 2
2
1 2 4 2
+ =
+
=
+
x x
x x x x

por lo tanto: ( )( ) 1 2 2 2 3 2
2
+ = + x x x x
3) 6 13 5
2 4
k k
Multiplicando los trminos del trinomio por 5: ( ) ( ) ( ) 6 5 13 5 5 5
2 4
k k
expresando el primer trmino en forma de cuadrado y para el segundo trmino se intercambia el
coeficiente 5 por el 13: ( ) ( ) 30 5 13 5
2
2
2
k k
aplicando el caso anterior de factorizacin se buscan dos nmeros que sumados sean 13 y
multiplicados sean 30 se tiene: ( )( ) 2 5 15 5
2 2
+ k k
se divide por 5 de forma que no queden cocientes:
( )( ) ( ) ( )
( )( ) 2 5 3
1
2 5
5
15 5
5
2 5 15 5
2 2
2 2 2 2
+ =
+
=
+
k k
k k k k

por lo tanto: ( )( ) 2 5 3 6 13 5
2 2 2 4
+ = k k k k


4) 9 15 4
3 6
+ q q
( ) ( ) ( ) ( ) ( ) ( )( ) 3 4 12 4 36 4 15 4 9 4 15 4 4 4
3 3 3
2
3 3 6
= + = + q q q q q q
( ) ( )
( )( ) 3 4 3
1
3 4
4
12 4
3 3
3 3
=

q q
q q

( )( ) 3 4 3 9 15 4
3 3 3 6
= + q q q q
5) 15 14 8
2
x x
( ) ( ) ( ) ( ) ( ) ( )( ) 6 8 20 8 120 8 14 8 15 8 14 8 8 8
2 2
+ = = x x x x x x
( ) ( )
( )( ) 3 4 5 2
2
6 8
4
20 8
+ =
+
x x
x x

( )( ) 3 4 5 2 15 14 8
2
+ = x x x x
6) 1 10 9
4 8
+ +
( ) ( ) ( ) ( ) ( ) ( )( ) 1 9 9 9 9 9 10 9 1 9 10 9 9 9
4 4 4
2
4 4 8
+ + = + + = + +
( ) ( )
( )( ) 1 9 1
1
1 9
9
9 9
4 4
4 4
+ + =
+ +



( )( ) 1 9 1 1 10 9
4 4 4 8
+ + = + +
7) 33 4
2
+ y y
( ) ( ) ( ) ( ) ( ) ( )( ) 11 4 12 4 132 4 1 4 33 4 4 4 4
2 2
+ = + = + y y y y y y
Pgina del Colegio de Matemticas de la ENP-UNAM Productos notables y factorizacin Autor: Dr. Jos Manuel Becerra Espinosa
24

( ) ( )
( )( ) 11 4 3
1
11 4
4
12 4
+ =
+
y y
y y

( )( ) 11 4 3 33 4
2
+ = + y y y y
8) 24 46 7
5 10
+ + z z
( ) ( ) ( ) ( ) ( ) ( )( ) 4 7 42 7 168 7 46 7 24 7 46 7 7 7
5 5 5
2
5 5 10
+ + = + + = + + z z z z z z
( ) ( )
( )( ) 4 7 6
1
4 7
7
42 7
5 5
5 5
+ + =
+ +
z z
z z

( )( ) 4 7 6 24 46 7
5 5 5 10
+ + = + + z z z z


V.2.8 FACTORIZACIN DEL CUBO DE UN BINOMIO

Una cantidad es cubo perfecto cuando es el producto de tres factores iguales, es decir, es el cubo de otra cantidad.

Por ejemplo,
3
125k es cubo perfecto, ya que es el cubo de k 5 .

El cubo de un binomio es de la forma:

( )
3 2 2 3 3
3 3 b ab b a a b a + + + = +

y cumple con las siguientes caractersticas:

Posee cuatro trminos.
El primero como el ltimo trmino son cubos perfectos
El segundo trmino es el triple producto del cuadrado de la raz cbica del primer trmino por la raz
cbica del ltimo.
El tercer trmino es el triple producto de la raz cbica del primer trmino por el cuadrado de la raz
cbica del ltimo.

Para verificar que la factorizacin de una expresin de cuatro trminos es el cubo de un binomio se debe
proceder de la siguiente manera:

1. Se ordena el polinomio en forma descendente o ascendente respecto a una literal.
2. Se extrae la raz cbica del primer y ltimo trminos del polinomio.
3. Se observa si todos los signos son iguales o si se alternan.
4. Se triplica el cuadrado de la raz cbica del primer trmino por la raz cbica del ltimo y se compara
con el segundo trmino del polinomio dado.
5. Se triplica la raz cbica del primer trmino por el cuadrado de la raz cbica del ltimo y se compara
con el tercer trmino de la expresin.
6. Si las dos comparaciones hechas en los pasos previos son iguales, se trata del desarrollo del cubo
de un binomio y se factoriza as: se forma un binomio con las races cbicas del primer y ltimo
trmino del polinomio, con los signos que se obtengan (si todos los signos son iguales) o por el signo
menos (si los signos se alternan). Finalmente, se eleva el binomio al cubo.

Ejemplos.
Factorizar los siguientes polinomios:

1) 1 3 3
2 3
+ + + k k k
Se extraen las races cbicas de los trminos extremos:
k k =
3 3

Pgina del Colegio de Matemticas de la ENP-UNAM Productos notables y factorizacin Autor: Dr. Jos Manuel Becerra Espinosa
25

1 1
3
=
El triple producto del cuadrado de la raz cbica del primer trmino por la raz cbica del ltimo es:
( ) ( )
2 2
3 1 3 k k = , que es igual al segundo trmino.
El triple producto de la raz cbica del primer trmino por el cuadrado de la raz cbica del ltimo es:
( )( ) k k 3 1 3
2
= , que es igual al tercer trmino.
Dado que todos los signos son positivos, el binomio al cubo formado por las races cbicas de los
extremos es: ( )
3
1 + k , as que: ( )
3 2 3
1 1 3 3 + = + + + k k k k
2)
3 2
9 27 27 x x x +
Se extraen las races cbicas de los trminos extremos:
3 27
3
=
x x =
3 3

El triple producto del cuadrado de la raz cbica del primer trmino por la raz cbica del ltimo es:
( ) ( ) x x 27 3 3
2
= , que es igual al segundo trmino.
El triple producto de la raz cbica del primer trmino por el cuadrado de la raz cbica del ltimo es:
( )( )
2 2
9 3 3 x x = , que es igual al tercer trmino.
Dado que los signos se alternan, el binomio al cubo formado por las races cbicas de los extremos es:
( )
3
3 x , as que: ( )
3 3 2
3 9 27 27 x x x x = +
3) n m n m mn
2 3 3 2
3 3 + + +
Se ordena el polinomio con respecto a m:

3 2 2 3
3 3 n mn n m m + + +
se extraen las races cbicas de los trminos extremos:
m m =
3 3

n n =
3 3

El triple producto del cuadrado de la raz cbica del primer trmino por la raz cbica del ltimo es:
( ) ( ) n m n m
2 2
3 3 = , que es igual al segundo trmino.
El triple producto de la raz cbica del primer trmino por el cuadrado de la raz cbica del ltimo es:
( )( )
2 2
3 3 mn n m = , que es igual al tercer trmino.
Dado que todos los signos son positivos, el binomio al cubo formado por las races cbicas de los
extremos es: ( )
3
n m+ , as que: ( )
3 3 2 2 3
3 3 n m n mn n m m + = + + +
4)
2 6 4 3 6 9
12 6 8 p q p q p q +
Se ordena el polinomio con respecto a q :

6 4 3 2 6 9
6 12 8 p p q p q q +
se extraen las races cbicas de los trminos extremos:
3
3
9
2 8 q q =
2
3
6
p p =
El triple producto del cuadrado de la raz cbica del primer trmino por la raz cbica del ltimo es:
( ) ( )
2 6 2
2
3
12 2 3 p q p q = , que es igual al segundo trmino.
El triple producto de la raz cbica del primer trmino por el cuadrado de la raz cbica del ltimo es:
( )( )
4 3
2
2 3
6 2 3 p q p q = , que es igual al tercer trmino.
Pgina del Colegio de Matemticas de la ENP-UNAM Productos notables y factorizacin Autor: Dr. Jos Manuel Becerra Espinosa
26

Dado que los signos se alternan, el binomio al cubo formado por las races cbicas de los extremos es:
( )
3
2 3
2 p q , as que: ( )
3
2 3 6 4 3 2 6 9
2 6 12 8 p q p p q p q q = +
5) ( )
3 2 3 2 3
1 5 1 15 75 125 15 75 1 125 + = + + + = + + + x x x x x x x
6) ( )
3
2 6 4 2 2 6 4
2 6 12 8 12 6 8 w w w w w w w = + = +
7) ( )
3
4 3 12 8 3 4 6 9 8 3 4 6 12 9
5 4 125 300 240 64 300 240 125 64 y x y y x y x x y x y x y x = + = +
8) ( )
3
3 2 3 2 6 4 9 6 6 4 9 6 3 2
1 6 1 18 108 216 108 216 1 18 + = + + + = + + + + b a b a b a b a b a b a b a


V.2.9 FACTORIZACIN DE LA SUMA O DIFERENCIA DE DOS POTENCIAS IGUALES

Sea n un nmero entero positivo.

La suma de potencias iguales impares es siempre divisible por la suma de las bases. Esto es:
n n
b a + es divisible por b a + . Por lo tanto: ( )( )
1 2 3 2 1
+ + + = +
n n n n n n
b b a b a a b a b a L

La suma de potencias iguales pares, no es divisible ni por la suma ni por la diferencia de las bases a
menos de que sea posible transformarla en una suma equivalente de potencias impares.

La diferencia de potencias iguales, sean pares o impares, es siempre divisible por la diferencia de las bases.
Esto es:
n n
b a es divisible por b a . Por lo tanto: ( )( )
1 2 3 2 1
+ + + + =
n n n n n n
b b a b a a b a b a L

La diferencia de potencias iguales pares, es siempre divisible por la suma de las bases. Esto es:
n n
b a es divisible por b a + . Por lo tanto: ( )( )
1 2 3 2 1
+ + + =
n n n n n n
b b a b a a b a b a L

Ejemplos.
Factorizar las siguientes sumas de potencias iguales:

1)
3 3
b a +
Solucin.
Las potencias son impares, entonces es divisible por b a + :

2 2
3 2
3 2
2 2
3 2
2 3
3 3
0
b ab a
b ab
b ab
ab b a
b b a
b a a
b a b a
+

+
+
+

+ +

Por lo tanto: ( )( )
2 2 3 3
b ab a b a b a + + = +

2) 32
5
+ k
Solucin.
Pgina del Colegio de Matemticas de la ENP-UNAM Productos notables y factorizacin Autor: Dr. Jos Manuel Becerra Espinosa
27

5 5 5
2 32 + = + k k , las potencias son impares, entonces es divisible por 2 + k :

16 8 4 2
0
32 16
32 16
16 8
32 8
8 4
32 4
4 2
32 2
2
32 2
2 3 4
2
2
2 3
3
3 4
4
4 5
5
+ +

+
+
+

+
+
+

+ +
k k k k
k
k
k k
k
k k
k
k k
k
k k
k k


Por lo tanto: ( )( ) 16 8 4 2 2 32
2 3 4 5
+ + + = + k k k k k k

3)
3 3
b a
Solucin.
La expresin es divisible por b a :

2 2
3 2
3 2
2 2
3 2
2 3
3 3
0
b ab a
b ab
b ab
ab b a
b b a
b a a
b a b a
+ +
+

+



Por lo tanto: ( )( )
2 2 3 3
b ab a b a b a + + =

4) 729
6
+ x
Solucin:
6 6 6
3 729 + = + x x , las potencias son pares, entonces no es divisible por 3 + x ni por 3 x .
Sin embargo, 729
6
+ x equivale a ( ) ( )
3
2
3
2
3 + x , expresin que es factorizable ya que:

Pgina del Colegio de Matemticas de la ENP-UNAM Productos notables y factorizacin Autor: Dr. Jos Manuel Becerra Espinosa
28

( )
( ) ( )
( )
( ) ( )
( )
( )
( ) ( )
2
1
2
2
2
3 2 2
3 2 2
2 2
2
2
3
2
2
2
2
3
2
3
3
2 2
9 9
0
9 9
9 9
9 9
9 9
9
9 9
+

+
+
+

+ +
x x
x
x
x x
x
x x
x x


Por lo tanto: ( )( ) 81 9 9 729
2 4 2 6
+ + = + x x x x

5) 729
6
x
Solucin:
Las potencias son pares, entonces es divisible por 3 + x :

343 81 27 9 3
0
729 343
729 343
343 81
729 81
81 27
729 27
27 9
729 9
9 3
729 3
3
729 3
2 3 4 5
2
2
2 3
3
3 4
4
4 5
5
5 6
6
+ +
+

+


+
x x x x x
x
x
x x
x
x x
x
x x
x
x x
x
x x
x x


Por lo tanto: ( )( ) 343 81 27 9 3 3 729
2 3 4 5 6
+ + + = + x x x x x x x .
La expresin, 729
6
+ x tambin se puede expresar como ( ) ( )
2
2
3
27 x , que es una diferencia de
cuadrados, por lo tanto, su mxima factorizacin es: ( )( ) 27 27 729
3 3 6
+ = + x x x y se puede ver la
ventaja sobre el planteamiento anterior para obtener la mxima factorizacin.

6)
4 4
q p
Solucin.
Las potencias son pares, entonces es divisible por q p + :
Pgina del Colegio de Matemticas de la ENP-UNAM Productos notables y factorizacin Autor: Dr. Jos Manuel Becerra Espinosa
29

3 2 2 3
4 3
4 3
3 2 2
4 2 2
2 2 3
4 3
3 4
4 4
0
q pq q p p
q pq
q pq
pq q p
q q p
q p q p
q q p
q p p
q p q p
+
+

+


+


Por lo tanto: ( )( )
3 2 2 3 4 4
q pq q p p q p q p + + = . Factorizando por agrupacin se obtiene su
mxima factorizacin: ( ) ( ) ( ) [ ] ( )( )( )
2 2 2 2 4 4
q p q p q p q p q q p p q p q p + + = + + =
Este mismo ejercicio pudo hacerse factorizando la diferencia de cuadrados: ( )( )
2 2 2 2 4 4
q p q p q p + = y
se puede ver la ventaja sobre el planteamiento anterior para obtener la mxima factorizacin.

7) 128
7
x
Solucin.
7 7 7
2 128 = x x , la expresin es divisible por 2 x :

64 32 16 8 4 2
0
128 64
128 64
64 32
128 32
32 16
128 16
16 8
128 8
8 4
128 4
4 2
128 2
2
128 2
2 3 4 5 6
2
2
2 3
3
3 4
4
4 5
5
5 6
6
6 7
7
+ + + + + +
+

+

x x x x x x
x
x
x x
x
x x
x
x x
x
x x
x
x x
x
x x
x x

Por lo tanto: ( )( ) 64 32 16 8 4 2 2 128
2 3 4 5 6 7
+ + + + + + = x x x x x x x x

Pgina del Colegio de Matemticas de la ENP-UNAM Productos notables y factorizacin Autor: Dr. Jos Manuel Becerra Espinosa
30

V.2.10 MNIMO COMN MLTIPLO DE POLINOMIOS

El mnimo comn mltiplo (MCM) de dos o ms expresiones algebraicas es la expresin algebraica de
menor coeficiente numrico y de menor grado que es divisible exactamente por cada una de las
expresiones dadas.

Ejemplos.

1) k 10 es el MCM de k 2 y de 5
2)
2
12ab es el MCM de a 3 y de
2
4b
3)
4 2 3
60 z y x es el MCM de
4
3xyz , de
2 3
5 y x , de z 4 y de
3 2 2
2 z y x .

Para obtener el mnimo comn mltiplo de monomios se encuentra el MCM de los coeficientes y a
continuacin se escriben las literales comunes y no comunes, dando a cada literal el mayor exponente
que tengan las expresiones dadas.

Ejemplos.
Obtener el mnimo comn mltiplo de los siguientes monomios:

1)
2 3
4 b a y
4
6ab
el MCM es:
4 3
12 b a
2) z y x
2 3
6 y
2 4
8 z xy
el MCM es:
2 4 3
24 z y x
3)
3 2 2
5 m j k y p kn
5
2
el MCM es: p n m j k
5 3 2 2
10
4) d c b a
3 2 3
8 y
2 4 6
12 e c b
el MCM es:
2 4 6 3
24 de c b a
5)
2
6mn ,
3 2
5 n m y n m
3
12
el MCM es:
3 3
60 n m
6)
3
3 ,
2 2
36 y
4 2
24
el MCM es:
4 2 2 3
72
7)
3 2
5 rs pq ,
3 2
4 r p ,
2 2
3 s q y s r p
4 4
8
el MCM es:
3 4 2 4
120 s r q p
8)
3 2
24 x a ,
4 2
36 y a ,
5 2
12 y x y
6 3
60 y a
el MCM es:
6 3 3
360 y x a

Para encontrar el mnimo comn mltiplo de polinomios primero se factorizan los polinomios dados en
sus factores primos y despus se multiplican (conservando el MCM en forma factorizada) los factores
primos, comunes y no comunes con su mayor exponente.

Ejemplos.
Obtener el mnimo comn mltiplo de los siguientes polinomios:
1) 5 5 + x y 10 10 x
Tomando como factor comn a 5 para la primera expresin:
( ) 1 5 + x
Pgina del Colegio de Matemticas de la ENP-UNAM Productos notables y factorizacin Autor: Dr. Jos Manuel Becerra Espinosa
31

Tomando como factor comn a 10 para la segunda expresin:
( ) 1 10 x
el MCM es: ( )( ) 1 1 10 + x x
2) ( )
2
1 a y 1
2
a
Factorizando cada una de las expresiones:
( ) ( )( ) 1 1 1
2
= a a a
( )( ) 1 1 1
2
+ = a a a
el MCM es: ( ) ( ) 1 1
2
+ a a
3) y x x
2 3
2 + y
2 2
4y x
Tomando como factor comn
2
x para la primera expresin:
( ) y x x 2
2
+
Factorizando la segunda expresin:
( )( ) y x y x 2 2 +
el MCM es: ( )( ) y x y x x 2 2
2
+

4)
2 2
4 8 4 bz byz by + y z c y c
2 2
6 6
Tomando como factor comn b 4 para la primera expresin:
( )
2 2
2 4 z yz y b +
Factorizando el TCP:
( )
2
4 z y b
Tomando como factor comn
2
6c para la segunda expresin:
( ) z y c
2
6
el MCM es: ( )
2 2
12 z y bc
5) 4
2
x , 6
2
x x y 4 4
2
+ + x x
Factorizando cada una de las expresiones:
( )( ) 2 2 4
2
+ = x x x
( )( ) 2 3 6
2
+ = x x x x
( )
2 2
2 4 4 + = + + x x x
el MCM es: ( ) ( )( ) 3 2 2
2
+ x x x
6) 2
2
+ k k , 3 4
2
+ k k y 10 3
2
k k
Factorizando todas las expresiones:
( )( ) 1 2 2
2
+ = + k k k k
( )( ) 1 3 3 4
2
= + k k k k
( )( ) 2 5 10 3
2
+ = k k k k
el MCM es: ( )( )( )( ) 5 3 1 2 + k k k k
7) 25
2
z , 125
3
z y 10 2 + z
Factorizando las expresiones:
( )( ) 5 5 25
2
+ = z z z
Pgina del Colegio de Matemticas de la ENP-UNAM Productos notables y factorizacin Autor: Dr. Jos Manuel Becerra Espinosa
32

( )( ) 25 5 5 125
2 3
+ + = z z z z
( ) 5 2 10 2 + = + z z
el MCM es: ( )( )( ) 25 5 5 5 2
2
+ + + z z z z
8) a ax ax 14 5
2
+ , x x x 49 14
2 3
+ + y
2 3 4
18 7 x x x +
Factorizando todas las expresiones:
( ) ( )( ) 2 7 14 5 14 5
2 2
+ = + = + x x a x x a a ax ax
( ) ( )
2 2 2 3
7 49 14 49 14 + = + + = + + x x x x x x x x
( ) ( )( ) 2 9 18 7 18 7
2 2 2 2 3 4
+ = + = + x x x x x x x x x
el MCM es: ( ) ( )( ) 9 2 7
2 2
+ + x x x ax


Pgina del Colegio de Matemticas de la ENP-UNAM Operaciones con fracciones algebraicas y radicales Autor: Dr. Jos Manuel Becerra Espinosa
1

OPERACIONES CON FRACCIONES
ALGEBRAICAS Y RADICALES

UNIDAD VI


VI.1 TEOREMAS DEL RESIDUO Y DEL FACTOR

Sea un polinomio en x de la forma:

( )
0 1
3
3
2
2
1
1
a x a x a x a x a x a x P
n
n
n
n
n
n
n
n
+ + + + + + =



donde
0 2 1
, , , , a a a a
n n n
L

son coeficientes numricos y n N,

se dice que c R es un cero o raz, de ( ) x P si y slo si ( ) 0 = c P . Es decir, la raz de un polinomio es
el nmero que toma la variable para que el valor numrico de ( ) x P sea cero.

Ejemplos.
1) En el polinomio ( ) 1
2
= x x P , sus races son:
1 = x ya que ( ) ( ) 0 1 1 1 1 1
2
= = = P
1 = x ya que ( ) ( ) 0 1 1 1 1 1
2
= = = P

2) En el polinomio ( ) x x x P =
2
4 , sus ceros son:
0 = x ya que ( ) ( ) 0 0 0 0 0 4 0
2
= = = P
4
1
= x ya que 0
4
1
16
4
4
1
4
1
4
4
1
2
= =

P

3) En el polinomio ( ) x x x x P 6 5
2 3
+ = , sus races son:
0 = x ya que ( ) ( ) ( ) 0 0 0 0 0 6 0 5 0 0
2 3
= + = + = P
2 = x ya que ( ) ( ) ( ) 0 12 20 8 2 6 2 5 2 2
2 3
= + = + = P
3 = x ya que ( ) ( ) ( ) 0 18 45 27 3 6 3 5 3 3
2 3
= + = + = P

Algoritmo de la divisin para polinomios

Dados dos polinomios ( ) x P (llamado dividendo) y ( ) x Q (llamado divisor) de modo que el grado del
dividendo sea mayor que el grado del divisor y ( ) 0 x Q .

Entonces, para
( )
( ) x Q
x P
existen dos polinomios nicos ( ) x c y ( ) x r tales que cumplen con:

( ) ( ) ( ) ( ) x r x c x Q x P + =

El polinomio ( ) x c se llama cociente y ( ) x r es el residuo de la divisin cuyo grado es menor que el de ( ) x P .
Pgina del Colegio de Matemticas de la ENP-UNAM Operaciones con fracciones algebraicas y radicales Autor: Dr. Jos Manuel Becerra Espinosa
2

Sean un polinomio ( ) x P de grado 1 n y a R.

Teorema del residuo

Si el polinomio ( ) x P se divide por a x , entonces el residuo es ( ) a P .

Demostracin:
Si se divide ( ) x P entre a x se tiene:

( ) ( )( ) R a x x Q x P + =

donde ( ) x Q es el cociente y R es el residuo.
Si ahora se evala a x = se obtiene:

( ) ( )( ) R R R a a a Q a P = + = + = 0
De donde ( ) a P es el residuo.

Ejemplo.
Sea el polinomio: ( ) 11 5 9 4
2 3
+ = x x x x P , comprobar el teorema de residuo si se divide por 2 x .

Solucin.
Dividiendo el polinomio por 2 x :
3 4
5
6 3
11 3
2
11 5
8 4
11 5 9 4 2
2
2
2
2 3
2 3
+

+
+
+
x x
x
x
x x
x x
x x
x x x x
ahora, evaluando para 2 = x :
( ) ( ) ( ) ( ) 5 11 10 36 32 11 2 5 2 9 2 4 2
2 3
= + = + = P
Los resultados son iguales, lo que comprueba el teorema del residuo.

Teorema del factor

Si a es una raz del polinomio ( ) x P , entonces a x es un factor del polinomio. O bien, si a x es un
factor de ( ) x P , entonces a es una raz del polinomio. Esto es:

( ) a x a P = 0 es un factor de ( ) x P .

Demostracin:
Si a x es factor de ( ) x P entonces se cumple que: ( ) ( )( ) a x x Q x P = porque ( ) ( )( ) 0 = = a a a Q a P
por lo tanto, a es raz de la ecuacin ( ) 0 = x P .
Pgina del Colegio de Matemticas de la ENP-UNAM Operaciones con fracciones algebraicas y radicales Autor: Dr. Jos Manuel Becerra Espinosa
3

Pero si a es raz de la ecuacin ( ) 0 = x P , esto implica que ( ) 0 = a P
Si se aplica el teorema del residuo se tiene que:
( ) ( )( ) ( ) ( )( ) ( )( ) a x x Q a x x Q a P a x x Q x P = + = + = 0
por lo tanto a x es factor de ( ) x P .

Ejemplo
Determinar si 2 + x es factor del polinomio ( ) 10 4
2 3
+ = x x x x P

Solucin:
Si 2 + x es factor, 2 = x es raz, entonces debe cumplir que el residuo sea cero:
( ) ( ) ( ) ( ) 0 10 2 16 8 10 2 2 4 2 2
2 3
= + + = + = P
Por lo tanto, 2 + x es factor del polinomio
Comprobando:
5 2
0
10 5
10 5
4 2
10 2
2
10 4 2
2
2
2
2 3
2 3
+
+




+ +
x x
x
x
x x
x x
x x
x x x x
Por lo tanto se cumple que: ( )( ) 2 5 2 10 4
2 2 3
+ + = + x x x x x x .

Divisin sinttica

Por el teorema del residuo, si a es una raz del polinomio ( ) x P , entonces ( ) x P es divisible por a x ,
pues el residuo de dividir ( ) x P entre a x es cero. A cada uno de las races se les designa por
n
x , , x , x , x L
3 2 1
.

Esto es, dado el polinomio ( )
0 1
3
3
2
2
1
1
a x a x a x a x a x a x P
n
n
n
n
n
n
n
n
+ + + + + + =

, entonces se
puede factorizar como: ( ) ( )( )( ) ( )
n
x x x x x x x x x P = L
3 2 1
, es decir, un polinomio de grado n
tiene exactamente n races.

La principal razn de factorizar un polinomio es encontrar sus races. Generalmente, para reconocer las races
enteras de un polinomio con coeficientes enteros se tiene en cuenta que stas son divisores del trmino
independiente. As, las races enteras del polinomio ( ) 12 4 9 6
2 3 4
+ + = x x x x x P estn entre los
divisores de 12 . Por lo tanto, pueden ser races de ( ) x P los nmeros 12 6 6 4 4 3 3 2 2 1 1 , , , , , , , , , ,
y 12 .

En el polinomio anterior, si se prueba para 1 = x :
( ) ( ) ( ) ( ) ( ) 4 12 4 9 6 1 12 1 4 1 9 1 6 1 1
2 3 4
= + + = + + = P , puesto que el residuo es distinto de
cero, se concluye que ( ) x P no es divisible por 1 x .
Pgina del Colegio de Matemticas de la ENP-UNAM Operaciones con fracciones algebraicas y radicales Autor: Dr. Jos Manuel Becerra Espinosa
4

Ahora, si se prueba para 1 = x :
( ) ( ) ( ) ( ) ( ) 0 12 4 9 6 1 12 1 4 1 9 1 6 1 1
2 3 4
= + + = + + = P , puesto que el residuo es cero,
se concluye que ( ) x P es divisible por 1 + x .

Para descomponerlo en factores se prueba sucesivamente por todas ellas aplicando un algoritmo
llamado Regla de Ruffini que aplica el teorema del residuo verificando cual de estos valores da como
residuo cero. Este es un procedimiento que permite hallar el cociente y el residuo sin efectuar la
secuencia descrita anteriormente. Esta regla aplica slo si el divisor es un polinomio de la forma a x .

En general, la divisin sinttica es un procedimiento abreviado para realizar la divisin de un polinomio de
la forma ( )
0 1
3
3
2
2
1
1
a x a x a x a x a x a x P
n
n
n
n
n
n
n
n
+ + + + + + =

entre un polinomio lineal


expresado como a x y slo sirve para obtener las races enteras.

La metodologa para encontrar las races enteras de un polinomio mediante la divisin sinttica es la siguiente:

La disposicin prctica requiere que en un primer rengln se escriban los coeficientes del dividendo
ordenado de forma descendente y completo hasta el trmino independiente. A la izquierda de una lnea
vertical se escribe un valor de prueba como probable raz, que como ya se mencion es un divisor de
0
a .
El primer coeficiente del dividendo se copia abajo en una tercera fila en la misma columna. Se
multiplica el valor de prueba por el primer coeficiente de la tercera fila y el resultado se escribe debajo
del siguiente coeficiente del dividendo.
Se suman los coeficientes de la segunda columna y el resultado se escribe en la tercera fila.
El resultado obtenido en el paso anterior reinicia el ciclo: se multiplica por el valor de prueba y el
resultado se escribe debajo del siguiente coeficiente del dividendo. Nuevamente se suman los
coeficientes de la tercera columna y el resultado se escribe en la tercera fila
El proceso contina hasta que se obtenga el resultado de la ltima columna. Este valor es el residuo.
Si es cero entonces el valor de prueba es una raz del polinomio
De no ser una raz, se repite la metodologa con otro valor de prueba hasta encontrar un valor cuyo
residuo sea cero.
Cuando el residuo es cero, los valores de la tercera fila representan los coeficientes del polinomio
reducido y se efecta el mismo procedimiento con estos coeficientes hasta que se llegue a un
polinomio de grado uno, a fin de que se pueda despejar x para obtener la ltima raz.

Ejemplo.
Encontrar las races enteras de los siguientes polinomios:

1) 0 6
2
= x x
Solucin.
Las posibles races son: 6 3 3 2 2 1 1 , , , , , , y 6
Probando con 1 = x :
6 0 1
0 1
6 1 1
1



Por lo tanto, no es raz.
Probando con 3 = x :
6 4 1
12 3
6 1 1
3



Pgina del Colegio de Matemticas de la ENP-UNAM Operaciones con fracciones algebraicas y radicales Autor: Dr. Jos Manuel Becerra Espinosa
5

Por lo tanto, no es raz.
Probando con 3 = x :
0 2 1
6 3
6 1 1
3


La primera raz es 3
1
= x
El polinomio reducido que queda es: 0 2 = + x
despejando se tiene la segunda raz: 2
2
= x

2) 0 24 22 4 2
2 3
= + x x x
Solucin.
Las posibles races son: 24 12 12 8 8 6 6 4 4 3 3 2 2 1 1 , , , , , , , , , , , , , , y 24 .
Probando con 4 = x :
80 26 12 2
104 48 8
24 22 4 2
4




Por lo tanto, no es raz.
Probando con 1 = x :
0 24 2 2
24 2 2
24 22 4 2
1




La primera raz es 1
1
= x
Trabajando ahora con el polinomio reducido:
Probando con 2 = x :
20 2 2
4 4
24 2 2
2



Por lo tanto, no es raz.
Probando con 4 = x :
0 6 2
24 8
24 2 2
4


La segunda raz es 4
2
= x
El polinomio reducido que queda es: 0 6 2 = + x
despejando se tiene la tercera raz: 3
3
= x

3) 0 15 13 3
2 3
= + x x x
Solucin.
Las posibles races son: 15 5 5 3 3 1 1 , , , , , , y 15 .
Probando con 1 = x :
Pgina del Colegio de Matemticas de la ENP-UNAM Operaciones con fracciones algebraicas y radicales Autor: Dr. Jos Manuel Becerra Espinosa
6

24 9 4 1
9 4 1
15 13 3 1
1



Por lo tanto, no es raz.
Probando con 3 = x :
24 13 0 1
39 0 3
15 13 3 1
3



Por lo tanto, no es raz.
Probando con 5 = x :
0 3 2 1
15 10 5
15 13 3 1
5



La primera raz es 5
1
= x
Por lo tanto, no es raz.
Probando con 1 = x :
0 3 1
3 1
3 2 1
1


La segunda raz es 1
2
= x
El polinomio reducido que queda es: 0 3 = + x
despejando se tiene la tercera raz: 3
3
= x

4) 0 48 28 26 4 2
2 3 4
= + + x x x x
Solucin.
Las posibles races son: 48 24 24 16 16 12 12 8 8 6 6 4 4 3 3 2 2 1 1 , , , , , , , , , , , , , , , , , , y 48 .
Probando con 1 = x :
0 48 20 6 2
48 20 6 2
48 28 26 4 2
1




La primera raz es 1
1
= x
Trabajando ahora con el polinomio reducido:
Probando con 2 = x :
48 0 10 2
0 20 4
48 20 6 2
2



Por lo tanto, no es raz.
Probando con 3 = x :
Pgina del Colegio de Matemticas de la ENP-UNAM Operaciones con fracciones algebraicas y radicales Autor: Dr. Jos Manuel Becerra Espinosa
7

0 16 12 2
48 36 6
48 20 6 2
3


La segunda raz es 3
2
= x
Trabajando ahora con el polinomio reducido:
Probando con 1 = x :
6 10 2
10 2
16 12 2
1
Por lo tanto, no es raz.
Probando con 4 = x :
0 4 2
16 8
16 12 2
4
La tercera raz es 4
3
= x
El polinomio reducido que queda es: 0 4 2 = + x
despejando se tiene la cuarta raz: 2
4
= x


VI.2 OPERACIONES CON FRACCIONES ALGEBRAICAS

Una expresin algebraica racional es el cociente de dos polinomios:

( )
( ) x Q
x P


Las expresiones racionales tienen las mismas propiedades que los nmeros racionales. Como no se
puede dividir por cero, las sustituciones de variables que hacen que el denominador sea cero no son
aceptables.

Ejemplos.

1) En la expresin racional
x
x x 7 5 3
2
+
, x no puede ser 0
2) En la expresin racional
2 + x
x
, x no puede ser 2
3) En la expresin racional
y x
4
, x no puede ser igual a y .

Una expresin racional est en su mnima expresin cuando el numerador y el denominador no tienen
factores comunes diferentes de 1 y 1

Ejemplos.
1) La fraccin
x
x
5
6 +
es su mnima expresin ya que ni 5 ni x son factores de 6 + x
Pgina del Colegio de Matemticas de la ENP-UNAM Operaciones con fracciones algebraicas y radicales Autor: Dr. Jos Manuel Becerra Espinosa
8

2) La fraccin
( )
( ) 2
2 7

x x
x
no es su mnima expresin ya que 2 x es un factor comn del numerador y del
denominador.

Para simplificar expresiones racionales, se procede de forma similar a cuando se simplifican nmeros
racionales, es decir, se factoriza el numerador y el denominador. Los factores se simplifican hasta 1. La
expresin simplificada es igual a la no simplificada excepto para aquellos valores en los que el factor que se
cancele sea igual a cero.

Ejemplos.
Simplificar las siguientes expresiones racionales:

1)
x
x
4
8 4

( )
x
x
x
x
x
x 2
4
2 4
4
8 4
=



2)
2 3
1
2
2
+ +

x x
x

( )( )
( )( ) 2
1
2 1
1 1
2 3
1
2
2
+

=
+ +
+
=
+ +

x
x
x x
x x
x x
x


3)
15 6
2 5

x
x

( )
( )
( ) 3
1
5 2 3
5 2 1
5 2 3
2 5
15 6
2 5
=

x
x
x
x
x
x


4)
4 8 4
14 12 2
2
2
+ +

x x
x x

( )
( )
( )( )
( )( ) ( ) 1 2
7
1 1 4
7 1 2
1 2 4
7 6 2
4 8 4
14 12 2
2
2
2
2
+

=
+ +
+
=
+ +

=
+ +

x
x
x x
x x
x x
x x
x x
x x


5)
( )( )
( ) ( ) y x y x
y xy x y x
12 6
2 12 3
2
2 2 2 2
+
+

( )( )
( ) ( )
( )( )
( ) ( )
( )( )
( )
( )
6
2 3
2 6
2 2 3
2 6
4 3
12 6
2 12 3
2
2 2 2
2
2 2 2 2
y x
y x
y x y x
y x y x
y x y x
y x y x
y xy x y x
=
+
+
=
+

=
+
+


2
2y x
=
6)
x
x x 2
2


En esta expresin racional x no puede ser 0 , y como es el factor que se cancela entonces se cumple que:
( )
2
2 2
2
=

x
x
x x
x
x x
porque 0 x .
Pgina del Colegio de Matemticas de la ENP-UNAM Operaciones con fracciones algebraicas y radicales Autor: Dr. Jos Manuel Becerra Espinosa
9

Para sumar fracciones se efecta el mismo procedimiento que se emplea cuando se suman nmeros
racionales. En general:

Se reducen las fracciones lo ms posible.
Se descomponen los denominadores
Se halla el mnimo comn mltiplo (MCM) de los denominadores, obteniendo as el denominador
comn.
Para hallar el numerador resultante, se divide el MCM por el denominador y se multiplica el cociente
obtenido por el numerador correspondiente, esto convierte al numerador en un polinomio que debe
descomponerse en factores para finalmente simplificar.

Ejemplos.
Efectuar las operaciones algebraicas siguientes:

1)
6
2 3
4
2 +
+
x x

Solucin.
Se obtiene el MCM de los denominadores: 12 :
( ) ( )
12
2 9
12
4 6 6 3
12
2 3 2 2 3
=
+ +
=
+ +
=
x x x x x

12
2 9
6
2 3
4
2
=
+
+

x x x


2)
30
4
15
2
12
x y y x y x
+
+
+


Solucin.
Se obtiene el MCM de los denominadores: 60 :
( ) ( ) ( )
60
4 2 2 4 5 x y y x y x + + +

reduciendo:
60
5
60
8 2 4 8 5 5 y x x y y x y x +
=
+ + +

60
5
30
4
15
2
12
y x x y y x y x +
=

+
+
+



3)
9
12
3
5
3
2
2

+
+ a
a
a
a
a
a

Solucin.
Se descompone el tercer denominador en sus factores:
( )( ) 3 3
12
3
5
3
2
+
+

+
+
=
a a
a
a
a
a
a

se obtiene el MCM de los denominadores: ( )( ) 3 3 + a a :
( ) ( )
( )( ) 3 3
12 5 3 2 3
+
+ + +
=
a a
a a a a a

eliminando parntesis:
( )( ) ( )( ) 3 3
21 7
3 3
12 15 5 6 2
2 2 2
+
+
=
+
+ + +
=
a a
a a
a a
a a a a a

Pgina del Colegio de Matemticas de la ENP-UNAM Operaciones con fracciones algebraicas y radicales Autor: Dr. Jos Manuel Becerra Espinosa
10

factorizando:
( )
( )( ) 3
7
3 3
3 7

=
+
+
=
a
a
a a
a a

3
7
9
12
3
5
3
2
2

+
+

a
a
a
a
a
a
a
a


4)
9 6
3
9
3
4
2
2 2 2
+ +
+
+

+
x x
x
x
x
x
x

Solucin.
Se descomponen los denominadores en sus factores:
( )( ) ( )( ) ( )
2
3
3
3 3
3
2 2
2
+
+
+
+

+
+
+
=
x
x
x x
x
x x
x

reduciendo:
3
1
3
1
2
1
+
+
+
+

=
x x x

se obtiene el MCM de los denominadores: ( )( ) 3 2 + x x :
( ) ( ) ( )
( )( ) 3 2
2 2 3
+
+ + +
x x
x x x

eliminando parntesis:
( )( ) 3 2
1 3
+

x x
x

( )( ) 3 2
1 3
9 6
3
9
3
4
2
2 2 2
+

=
+ +
+
+

x x
x
x x
x
x
x
x
x


5)
2 3
1
6
3
10 7
5
2 2 2
+ +
+
+

+
+ +
+
a a
a
a a
a
a a
a

Solucin.
Se descomponen los denominadores en sus factores:
( )( ) ( )( ) ( )( ) 2 1
1
2 3
3
2 5
5
+ +
+
+
+

+
+ +
+
=
a a
a
a a
a
a a
a

reduciendo:
2
1
2
1
2
1
+
+
+
+
+
=
a a a 2
3
+
=
a

2
3
2 3
1
6
3
10 7
5
2 2 2
+
=
+ +
+
+

+
+ +
+

a a a
a
a a
a
a a
a


6)
1
1
2
2
2

+ x
x
x x
x

Solucin.
Se descomponen los denominadores en sus factores:
( ) ( )( ) 1 1
1
1
2
+

+
=
x x
x
x x
x

se obtiene el MCM de los denominadores: ( )( ) 1 1 + x x x :
Pgina del Colegio de Matemticas de la ENP-UNAM Operaciones con fracciones algebraicas y radicales Autor: Dr. Jos Manuel Becerra Espinosa
11

( ) ( )
( )( ) 1 1
1 1
2
+

=
x x x
x x x x

eliminando los parntesis y ordenando:
( )( ) ( )( ) ( )( ) 1 1
2
1 1
2
1 1
2 3 3 2 3 2
+
+
=
+
+
=
+
+
=
x x x
x x x
x x x
x x x
x x x
x x x x

factorizando el numerador y simplificando:
( )
( )( )
( )( )
( )( ) 1
2
1 1
1 2
1 1
2
2
+
+
=
+
+
=
+
+
=
x
x
x x x
x x x
x x x
x x x

1
2
1
1
2
2
2
+
+
=

x
x
x
x
x x
x


7)
y x
y x
y x
y x
6 6 2 2
2 2

+

Solucin.
Se descomponen los denominadores en sus factores:
( ) ( ) y x
y x
y x
y x

=
6 2
2 2

se obtiene el MCM de los denominadores: ( ) y x 6 :
( ) ( )
( ) y x
y x y x

+
=
6
3
2 2

factorizando el numerador y simplificando:
( ) ( )( )
( )
( )( )
( ) 6
3
6
3
6
3 y x
y x
y x y x
y x
y x y x y x + +
=

+ +
=

+ +
=
6
3
6 6 2 2
2 2
y x
y x
y x
y x
y x + +
=

+


Para multiplicar expresiones racionales se procede de forma similar que con los nmeros racionales.

Ejemplos.
Multiplicar las siguientes expresiones algebraicas:

1)

+

5
2
4 4
10 3
2
2
x
x
x x
x x

Solucin.
Se descompone la fraccin en sus factores:
( )( )
( )( ) 5
2
2 2
2 5
5
2
4 4
10 3
2
2


+
=

+

x
x
x x
x x
x
x
x x
x x

simplificando:
2
2

+
=
x
x

2
2
5
2
4 4
10 3
2
2

+
=

x
x
x
x
x x
x x

Pgina del Colegio de Matemticas de la ENP-UNAM Operaciones con fracciones algebraicas y radicales Autor: Dr. Jos Manuel Becerra Espinosa
12

2) ( ) 4 4
6
12 10 2
2
2
2
+ +

+
y y
y y
y y

Solucin.
Se descompone la fraccin en sus factores:
( )
( )( )
( )
( )( )
( )( )
( ) 4 4
2 3
2 3 2
4 4
2 3
6 5 2
2 2
2
+ +
+

= + +
+
+
= y y
y y
y y
y y
y y
y y

factorizando el trinomio:
( )( )
( )( )
( )( ) 2 2
2 3
2 3 2
+ +
+

y y
y y
y y

simplificando:
( )( ) 2 2 2 + = y y
( ) ( )( ) 2 2 2 4 4
6
12 10 2
2
2
2
+ = + +

+
y y y y
y y
y y


3)
( )

+
+ +
y x
y x
y x
y xy x
2 2
6 12 6
2 2
2
2 2

Solucin.
Tomando como factor comn al 6 en el numerador de la primera fraccin y al 2 en el denominador de la
segunda:
( )
( ) ( )

+
+ +
=
y x
y x
y x
y xy x
2
2 6
2 2
2
2 2

factorizando:
( )
( )
( )( )
( )

+
+

+
+
=
y x
y x y x
y x
y x
2
6
2
2

simplificando:
( ) y x = 3
( )
( ) y x
y x
y x
y x
y xy x
=

+
+ +
3
2 2
6 12 6
2 2
2
2 2


4)

+
+

+
+ +

5
14 9
56 22 2
25 10
10 3
8 32
2
2
2
2
a
a a
a a
a a
a a
a

Solucin.
Tomando como factor comn al 8 en el numerador de la primera fraccin y al 2 en el denominador de
la segunda:
( )
( )

+
+

+
+ +

+

=
5
14 9
28 11 2
25 10
10 3
4 8
2
2
2
2
a
a a
a a
a a
a a
a

factorizando:
( )
( )( )
( )( )
( )( )
( )( )


+ +

+

=
5
7 2
4 7 2
5 5
2 5
4 8
a
a a
a a
a a
a a
a

simplificando:
Pgina del Colegio de Matemticas de la ENP-UNAM Operaciones con fracciones algebraicas y radicales Autor: Dr. Jos Manuel Becerra Espinosa
13

4
2
8
=

=
4
5
14 9
56 22 2
25 10
10 3
8 32
2
2
2
2
=

+
+

+
+ +

a
a a
a a
a a
a a
a


Para dividir expresiones racionales se procede de la misma forma que se efecta con los nmeros
racionales. Para dividir expresiones racionales, se multiplica la primera expresin por el recproco del
divisor.

Ejemplos.
Dividir las siguientes expresiones algebraicas:

1)
6
5
3
15
2
4
x
x

Solucin.
Simplificando:
( )
( )
2
2
4
2
4
6
15
90
5 3
6 15
x
x
x
x
x
= =
2
2
4
6
6
5
3
15
x
x
x
=

2)
1 2
1
1
3 3
2
2
+
+

+
x x
x
x
x

Solucin.
Factorizando las fracciones al mximo:
( )
( )( )
( )( ) 1 1
1
1 1
1 3

+
+
+
x x
x
x x
x

simplificando:
( )( )( )
( )( )( )
( )
1
1 3
1 1 1
1 1 1 3
+

=
+ +
+
=
x
x
x x x
x x x

( )
1
1 3
1 2
1
1
3 3
2
2
+

=
+
+

x
x
x x
x
x
x


Pgina del Colegio de Matemticas de la ENP-UNAM Operaciones con fracciones algebraicas y radicales Autor: Dr. Jos Manuel Becerra Espinosa
14

3)
1
6 4
15 15
30 20
2 3
2
+

x
x
x x
x x

Solucin.
Factorizando las fracciones al mximo:
( )
( )
( )
1
3 2 2
1 15
3 2 10
2
+

=
x
x
x x
x x

simplificando:
( )( )
( ) ( ) x x
x
x x x
x x x
3
1
30
10
3 2 2 1 15
1 3 2 10
2 2
= =
+
+
=

4)
x a
a
x a
a
x a
a
x a
a
+
+

+ 2 2

Solucin.
El MCM de x a + y de x a 2 2 + es: x a 2 2 + , por su parte, el MCM de x a y x a + es: ( )( ) x a x a + ,
por lo que escribiendo la expresin como el cociente de dos fracciones se tiene:
( ) ( )
( )( ) x a x a
x a a a x a
x a
a a
+
+ +
+

=
2 2
2

reduciendo:
( )( ) ( )( ) x a x a
a
x a
a
x a x a
ax a ax a
x a
a
+
+
=
+
+ +
+
=
2 2 2
2
2 2 2 2

factorizando:
( )
( )( )
( )( )
( )
2 2
2 2 2
2
a x a
x a x a a
x a x a
a
x a
a
+
+
=
+
+
=
simplificando:
( )
a
x a
4

=
a
x a
x a
a
x a
a
x a
a
x a
a
4
2 2

=
+
+

+



Pgina del Colegio de Matemticas de la ENP-UNAM Operaciones con fracciones algebraicas y radicales Autor: Dr. Jos Manuel Becerra Espinosa
15

VI.3 OPERACIONES CON RADICALES

Un radical es cualquier raz indicada de una expresin. La radicacin es la operacin inversa de la
potenciacin y se representa por el smbolo
n
, donde n es el ndice del radical y dentro se ubica una
expresin denominada subradical.

Para resolver una raz, se busca una cantidad que elevada a un exponente igual al ndice del radical sea
igual al subradical.

El radical puede ser racional si la raz indicada es exacta o irracional si no lo es.

Ejemplos.
1) El subradical de la expresin 3 5 + x es 3 5 + x
2)
2
16x es un radical racional porque su resultado, x 4 , es exacto.
3)
3 4
17x es un radical irracional porque su resultado no es exacto.
5)
4
4 6 d c es un radical de cuarto grado

En los radicales de segundo grado se omite su ndice, esto es:
2
a a = .

Si b a
n
= , a es una raz ensima de b .

Ejemplos

1) Si 9 3
2
= entonces 3 es una raz cuadrada de 9
2) Si 625 5
4
= entonces 5 es una raz cuarta de 625

Si n es par, 0
n
a , por lo que un nmero negativo no puede tener raz ensima .

Ejemplos

1) Si 16 no tiene raz cuadrada en R.
2) Si
6
64 no tiene raz sexta en R.

Si n es par y
n
a b = , tambin ( )
n
a b = , as que b tiene dos races ensimas, a y a .

Ejemplos

1) Como 25 5
2
= y ( ) 25 5
2
= , 5 y 5 son races cuadradas de 25.
2) Como 81 3
4
= y ( ) 81 3
4
= , 3 y 3 son races cuartas de 25.

Si n es impar, todo nmero real tiene exactamente una raz ensima.

Ejemplos

1) 6 216
3
= .
2) 2 32
5
=
Pgina del Colegio de Matemticas de la ENP-UNAM Operaciones con fracciones algebraicas y radicales Autor: Dr. Jos Manuel Becerra Espinosa
16

Si 0 b , hay una nica raz ensima no negativa de b representada por
n
b

Ejemplo.
Si
2
7 49 = , entonces 7 es una raz cuadrada de 49 y como ( )
2
7 49 = , 7 es otra raz cuadrada de
49. Pero 49 denota exclusivamente a la raz no negativa de 49.

Si n , m , x 0 N, a ley de exponentes fraccionarios establece que:

n m
n
m
x x =

Esto es, cualquier expresin elevada a un exponente fraccionario es igual a una raz cuyo ndice es el
denominador y el subradical es la misma expresin elevada a la potencia que tiene el numerador.

En el caso particular, si n m= , se tiene que:
n n
x x =

Los radicales cumplen con las siguientes propiedades:

1) El producto de dos radicales de un mismo ndice es igual a la raz del producto de los subradicales.
Esto es:
n n n
b a b a = si > > n , b , a 0 0 N.
2) El cociente de dos radicales de un mismo ndice es igual a la raz del cociente de los subradicales.
Esto es:
n
n
n
b
a
b
a
= si > > n , b , a 0 0 N.
3) Un radical de ndice n elevado a una potencia m equivale a una raz de ndice n y de subradical
elevado a la potencia m. Esto es: ( )
n m
m
n
a a = si > n , m , a 0 N.
4) La raz de ndice m de un radical de ndice n es equivalente a una raz de ndice n de un radical de
ndice m y es igual a una raz de ndice n m . Esto es:
n m n m m n
a a a

= = si > n , m , a 0 N.

Es importante notar que la suma algebraica de dos radicales de cualquier ndice no es igual a la raz de la
suma algebraica de los subradicales. Es decir:
n n n
b a b a

De acuerdo con la ley de exponentes fraccionarios y de las propiedades de los radicales, el objetivo de
simplificar un radical es expresarlo en su forma ms simple. Es decir, un radical est simplificado cuando:

No se puede extraer ningn factor del radicando (es el menor posible).
No puede reducirse su ndice (es el menor posible).
El radicando no es una fraccin.
No hay radicales en el denominador de una fraccin.

VI.3.1 SIMPLIFICACIN DE RADICALES A TRAVS DE LA EXTRACCIN DE FACTORES DEL
SUBRADICAL

Un radical se puede simplificar cuando contiene factores cuyos exponentes son divisibles por el ndice y
se procede de la siguiente manera:

La parte numrica del subradical se descompone en factores de tal forma que sean potencias con
exponentes mltiplos del ndice de la raz, a fin de poder extraer del radical.
La parte literal del subradical se descompone de tal manera que se exprese la mayor parte posible
con exponentes mltiplos del ndice de la raz.
Pgina del Colegio de Matemticas de la ENP-UNAM Operaciones con fracciones algebraicas y radicales Autor: Dr. Jos Manuel Becerra Espinosa
17

Ejemplos.

1) a a a a a a a 2 3 2 3 2 9 18
2 4 2 4 5
= = =
2)
4 3 4 3 4 4 4 3 4 4 7
3 3 3 3 3 81 243 k k k k k k k = = =
3)
3
2 2
3
6 2 3 3
3
6 2 3
3
7 5
4 5 4 5 4 125 500 y x xy y y x x y y x x y x = = =
4)
5 4 3 5 4 5 5 3 5 5 5 9 6 8 5 9 6 8
2 2 2 2 2 32 64 wz v vwz z z w w v v z w v z w v = = =
5) ( ) ( ) ab a a ab a a ab a a b a a 2 2 2 2 2 4 8 4
2 2 2 2 2 2 3 4
= = =
6) ( ) ( ) ( ) n m a n m a n mn m a an amn am + = + = + + = + + 2 2 2 2 2 4 2
2 2 2 2 2

7)
3
2
3
2
2
3
2 3 3
3 6
3
2 3
3
3
5
4
2 2
9
2 2
3
2 2
3
2 8
729
16
729
b
a
b
a
b
a
b
a
b b
a a
b b
a a
b
a
= =


=


=
8) abc c ab c c b b a a c c b b a a c b a 11 2 11 2 11 4 44
4 3 8 6 2 2 8 6 2 9 7 3
= = =

VI.3.2 INCLUSIN DE UN FACTOR EN UN SUBRADICAL

En este caso se eleva la expresin por introducir a la potencia que indique el ndice del radical, se efecta
el producto de subradicales y el resultado se expresa con el mismo ndice.

Ejemplos.

1) 80 5 16 5 4 5 4
2
= = =
2) ( )
3 2 2
12 3 4 3 2 3 2 a a a a a a a = = =
3) ( )
2 2 2
25 25 5 5 = = =
4) 2 18
9
1
18
3
1
18
3
1
2
= =

=
5) ( ) ( )
( )
( ) xy x x y x
y x
x y x
y x
x
y x
y x
x
y x + = + =
+
+
=
+
+ =
+
+
2
2
2

6) ( )
3 5 3 2 3 3 3 2
3
3
3 2
128 2 64 2 4 2 4 w w w w w w w = = =
7) ( )
4 3 5 4 3 4 4 4 3
4
4
4 3
96 6 16 6 2 6 2 b a ab a ab a ab a = = =
8) ( )
5 13
5
4
3
5 5 10
5
4
3
5
5
2
5
4
3
2
9
27
243
27
3
27
3 m k
m
k
m k
m
k
m k
m
k
m k = = =

VI.3.3 EXPRESAR UN RADICAL COMO UNO DE NDICE MENOR

Otra forma de simplificacin de un radical consiste en transformarlo a uno equivalente que posea un
ndice menor. Para ello, se expresa cada uno de los factores del subradical en su forma de exponente
fraccionario, se simplifican las fracciones y se vuelve a transformar a radical.

Ejemplos.

1) x x x x = = =
2
1
4
2
4 2

Pgina del Colegio de Matemticas de la ENP-UNAM Operaciones con fracciones algebraicas y radicales Autor: Dr. Jos Manuel Becerra Espinosa
18

2) ( )
4 3
4
3
8
6
8
1
6 8 6
k k k k k = = = =
3) ( )
4 4
1
12
3
12
1
3 12
6 6 6 6 216 = = = =
4) ( )
3 3
1
3
1
6
1
2 2 6 2 2
mn n m n m n m = = =
5)
2 2
2 2
2 32
2
1
2
1
2
1
10
5
10
5
10
1
5
5
10
5
a a a a a a
=

= = =

=
6) ( ) eh h e h e h e h e 5 5 5 5 25
2
1
2
1
2
1
4
2
4
2
4
2
4
1
2 2 2 4 2 2
= = = =
7) ( )
3
2 2
3
2
3
2
3
2
9
6
9
6
9
6
9
1
6 6 6
9
6 6
4 2 2 2 64 = = = =
8) ( )
4
2
4
2
4
1
4
2
8
4
8
2
8
4
8
1
4 2 4
8
4 2
4 2 2 2 16 xy y x y x y x y x = = = =

VI.3.4 OPERACIONES CON RADICALES DEL MSMO NDICE.

Radicales semejantes son aquellos que tienen igual radicando y el mismo ndice, es decir, slo difieren
por el coeficiente.

Ejemplos.

1) x 4 y x 9 son radicales semejantes
2)
3 2
6 ab y
3 2
4
5
ab son radicales semejantes
3) x 8 y
3
7 x no son radicales semejantes

Para sumar o restar radicales se simplifican a su forma ms elemental y se reducen los radicales semejantes.

Ejemplos.

1) 5 6 5 2 5 4 5 2 5 4 5 4 5 16 20 80
2 2
= + = + = + = +
2) 5 2 3 3 5 3 5 2 3 3 5 3 5 4 3 9 5 9 20 27 45
2 2 2
= = =
3 3 5 =
3) 3 25 2 7 9 3 81 7 25 75 2 63 243 175 + = +
3 10 7 3 3 9 7 5 3 5 2 7 3 3 9 7 5 3 5 2 7 3 3 9 7 5
2 2 2 2
+ = + = + =
3 7 2 =
4) 0 3 7 3 2 3 5 3 7 3 2 3 5 3 49 3 4 3 25 147 12 75
2 2 2
= + = + = + = +
5) 2 16 25 5 5 16 3 4 5 16 4 2 9 25 7 800 5 80 3 320 4 450 7 + = +
2 4 5 5 5 4 3 5 2 4 4 2 3 5 7 2 4 5 5 5 4 3 2 5 4 4 2 3 5 7
2 2 2 2 2 2 2
+ = + =
5 20 2 5 2 100 5 12 5 32 2 105 = + =
6) 2 6 2 5 2 4 2 36 2 25 2 16 72 50 32
2 2 2
+ = + = + 2 6 2 5 2 4 + =
2 3 =
Pgina del Colegio de Matemticas de la ENP-UNAM Operaciones con fracciones algebraicas y radicales Autor: Dr. Jos Manuel Becerra Espinosa
19

7) 2 10 2 5 2 9 2 100 2 25 2 81 200 50 162
2 2 2
+ = + = +
2 4 2 10 2 5 2 9 = + =
8) 3 2 3 2 3 5 3 4 9 3 4 3 3 9 5 3 16 9 12 3 27 5 48 9
2 2 2
+ = + = +
3 27 3 6 3 15 3 36 3 2 3 3 3 5 3 4 9 = + = + =

Para efectuar la multiplicacin de radicales se multiplican respectivamente los coeficientes y los
subradicales, ubicando este ltimo producto bajo el signo de radical y se simplifica.

Ejemplos.

1) 2 3 2 3 18 6 3
2
= = =
2) 7 30 7 3 10 7 3 10 63 10 3 2 21 5
2
= = = =
3)
3 3 3 3 3 3 2
18 3 6 27
4
24
3 8 9
4
3
b a b a b a ab a = = =
4)
3 3 3 3 3
3 3 3 3
4 30 4 3 5 2 4 3 5 2 500 13 2 20 4 15
6
1
45 3 = = = = ,

Para dividir dos radicales, se dividen respectivamente los coeficientes y los subradicales, ubicando este
ltimo cociente bajo el signo de radical y se simplifica.

Ejemplos.
1) 2 2
3 2
6 4
=
2) 3
5
1
10
3 2
=
a
a

3) k k k k k
k
k
2
3
4
6
2
4
3
2
4
3
8
4
3
2 4
16 3
3 3 3 3 3
3 2
3 5
= = = = =
4) y y y y y y y
xy
xy
2 3
6
4
3
6
4
9
6
4
4
4
3
36
2
1
2 2 3
5
8
= = = =

VI.3.5 SIMPLIFICACIN DE RADICALES DE NDICE DIFERENTE

Los radicales no semejantes no se pueden reducir, por lo que la suma y la resta no son posibles.

Para multiplicar dos radicales de diferente ndice:

Se halla el MCM de los ndices.
El MCM se divide entre cada ndice de la raz y cada radicando se eleva a este resultado.
Se resuelven los radicandos como potencia de otra potencia, es decir multiplicando los exponentes.
Se multiplican los radicandos como potencias de la misma base, es decir sumando los exponentes.
El radicando se descompone en factores procurando que sean potencias con exponentes mltiplos
del ndice de la raz, a fin de poder extraer del radical aquella parte que lo permita.

Pgina del Colegio de Matemticas de la ENP-UNAM Operaciones con fracciones algebraicas y radicales Autor: Dr. Jos Manuel Becerra Espinosa
20

Ejemplos.
1)
3 2
2x x
el ndice comn es 6 , por lo tanto:
( )
6 6 6 6 7 6 4 3 6
2
2 6 3 3 2
4 4 4 4 2 2 x x x x x x x x x x x = = = = =
2)
4 3
8 4 2 3 a ab
el ndice comn es 8, por lo tanto:
( ) ( ) ( )
8 6 6 4 4 4 8 6
2
3 4 4 4 8
2
3
8
4
4 3
2 2 12 2 2 12 8 4 2 3 8 4 2 3 a b a a b a a ab a ab = = =
8 4 2 8 2 8 8 4 10 10
2 2 12 2 12 b a a b a = =
4 2 8 4 2 2
2 24 2 2 12 ab a b a a = =
3)
4 3 3 2 2
3 2 b a b a
el ndice comn es 12 , por lo tanto:
( ) ( )
12 11 5 12 3 12 11 17 3 12 3 9 3 8 8 12
3
3 12
4
2 2 4 3 3 2 2
3 2 3 2 3 2 3 2 3 2 b a a b a b a b a b a b a b a b a = = = =
12 11 5
27 2 b a a =
4)
5 4 3 2
16
4
3
4
3
2
n m m
el ndice comn es 15, por lo tanto:
( ) ( ) ( ) ( )
15 3 12
3
4 10
5
2 15 3 12 3 10 5 15
3
4 15
5
2 5 4 3 2
2 2
2
1
16 4
2
1
16 4
12
6
16
4
3
4
3
2
n m m n m m n m m n m m = = =
15 3 12 12 10 10
2 2
2
1
n m m =
15 3 7 15 3 7 7 15 3 7 15 7 15 15 3 22 22
128 2 2
2
1
2 2
2
1
2
2
1
n m m n m m n m m n m = = = =

Para dividir dos radicales de diferente ndice:

Se halla el MCM de los ndices.
El MCM se divide entre cada ndice de la raz y cada radicando se eleva a este resultado.
Se resuelven los radicandos como potencia de otra potencia, es decir multiplicando los exponentes.
Se dividen los radicandos como potencias de la misma base, es decir restando los exponentes.
El radicando se descompone en factores procurando que sean potencias con exponentes mltiplos
del ndice de la raz, a fin de poder extraer del radical aquella parte que lo permita.

Ejemplos.
1)
3
2
3
3
x
x

el ndice comn es 6 , por lo tanto:
( )
( )
6 4
6
2
6
6
2
6
3
2
3
2
3
9
27
3
3
3
3
x
x
x
x
x
x
x
= = =
2)
4 2
3 3
4
8
a
b a

el ndice comn es 12 , por lo tanto:
Pgina del Colegio de Matemticas de la ENP-UNAM Operaciones con fracciones algebraicas y radicales Autor: Dr. Jos Manuel Becerra Espinosa
21

( )
( )
6 2 3 12 4 6 2 12 4 6
12
6
4 12
12
6 3
4 12 4
12
3
2
12
4
3
4 2
3 3
8 8 64
64
096 4
4
8
4
8
4
8
b a b a b a
a
b a ,
a
b a
a
b a
a
b a
= = = = = =
3)
5 2
3 3 4
5
mn
n m

el ndice comn es 15, por lo tanto:
( )
( )
15
9 2
15
9 2 15
15
9 17
15
6 3
15 20
15
3
2
15
5
3 4
5 2
3 3 4
125 3 125 3 125 3
125 3 5 5
n m , m n m m , n m ,
n m
n m ,
mn
n m
mn
n m
= = = = =
4)
4
3 2 2
6
5 4 3
3
18
z y x
z y x

el ndice comn es 12 , por lo tanto:
( )
( )
12
2
12
9 6 6
10 8 6
12
3
3 2 2
12
2
5 4 3
4
3 2 2
6
5 4 3
12
27
324
3
18
3
18
z y
z y x
z y x
z y x
z y x
z y x
z y x
= = =

para extraer la raz de un radical, se multiplican los ndices y se simplifica.

Ejemplos.
1)
3 6 2 3 2
a a a = =
2) 2 2 8 8
6 3 6 3
= = =
3) ( )
4 8
2
8 2 4 2
5 5 25 25 a a a a = = =
4) ( )
3 2 15
5
2 15 10 5 3 10
x x x x = = =

VI.3.6 RACIONALIZACIN DE RADICALES

Racionalizar consiste en eliminar los radicales del denominador de una fraccin. Para lograr esto, se
multiplican las dos componentes del cociente por una expresin que contenga el radical por eliminar y
que cumpla que al multiplicarse, el denominador resulte una expresin racional.

Ejemplos.
Racionalizar las siguientes fracciones:

1)
3
1

multiplicando el numerador y el denominador por 3 :
3
3
3
3
3
1
=
2)
5 4
3

multiplicando el numerador y el denominador por 5 :
Pgina del Colegio de Matemticas de la ENP-UNAM Operaciones con fracciones algebraicas y radicales Autor: Dr. Jos Manuel Becerra Espinosa
22

( ) 20
5 3
5 4
5 3
5
5
5 4
3
= =
3)
4
9
3
a

multiplicando el numerador y el denominador por ( )
4
3
9a :
( )
( )
( )
a
a
a
a
a
a
a
a
a 3
729
9
729 3
9
9 3
9
9
9
3
4 3 4 3 4
3
4
3
4
3
4
= = =
4)
3
3 5
6
x

multiplicando el numerador y el denominador por ( )
3
2
3x :
( )
( )
( )
( ) x
x
x
x
x
x
x
x
x 5
9 2
15
9 6
3 5
3 6
3
3
3 5
6
3 2 3 2 3
2
3
2
3
2
3
= = =

Ejemplo.
Efectuar la operacin
4
3
2
1
3
1
+ y racionalizar el resultado.

Solucin.
3 2
3
2
2
3
1
3
3
2
3
2
2
2
1
3
1
2
3
2
1
3
1
4
3
2
1
3
1
4
3
2
1
3
1
+ = + = + = + = +
2
2
6
3 5
2
2
3
3
3 2
5
2
2
3 2
5
= = =

Cuando se quiere racionalizar una fraccin cuyo denominador sea un binomio que posea radicales de
segundo grado, se multiplican las dos componentes del cociente por el binomio conjugado del
denominador y se simplifica.

Ejemplos.
Racionalizar las siguientes fracciones:

1)
2 1
2 3
+


multiplicando el numerador y el denominador por 2 1 , que es el binomio conjugado del denominador:
5 2 4
1
2 4 5
2 1
2 2 2 3 3
2 1
2 1
2 1
2 3
=

+
=


2)
3 4
3 2 5

+

multiplicando el numerador y el denominador por 3 4 + , que es el binomio conjugado del denominador:
3 2
13
3 13 26
3 16
6 3 8 3 5 20
3 4
3 4
3 4
3 2 5
+ =
+
=

+ + +
=
+
+

+

Pgina del Colegio de Matemticas de la ENP-UNAM Operaciones con fracciones algebraicas y radicales Autor: Dr. Jos Manuel Becerra Espinosa
23

3)
3 4 2 5
19


multiplicando el numerador y el denominador por 3 4 2 5 + , que es el binomio conjugado del denominador:
2
3 76 2 95
48 50
3 76 2 95
3 16 2 25
3 76 2 95
3 4 2 5
3 4 2 5
3 4 2 5
19 +
=

+
=

+
=
+
+


4)
7 3 3 2
7 3 3 4
+


multiplicando el numerador y el denominador por 7 3 3 2 , que es el binomio conjugado del denominador:
51
21 18 87
63 12
63 21 18 24
7 9 3 4
7 9 21 6 21 12 3 8
7 3 3 2
7 3 3 2
7 3 3 2
7 3 3 4

+
=

+
=


17
29 21 6
51
87 21 18
=

=


VI.4 INTRODUCCIN A LOS NMEROS COMPLEJOS

Existen nmeros llamados complejos que forman un sistema numrico que comparte muchas propiedades
con los nmeros reales. En este sistema es posible encontrar soluciones a ecuaciones como r x =
2
con
r R
+
para los cuales el conjunto de los nmeros reales resulta insuficiente.

Se define como unidad imaginaria i al nmero que elevado al cuadrado es 1 .

Formalmente, el conjunto de los nmeros imaginarios I, se define como:

I { = = b bi x R, } 1 = i

Ejemplos de nmeros imaginarios:

i x 8
1
=
i x
4
5
2
=
i . x 7698 3
3
=
i x 7
4
=

Dado que x x 1 = , entonces la solucin de una raz cuadrada de un nmero real negativo
x siempre est dado por la raz no negativa x i .

Ejemplos.

1) i 4 16 =
2) i 7 49 =
3) i 2 2 8 =
Pgina del Colegio de Matemticas de la ENP-UNAM Operaciones con fracciones algebraicas y radicales Autor: Dr. Jos Manuel Becerra Espinosa
24

Las potencias de la i cumplen lo siguiente:
1 = i
1
2
= i
( ) i i i i i = = = 1
2 3

( ) 1
2 3 4
= = = = i i i i i i
( ) i i i i i = = = 1
4 5

( ) 1
2 5 6
= = = = i i i i i i
De acuerdo con lo anterior, en los nmeros imaginarios no se cumple que ( )
n n
n
n
a a si 0 < a .

Ejemplos.
Efectuar los siguientes productos de nmeros imaginarios:

1) ( ) 15 1 15 15 5 3
2
= = = i i i
2) ( ) i i i i i i 56 56 56 7 4 2
3
= = =
3) ( ) 9 1 9 9
3
15
5
6
2
4
3
4
= = = i i i i i
4) ( ) i i i , i i i i i 42 10 42 10 200 4 10 7 5 2 6
5
= = =
5) ( ) ( ) ( ) 025 2 1 025 2 025 2 25 81 5 3
6 2 4 2 4
, , i , i i i i = = = =

Se denomina nmero complejo a toda expresin de la forma bi a z + = donde b a, son nmeros reales
e i es la unidad imaginaria. El primer trmino del binomio es la parte real del nmero complejo y la
segunda es su parte imaginaria (que es un nmero real multiplicado por la unidad imaginaria).

En trminos generales, el conjunto de los nmeros complejos, denotado por C, en forma binmica puede
expresarse de la siguiente forma:

C { + = = b , a , bi a z R, } 1 = i

Ejemplos de nmeros complejos:

i z 5 2
1
+ =
i z 3 4
2
=
i z
4
7
3
1
3
+ =
i . . z 37 9 29 8
4
=
11
5
+ = z

Si 0 = a , el nmero complejo es un imaginario puro. Si 0 = b el nmero complejo es un nmero real. De
esto, se deduce que los nmeros reales y los nmeros imaginarios son subconjuntos de los nmeros
complejos:



Pgina del Colegio de Matemticas de la ENP-UNAM Operaciones con fracciones algebraicas y radicales Autor: Dr. Jos Manuel Becerra Espinosa
25




Un nmero complejo es igual a cero slo si sus dos partes son iguales a cero. Dos nmeros complejos
son iguales si son iguales sus respectivas partes reales e imaginarias.

Suma de nmeros complejos

Sean bi a z + =
1
y di c z + =
2
dos nmeros complejos, entonces
2 1
z z + se define como:

( ) ( )i d b c a z z + + + = +
2 1


Ejemplos.
Sumar los siguientes nmeros complejos:

1) i z 4 3
1
+ = y i z 5 2
2
+ =
Solucin:
( ) ( ) i i z z 9 5 5 4 2 3
2 1
+ = + + + = +
2) i z 6
2
9
1
+ = y i z 8
2
3
2
=
Solucin:
( ) ( ) i i z z 2 6 8 6
2
3
2
9
2 1
= + +

+ = +
3) i . . z 6 1 9 4
1
= y i . . z 2 2 3 5
2
=
Solucin:
( ) ( ) ( ) ( ) i . . i . . . . z z 8 3 4 0 2 2 6 1 3 5 9 4
2 1
= + + + = +

Resta de nmeros complejos

Sean bi a z + =
1
y di c z + =
2
dos nmeros complejos, entonces
2 1
z z se define como:

( ) ( )i d b c a z z + =
2 1


Ejemplos.
Restar los siguientes nmeros complejos:

1) i z 11 3
1
+ = y i z 7 2
2
+ =
Solucin:
Nmeros
reales
Nmeros
imaginarios
a
bi
Nmeros Complejos = bi a +
Nmeros
reales
Nmeros
imaginarios
a
bi
Nmeros Complejos = bi a + Nmeros Complejos = bi a +
Pgina del Colegio de Matemticas de la ENP-UNAM Operaciones con fracciones algebraicas y radicales Autor: Dr. Jos Manuel Becerra Espinosa
26

( ) ( ) i i z z 4 1 7 11 2 3
2 1
+ = + =
2) i z
3
2
4
5
1
+ = y i z
5
7
3
1
2
=
Solucin:
i z z

=
5
7
3
2
3
1
4
5
2 1

i i
15
31
12
19
15
21 10
12
4 15
+ =
+
+

=
3) i . . z 7 3 2 5
1
= y i . . z 3 3 8 1
2
=
Solucin:
( ) ( ) ( ) ( ) i . i . . . . z z 4 0 7 3 3 7 3 8 1 2 5
2 1
= + =
Producto de nmeros complejos

Sean bi a z + =
1
y di c z + =
2
dos nmeros complejos, entonces
2 1
z z viene dado por:
( ) ( )
2
2 1
bdi bci adi ac di c bi a z z + + + = + + = , pero considerando que 1
2
= i y agrupando las
respectivas partes reales y las imaginarias, se tiene que:

( ) ( )i bc ad bd ac z z + + =
2 1


Ejemplos.
Multiplicar los siguientes nmeros complejos:

1) i z 5 4
1
+ = y i z 3 2
2
+ =
Solucin:
( ) ( ) ( ) ( ) ( ) ( ) ( ) ( ) i i i z z 22 7 10 12 15 8 2 5 3 4 3 5 2 4
2 1
+ = + + = + + =
2) i z 9 1
1
= y i z
4
3
8
2
=
Solucin:
( ) ( ) ( )( ) i z z

= 8 9
4
3
1
4
3
9 8 1
2 1
i

= 72
4
3
4
27
8
i i
4
291
4
5
4
288 3
4
27 32
=


=
3) i . z 5 5 2
1
= y i . z 5 4 10
2
=
Solucin:
( ) ( )( ) ( ) ( ) ( )( ) ( )i . . . . z z 10 5 5 4 5 2 5 4 5 10 5 2
2 1
+ + =
( ) ( ) i . . i . . 75 38 5 47 50 25 11 5 22 25 + = + + =

Complejos conjugados

Dos nmeros complejos se llaman conjugados si tienen iguales sus componentes reales y opuestas sus
componentes imaginarias.

Esto es, dado un nmero complejo bi a z + = , su conjugado denotado como z es:

bi a z = .
Pgina del Colegio de Matemticas de la ENP-UNAM Operaciones con fracciones algebraicas y radicales Autor: Dr. Jos Manuel Becerra Espinosa
27

Ejemplos.

1) i z 4 6
1
=
i z 4 6 1 + =
2) i . . z 3 8 5 1
2
+ =
i . . z 3 8 5 1 2 =
3) i z
2
1
3
7
3
+ =
i z
2
1
3
7
3 =

Cociente de nmeros complejos

Sean bi a z + =
1
y di c z + =
2
dos nmeros complejos. Para obtener
2
1
z
z
basta con multiplicar el
numerador y el denominador por el complejo conjugado del
2
z a fin de que el denominador resultante
sea real:
2 2 2
2
2
1
i d c
bdi bci adi ac
di c
di c
di c
bi a
z
z

+
=

+
+
=
ordenando se tiene:

( ) ( )
2 2
2
1
d c
i ad bc bd ac
z
z
+
+ +
=

Ejemplos.
Dados los siguientes nmeros complejos, obtener el cociente
2
1
z
z
:

1) i z 3 14
1
+ = y i z 4 5
2
+ =
Solucin:
( ) ( )( ) [ ] ( )( ) ( )( ) [ ] ( ) ( )
i
i i i
z
z
=

=
+
+ +
=
+
+ +
= 2
41
41 82
16 25
56 15 12 70
4 5
4 14 5 3 4 3 5 14
2 2
2
1

2) i z 26 18
1
+ = y i z 2 6
2
=
Solucin:
( )( ) ( )( ) [ ] ( ) ( )( ) [ ]
( )
( ) ( )
i
i i i
z
z
3 4
40
120 160
4 36
36 156 52 108
2 6
2 18 6 26 2 26 6 18
2 2
2
1
+ =
+
=
+
+
=
+
+ +
=
3) i z 9 6
1
= y i z = 2
2

Solucin:
( )( ) ( )( ) [ ] ( )( ) ( )( ) [ ]
( ) ( )
( ) ( )
i
i i i
z
z
5
24
5
3
5
24 3
1 4
6 18 9 12
1 2
1 6 2 9 1 9 2 6
2 2
2
1
+ =
+
=
+
+ + +
=
+
+ +
=





Pgina del Colegio de Matemticas de la ENP-UNAM Ecuaciones y desigualdades Autor: Dr. Jos Manuel Becerra Espinosa
1

ECUACIONES Y DESIGUALDADES

UNIDAD VII


VII.1 CONCEPTO DE ECUACIN

Una igualdad es una relacin de equivalencia entre dos expresiones, numricas o literales, que se
cumple para algn, algunos o todos los valores y se representa por el signo =. Cada una de las
expresiones recibe el nombre de miembro. Se llama primer miembro a lo que est a la izquierda del signo
igual y segundo miembro a lo que est a su derecha.

b expresin a expresin =

Las igualdades pueden ser numricas (establecen relaciones entre nmeros) o algebraicas (si contienen
literales). Pueden ser ciertas (si se cumplen) o falsas (si no siempre se cumplen).

Ejemplos

1) La igualdad 2 8 10 + = es numrica y cierta
2) La igualdad ( )
2 2 2
2 b ab a b a + + = + es algebraica y cierta para cualesquiera valores de a y b .
3) La igualdad x x = 14 3 es algebraica y cierta para 7 = x , pero es falsa para cualquier otro valor de x .

Por lo tanto, las igualdades pueden ser de dos tipos:

Identidades. Son igualdades que se verifican siempre, ya sean numricas o algebraicas.
Ecuaciones. Son igualdades que se verifican para algunos valores determinados de las literales
desconocidas llamadas incgnitas.

Ejemplos.

1)
2
1
6
3
= es una identidad numrica
2) ( )( ) b a b a b a + =
2 2
es una identidad algebraica
3) 10 2 4 = x es una ecuacin que se verifica slo para 3 = x
4) 4
2
= x es una ecuacin que se verifica slo para 2 = x y 2 = x .

En una ecuacin, las cantidades desconocidas o incgnitas generalmente se designan por letras
minsculas de la parte final del alfabeto. Por su parte, las cantidades conocidas o coeficientes
normalmente se denotan por las letras minsculas iniciales del alfabeto
1
.

Las ecuaciones de una sola variable son aquellas que tienen una sola incgnita, normalmente la x . Por
ejemplo: 4 1
2
+ = + x x .

Las ecuaciones en dos o ms variables poseen ms de una cantidad desconocida. Por ejemplo, en la
ecuacin 0 8 5 2 = + y x , las incgnitas son x y y .

Las ecuaciones se clasifican de acuerdo con el exponente mayor que posea la incgnita.

1
Esta nomenclatura la introdujo el matemtico Ren Descartes en 1637.
Pgina del Colegio de Matemticas de la ENP-UNAM Ecuaciones y desigualdades Autor: Dr. Jos Manuel Becerra Espinosa
2

Ejemplos.

7 35 6 = x es una ecuacin de primer grado.
7 5 18 6 3
2
+ = + x x x es una ecuacin de segundo grado.
2 3 2 3
6 8 2 5 2 7 x x x y x x = + es una ecuacin de tercer grado.

Una ecuacin es entera, si todos sus trminos son enteros o es racional si alguno de sus trminos est
expresado como fraccin.

Ejemplos.

1) x y x 6 5 2 4 = es una ecuacin en dos variables, de primer grado y entera
2)
2
1
5
4
3
2
= x x es una ecuacin en una variable, de segundo grado y racional
3) 64
4 16
2 2
=
y x
es una ecuacin en dos variables, de segundo grado y racional
4) 11 8 2 7 = z y x es una ecuacin en tres variables, de primer grado y entera

Resolver una ecuacin es hallar el conjunto solucin. Se conocen como races o soluciones de la
ecuacin a los valores de las incgnitas que satisfacen la igualdad
2
.

Ejemplos.
1) En la ecuacin 1 7 4 + = + x x
El resultado es 2 = x , porque si se sustituye el valor en ambos miembros, cumple la igualdad:
( ) 1 2 7 2 4 + = +
1 7 8 = +
1 1

2) En la ecuacin 0 12
2
= + x x
Los resultados son 4
1
= x y 3
2
= x , porque si se sustituyen los valores, cumplen la igualdad:
Sustituyendo 4
1
= x :
( ) ( ) 0 12 4 4
2
= +
0 12 4 16 =
0 0
Sustituyendo 3
2
= x :
0 12 3 3
2
= +
0 12 3 9 = +
0 0

Dos ecuaciones son equivalentes si tienen el mismo conjunto solucin.

Ejemplo.
Las ecuaciones 5 3 2 = x y 8 2 = x son equivalentes porque su solucin es 4 = x


2
En situaciones reales la solucin de la ecuacin debe tener sentido en el contexto en que se trabaja. Esto significa que no basta
con resolver una ecuacin sino que tambin hay que analizar la pertinencia de la solucin, esto es si el resultado pertenece al
conjunto definido por la situacin particular a la que se refiere la ecuacin. En este tema se abordarn soluciones de ecuaciones
que slo existan en los nmeros reales.
Pgina del Colegio de Matemticas de la ENP-UNAM Ecuaciones y desigualdades Autor: Dr. Jos Manuel Becerra Espinosa
3

Para resolver una ecuacin, se transforma sta en una ecuacin equivalente con la variable despejada.
Esta transformacin se logra aplicando las siguientes propiedades:

Si se suma una misma cantidad a cada lado de la ecuacin dada, la igualdad no se altera.
Si se resta una misma cantidad a cada miembro de la ecuacin dada, la igualdad no se altera.
Si se multiplica o se divide a ambos lados de la ecuacin por cualquier cantidad diferente de cero, la
igualdad no se altera.

Ejemplos.
1) Sumando la misma cantidad, 7 a cada lado de la ecuacin 8 6 7 3 = + x se tiene:
7 8 7 6 7 3 + = + + x , que reducida es: 15 6 3 = + x . Ntese como 7 es el simtrico de 7

2) Restando la misma cantidad, 6 a cada lado de la ecuacin 15 6 3 = + x se tiene:
6 15 6 6 3 = + x , que reducida es: 9 3 = x . Ntese como 6 es el simtrico de 6

3) Multiplicando la misma cantidad,
3
1
a cada lado de la ecuacin 9 3 = x se tiene:
( ) ( ) 9
3
1
3
3
1
= x , que reducida es: 3 = x . Ntese como
3
1
es el inverso multiplicativo o recproco de 3


VII.2 ECUACIONES DE PRIMER GRADO EN UNA VARIABLE

Una ecuacin de primer grado es una ecuacin en la cual, despus de simplificarla o reducir sus trminos
semejantes, el mximo exponente de la incgnita es uno.

En trminos generales, una ecuacin de primer grado con una variable es de la forma:

0 = + b ax

donde a y b son coeficientes numricos, 0 a y x es la incgnita.

Si se suma b en ambos miembros de la ecuacin, se tiene: b ax b b b ax = = + 0 , y si se
multiplica por el recproco de a en ambos lados se tiene: ( ) ( ) b
a
ax
a
=
1 1
, entonces la solucin de una
ecuacin de primer grado en su forma general est dada por
a
b
x = .

VII.2.1 ECUACIONES ENTERAS

Para resolver una ecuacin de este tipo se debe aplicar la metodologa antes citada. En este caso, se
deben transponer los trminos, esto es traspasarlos de un lado a otro de la ecuacin de manera que
todos los trminos que tengan la incgnita queden en el primer miembro y los trminos independientes en
el otro. Para fines prcticos, cada vez que se transpone un trmino de un miembro a otro, ste cambia de
signo, se reducen trminos semejantes y finalmente, para despejar la incgnita se divide por su
coeficiente.

Ejemplos.
Resolver las siguientes ecuaciones enteras:

1) x x x x x 8 19 3 2 13 2 4 7 6 + + + = +
Se transponen trminos:
Pgina del Colegio de Matemticas de la ENP-UNAM Ecuaciones y desigualdades Autor: Dr. Jos Manuel Becerra Espinosa
4

4 7 19 2 8 3 13 2 6 + + = x x x x x
se reducen los trminos semejantes:
20 20 = x
dividiendo por 20 :
1
20
20
=

= x
Comprobacin:
( ) ( ) 7 2 4 7 6 1 2 4 7 1 6 = + + = +
( ) ( ) ( ) 7 8 19 3 2 13 1 8 19 1 3 2 1 13 = + = + + +
7 7

2) x x x x x 11 12 3 9 6 8 2 7 4 + = +
Transponiendo trminos:
8 7 12 9 11 3 6 2 4 + = + x x x x x
se reducen los trminos semejantes:
22 4 = x
dividiendo por 4 :
2
11
4
22
=

= x
Comprobacin:
|

\
|

|

\
|
+
|

\
|
= +
|

\
|

|

\
|

2
11
11 12
2
11
3 9
2
11
6 8
2
11
2 7
2
11
4 10 8 11 7 22 = + + =
10
2
121
12
2
33
9 33 = +
10 10

3) ( ) ( ) ( ) 1 4 3 1 5 3 2 = + x x x x
Se eliminan los parntesis:
4 4 3 5 5 6 2 + = + + x x x x
despus, se transponen trminos:
5 6 4 4 3 5 2 = + x x x x
Se reducen los trminos semejantes:
7 2 = x
dividiendo por 2 :
2
7
2
7
=

= x
Comprobacin:
2
1
2
25
13
2
5
5
2
13
2 1
2
7
5 3
2
7
2 = =
|

\
|

\
|
=
|

\
|

|

\
|
+
2
1
10
2
21
2
5
4
2
21
1
2
7
4
2
7
3 = =
|

\
|
=
|

\
|

|

\
|

2
1
2
1


4) ( ) ( ) ( ) 0 1 26 16 10 8 9 5 6 3 = + x x x
Se eliminan los parntesis:
Pgina del Colegio de Matemticas de la ENP-UNAM Ecuaciones y desigualdades Autor: Dr. Jos Manuel Becerra Espinosa
5

0 26 26 16 90 72 15 18 = + + + + x x x
despus, se transponen trminos:
26 16 72 15 26 90 18 + + = + + x x x
Se reducen los trminos semejantes:
67 98 = x
dividiendo por 98 :
98
67
= x
Comprobacin:
0
49
403
16
49
513
49
132
98
67
1 26 16
98
67
10 8 9 5
98
67
6 3 = + = |

\
|
+
(

\
|

(

\
|


Una ecuacin de primer grado literal es aquella que contiene otras expresiones literales aparte de la
incgnita, las cuales deben considerarse como valores constantes.

Para resolver ecuaciones literales se efecta el mismo procedimiento aplicado en los ejemplos anteriores.
La variante es que cuando se tengan todos los trminos que contengan a la incgnita en el primer
miembro de la ecuacin, se factoriza para poder despejarla.

5) ( ) ( ) a x x b ax + = 3 1
Se eliminan los parntesis:
a x b bx ax 3 3 + = +
transponiendo trminos:
b a x bx ax = 3 3
Se factoriza:
( ) b a x b a = 3 3

dividiendo por ( ) 3 b a :
3
3

=
b a
b a
x
Comprobacin:
( ) ( ) ( )
3
3 3 3
3
3 3 3
1
3
3
3
3
2 2 2

+ +
=

+
= |

\
|

\
|

b a
b b ab b ab ab a
b a
b a b b a b b a a
b a
b a
b
b a
b a
a
3
3 3 3
2


=
b a
b ab a

( )
3
3 3 3
3
9 3 3 3 9
3
3 3 3 9
3
3
3
2 2
+

=
+
+
=
+
+
=
|

\
|
+

b a
b ab a
b a
a ab a b a
b a
b a a b a
a
b a
b a

3
3 3 3
3
3 3 3
2 2
+

+

b a
b ab a
b a
b ab a



VII.2.2 ECUACIONES FRACCIONARIAS

Para resolver una ecuacin fraccionaria de primer grado, se multiplican los dos miembros por el mnimo
comn mltiplo de los denominadores con el objeto de eliminarlos y se reduce para convertirla en una
ecuacin entera.

Ejemplos.
Resolver las siguientes ecuaciones fraccionarias:
Pgina del Colegio de Matemticas de la ENP-UNAM Ecuaciones y desigualdades Autor: Dr. Jos Manuel Becerra Espinosa
6

1)
3
7
4
3
3
5
4
1
3
2
+ = + x x x
Se multiplican ambos miembros por el mnimo comn mltiplo de los denominadores, que es 12:
|

\
|
+ =
|

\
|
+
3
7
4
3
3
5
12
4
1
3
2
12 x x x
se efectan las operaciones para cada trmino:
28 9 20 3 8 + = + x x x
se transponen trminos:
8 28 9 20 3 = + x x x
Se reducen los trminos semejantes:
20 8 = x
dividiendo por 8 :
2
5
8
20
= = x
Comprobacin:
24
1
24
15
24
16
8
5
3
2
2
5
4
1
3
2
= = =
|

\
|
+
24
1
24
56
24
45
24
100
3
7
8
15
6
25
3
7
2
5
4
3
2
5
3
5
= + + = + + = +
|

\
|

|

\
|

24
1
24
1


2) x x x x 2
3
11
8
5
6
7
3
2
5
4
= +
Se multiplican ambos miembros por el mnimo comn mltiplo de los denominadores, que es 15:
|

\
|
=
|

\
|
+ x x x x 2
3
11
8
5
6
15 7
3
2
5
4
15
se efectan las operaciones para cada trmino:
x x x x 30 55 120 18 105 10 12 = +
se transponen trminos:
105 12 120 30 55 18 10 + = + + x x x x
Se reducen los trminos semejantes:
27 77 = x
dividiendo por 77 :
77
27
= x
Comprobacin:
385
2477
385
2695
385
90
385
308
7
77
18
5
4
7
77
27
3
2
5
4
= = =
|

\
|
+
385
2477
385
270
385
495
385
3080
385
162
77
54
7
9
8
385
162
77
27
2
77
27
3
11
8
77
27
5
6
= + + = + + =
|

\
|

|

\
|

|

\
|

385
2477
385
2477

Pgina del Colegio de Matemticas de la ENP-UNAM Ecuaciones y desigualdades Autor: Dr. Jos Manuel Becerra Espinosa
7

3) 0
10
5 3
6
4 2
=

x x

Se multiplican ambos miembros por el mnimo comn mltiplo de los denominadores, que es 30 :
( ) 0 30
10
5 3
6
4 2
30 =
|

\
|

x x

se efectan las operaciones para cada trmino:
( ) ( ) 0 5 3 3 4 2 5 = x x
se eliminan los parntesis:
0 15 9 20 10 = + x x
se transponen trminos:
15 20 9 10 = x x
Se reducen los trminos semejantes:
5 = x
Comprobacin:
( ) ( )
0 1 1
10
10
6
6
10
5 15
6
4 10
10
5 5 3
6
4 5 2
= = =




VII.2.3 ECUACIONES QUE CONTIENEN FRACCIONES ALGEBRAICAS

Para resolver este tipo de ecuaciones se multiplica por el MCM de los denominadores que pueden ser un
polinomio. En algunos casos, la ecuacin resultante puede no ser equivalente a la original y la expresin
dada no tiene solucin, en este caso la igualdad es un enunciado falso.

Ejemplos.
Resolver las siguientes ecuaciones que contienen fraccionarias algebraicas:

1)
3
2
8
15
6
5
8
3
5
9
5
4
= +
x x x

Se multiplican ambos miembros por el mnimo comn mltiplo de los denominadores, que es x 15 :
|

\
|
=
|

\
|
+
3
2
8
15
6
5
8
15
3
5
9
5
4
15
x
x
x x
x
se efectan las operaciones para cada trmino:
x x x x 10 120 6 24 25 135 12 = +
se transponen trminos:
25 12 24 10 120 6 135 = + + + x x x x
Se reducen los trminos semejantes:
13 = x
Comprobacin:
( ) ( ) 195
1792
195
25
195
1755
195
12
39
5
9
65
4
13 3
5
9
13 5
4
= = =


( ) 195
1792
195
130
195
1560
195
78
195
24
3
2
8
195
6
65
8
3
2
8
15
6
13 5
8
= = + =


195
1792
195
1792


Pgina del Colegio de Matemticas de la ENP-UNAM Ecuaciones y desigualdades Autor: Dr. Jos Manuel Becerra Espinosa
8

2)
3
1
6
1
5
7
3
1
5
2
2
1
+ = +
x x x

Se multiplican ambos miembros por el mnimo comn mltiplo de los denominadores, que es x 30 :
|

\
|
+ =
|

\
|
+
3
1
6
1
5
7
30
3
1
5
2
2
1
30
x
x
x x
x
se efectan las operaciones para cada trmino:
x x x 10 5 42 10 12 15 + = +
se transponen trminos:
10 12 42 10 5 15 + = + x x x
Se reducen los trminos semejantes:
40 20 = x
dividiendo por 20 :
2
20
40
= = x
Comprobacin:
( ) ( ) 15
8
30
16
30
5
30
6
30
15
6
1
10
2
2
1
2 3
1
2 5
2
2
1
= = + = + = +
( ) 15
8
30
16
30
10
30
5
30
21
3
1
6
1
10
7
3
1
6
1
2 5
7
= = + = + = +
15
8
15
8


3) 10 4
3 5
6
= +
x

Se multiplican ambos miembros por el mnimo comn mltiplo de los denominadores, que es x 3 5 :
( ) ( ) ( )10 3 5 4 3 5
3 5
6
3 5 x x
x
x = +


se efectan las operaciones para cada trmino:
x x 30 50 12 20 6 = +
se transponen trminos:
20 6 50 30 12 = + x x
Se reducen los trminos semejantes:
24 18 = x
dividiendo por 18:
3
4
18
24
= = x
Comprobacin:
10 4 6 4
4 5
6
4
3
4
3 5
6
= + = +

= +
|

\
|


10 10

4)
x x x x 10
4
2
3
4
32
8
4
6
5
2
=
Se multiplican ambos miembros por el mnimo comn mltiplo de los denominadores, que es x 20 :
Pgina del Colegio de Matemticas de la ENP-UNAM Ecuaciones y desigualdades Autor: Dr. Jos Manuel Becerra Espinosa
9

|

\
|
=
|

\
|

x x
x
x x
x
10
4
2
3
4
32
20 8
4
6
5
2
20
se efectan las operaciones para cada trmino:
8 30 160 160 30 8 = x x
se transponen trminos:
30 8 8 30 160 160 + = + x x
Se reducen los trminos semejantes:
16 0 = x
Como la divisin por cero no est definida, entonces el ejemplo planteado no es ecuacin sino un
enunciado falso.

5) 5
6 2
8
2
3
4
+

=
x x

Se multiplican ambos miembros por el mnimo comn mltiplo de los denominadores, que es
6 2 x :
( ) ( )
|

\
|
+

=
|

\
|

5
6 2
8
6 2 2
3
4
6 2
x
x
x
x
se efectan las operaciones para cada trmino:
( ) ( ) ( )5 6 2 8 2 6 2 4 2 + = x x
30 10 8 12 4 8 + = + x x
se transponen trminos:
12 8 30 8 10 4 = x x
Se reducen los trminos semejantes:
42 14 = x
dividiendo por 14 :
14
42

= x
3 = x
Comprobacin:
2
0
4
2
3 3
4
=


( )
5
0
8
5
6 6
8
5
6 3 2
8
+ =

= +


Como la divisin por cero no est definida, entonces el ejemplo planteado no es ecuacin sino un
enunciado falso. Para ambas fracciones, el valor 3 = x no es aceptable. Por lo tanto, la solucin es el
conjunto vaco.


VII.2.4 PROBLEMAS DE APLICACIN

Una de las aplicaciones ms importantes de las ecuaciones es la de resolver problemas de la vida
cotidiana.

Para plantear ecuaciones es conveniente saber traducir un enunciado a una expresin algebraica. Una
til lista de interpretaciones de enunciado a expresin algebraica es la siguiente:




Pgina del Colegio de Matemticas de la ENP-UNAM Ecuaciones y desigualdades Autor: Dr. Jos Manuel Becerra Espinosa
10

Enunciado
Expresin
Algebraica
El doble de x x 2
El triple de x x 3
El cudruplo de x x 4
El cuadrado de x
2
x
El cubo de x
3
x
El antecesor del nmero entero x 1 x
El sucesor del nmero entero x 1 + x
El cuadrado del doble de x
( )
2
2x
El doble del cuadrado de x
2
2x
Un nmero par x 2
Un nmero impar 1 2 + x
Dos nmeros consecutivos x y 1 + x
Dos nmeros pares consecutivos x 2 y 2 2 + x
Dos nmeros impares consecutivos 1 2 x y 1 2 + x
La mitad de x x
2
1

La tercera parte de x x
3
1


Ejemplos.

1) Qu nmero es aquel que si se duplica, y luego se le resta 12, da por resultado el nmero
aumentado en 3 ?
Solucin.
Si x es el nmero buscado.
3 12 2 + = x x
12 3 2 + = x x
15 = x
Por lo tanto, el nmero es el 15.

2) Erick tiene un ao ms que el doble de la edad de Jorge y sus edades suman 97 . Qu edad tienen
ambos?
Solucin.
Si x es la edad de Jorge, entonces la edad de Erick es 1 2 + x
Planteando que la suma de las edades es 97 , se obtiene la ecuacin:
97 1 2 = + + x x
1 97 2 = + x x
96 3 = x
32
3
96
= = x
reemplazando este valor de x en la expresin 1 2 + x se tiene: ( ) 65 1 62 1 32 2 = + = +
Por lo tanto, la edad de Jorge es 32 aos y la de Erick es 65 aos.


Pgina del Colegio de Matemticas de la ENP-UNAM Ecuaciones y desigualdades Autor: Dr. Jos Manuel Becerra Espinosa
11

3) Blanca tiene 300 pesos ms que Ana. Si entre ambas tienen 200 1, , cul es el capital de Blanca?
Solucin.
Si Ana tiene x , entonces Blanca tiene 300 + x
1200 300 = + + x x
300 1200 = + x x
900 2 = x
450
2
900
= = x
Por lo tanto, el capital de Blanca es 750 300 = + x pesos.

4) El permetro de un jardn rectangular es de 58 m. Si el lado mayor mide 11m. ms que el lado
menor. Cunto miden los lados del jardn?
Solucin.
Sea x el lado menor del rectngulo, entonces el lado mayor es 11 + x
Al sumar todos los lados del rectngulo e igualar al permetro dado se obtiene:
58 11 11 = + + + + + x x x x
11 11 58 = + + + x x x x
36 4 = x
9
4
36
= = x
El lado mayor mide: 20 11 9 11 = + = + x m
Por lo tanto, los lados del jardn miden 9 m. y 20 m.

5) El hermano mayor de una familia con tres hermanos tiene 4 aos ms que el segundo y ste 3 ms que
el menor. Si entre los tres suman la edad del padre que tiene 40 aos qu edad tiene cada hermano?
Solucin.
Sea x la edad del hermano menor
3 + x es la edad del hermano mediano
7 + x es la edad del hermano mayor
40 7 3 = + + + + x x x
7 3 40 = + + x x x
30 3 = x
10
3
30
= = x , entonces el hermano mediano tiene 13 3 = + x aos y el mayor 17 7 = + x aos.
Por lo tanto, las edades de los tres hermanos son: 13 10, y 17 aos.

6) Un examen consta de 20 reactivos. Cada respuesta correcta se valora con 3 puntos y cada
respuesta incorrecta se resta 2 puntos. Si al final del examen, un alumno consigui 30 puntos.
Cuntos reactivos contest correctamente y cuntos incorrectamente?
Solucin.
Sea x el nmero de reactivos correctos
x 20 es el nmero de reactivos incorrectos
x 3 es el nmero de puntos conseguidos por los reactivos correctos
( ) x 20 2 es el nmero de puntos perdidos por los reactivos incorrectos
( ) 30 20 2 3 = x x
30 2 40 3 = + x x
40 30 2 3 + = + x x

Pgina del Colegio de Matemticas de la ENP-UNAM Ecuaciones y desigualdades Autor: Dr. Jos Manuel Becerra Espinosa
12

70 5 = x
14
5
70
= = x
6 14 20 20 = = x
Por lo tanto, el alumno tuvo 14 respuestas correctas y 6 incorrectas.

7) Hallar un nmero tal que su mitad ms su cuarta parte ms 1, sea igual al nmero pedido.
Solucin.
Sea x el nmero buscado
x x x = + + 1
4
1
2
1

( ) x x x 4 1
4
1
2
1
4 =
|

\
|
+ +
x x x 4 4 2 = + +
4 4 2 = + x x x
4 = x
4
1
4
=

= x
El nmero buscado es 4 .

8) Una llave llena un depsito en 3 horas y otra lo hace en 6 horas. Si el depsito est vaco y se abren
las dos llaves a la vez, cunto tiempo tardar en llenarse?
Solucin.
La primera llave llena
3
1
del depsito en una hora
La segunda llave llena
6
1
del depsito en una hora
Si x el tiempo en horas que las llaves llenan juntas el depsito, entonces
x
1
es la fraccin de depsito
que llenan juntas en una hora, As que:
x
1
6
1
3
1
= +
|

\
|
=
|

\
|
+
x
x x
1
6
6
1
3
1
6
6 2 = + x x
6 3 = x
2
3
6
= = x
Por lo tanto, el depsito se llenara en dos horas.


VII.2.5 GRFICA DE UNA ECUACIN DE PRIMER GRADO

Resolver una ecuacin es encontrar un valor de x que, al ser sustituido en la ecuacin y realizar las
operaciones indicadas, se llegue a que la igualdad es cierta. Para ello se debe dejar sola a la variable x
de un lado de la ecuacin. A esto se le llama despejar a la variable.
Pgina del Colegio de Matemticas de la ENP-UNAM Ecuaciones y desigualdades Autor: Dr. Jos Manuel Becerra Espinosa
13

x
2 1 -1 -2
1
2
y
-1
-2
1 = x
x
7 5 3 1
1
3
y
-1
-5
10 = x
9 11 -1
5
-3
x
4 2 -2 -4
2
4
y
-2
-4
9
25
= x
Grficamente, la solucin de la ecuacin est representada por una lnea recta vertical en el plano
cartesiano. La solucin es el valor de la abscisa del punto en el que esa recta corta al eje x .

Ejemplos.
Representar grficamente la solucin de las siguientes ecuaciones de primer grado:




1) ( ) ( ) 1 2 4 1 3 + = x x
Solucin.
2 2 4 3 3 = x x
3 2 4 2 3 + = + x x
5 5 = x
1
5
5
= = x







2) 5
3 6
= +
x x

( ) 5 6
3 6
6 =
|

\
|
+
x x

30 2 = + x x
30 3 = x
10
3
30
= = x







3)
5
1
7
6
1
2
7
8
3
1
+ =
x x x

|

\
|
+ =
|

\
|

5
1
7
6
1
30
2
7
8
3
1
30
x
x
x x
x
x x x 6 210 5 105 240 10 + =
105 10 5 6 210 240 + = + x x x
100 36 = x
77 2
9
25
36
100
. x =

=


Pgina del Colegio de Matemticas de la ENP-UNAM Ecuaciones y desigualdades Autor: Dr. Jos Manuel Becerra Espinosa
14

VII.3 DESIGUALDADES DE PRIMER GRADO EN UNA VARIABLE

La expresin b a significa que " a " no es igual a " b ".

Segn los valores particulares de a y de b , puede tenerse b a > , que se lee a mayor que b , cuando
la diferencia b a es positiva y b a < que se lee a menor que b , cuando la diferencia b a es
negativa.

La notacin b a , que se lee a es mayor o igual que b , significa que b a > o que b a = pero no
ambos. Por su parte, la notacin b a que se lee a es menor o igual que b , significa que b a < o que
b a = pero no ambos.

Una desigualdad se obtiene al escribir dos expresiones numricas o algebraicas relacionadas con alguno
de los smbolos < > , , o .

Ejemplos de desigualdades:
1) 3 4 >
2) 10 < a
3) 5 b
4) 1
2
x

Lo mismo que en las igualdades, en toda desigualdad, los trminos que estn a la izquierda del signo
mayor o menor, forman el primer miembro de la desigualdad, y los trminos de la derecha, forman el
segundo miembro.

De la definicin de desigualdad, se deduce que:

Todo nmero positivo es mayor que cero
Todo nmero negativo es menor que cero
Si dos nmeros son negativos, es mayor el que tiene menor valor absoluto
Si b a > entonces a b < .

Los signos > o < determinan dos sentidos opuestos en las desigualdades, dependiendo si el primer
miembro es mayor o menor que el segundo. Se dice que una desigualdad cambia de sentido, cuando el
miembro mayor se convierte en menor o viceversa.

Existen dos clases de desigualdades: las absolutas y las condicionales.

Desigualdad absoluta es aquella que se verifica para cualquier valor que se atribuya a las literales
que figuran en ella. Por ejemplo: x x > +1
2


Desigualdad condicional es aquella que slo se verifica para ciertos valores de las literales. Por
ejemplo: 0 15 3 > x que solamente satisface para 5 > x . En este caso se dice que 5 es el lmite
de x .

Las desigualdades condicionales se llaman inecuaciones.
Sean b , a R y 0 a , una desigualdad de primer grado en una variable x se define como:

+
< +
+
> +
0
0
0
0
b ax
b ax
b ax
b ax


Pgina del Colegio de Matemticas de la ENP-UNAM Ecuaciones y desigualdades Autor: Dr. Jos Manuel Becerra Espinosa
15

Propiedades de las desigualdades:

Sean c , b , a tres nmeros reales.

I. Una desigualdad no cambia de sentido cuando se aade o se resta un mismo nmero a cada miembro

Esto es, si b a > , entonces se cumple que c b c a + > + .

Ejemplos.
1) Si a la desigualdad 3 7 > se le suma 2 a ambos miembros, entonces, se cumple que 2 3 2 7 + > + ,
ya que: 5 9 >

2) Si a la desigualdad 8 16 > se le resta 5 a ambos miembros, entonces, se cumple que 5 8 5 16 > ,
ya que: 3 11 >
Consecuencia de esta propiedad, puede suprimirse un trmino en un miembro de una desigualdad,
teniendo cuidado de agregar en el otro miembro el trmino simtrico del suprimido. Es decir, se puede
pasar un trmino de un miembro a otro, cambiando su signo, porque esto equivale a sumar o restar una
misma cantidad a los dos miembros.

Ejemplo.
9 3 4 8 > x x
4 9 3 8 + > x x

II. Una desigualdad no cambia de sentido cuando se multiplican sus dos miembros por un mismo factor
positivo, o se dividen por un mismo divisor, tambin positivo.

Esto es, dado un nmero 0 > c , si b a > entonces se cumple que c b c a > y que
c
b
c
a
>

Ejemplos.
1) Si a la desigualdad 2 5 > se multiplica por 3 a ambos miembros, entonces, se cumple que 3 2 3 5 >
, ya que 6 15 >

2) Si a la desigualdad 28 36 > se divide por 4 a ambos miembros, entonces, se cumple que
4
28
4
36
> ,
ya que 7 9 >

III. Una desigualdad cambia de sentido cuando se multiplican sus dos miembros por un mismo factor
negativo, o se dividen por un mismo divisor, tambin negativo.

Esto es, dado un nmero 0 < c , si b a > entonces se cumple que c b c a < y que
c
b
c
a
<

Ejemplos.
1) Si a la desigualdad 3 6 > se multiplica por 4 a ambos miembros, entonces, se cumple que
( ) ( ) 4 3 4 6 < , ya que 12 24 <

2) Si a la desigualdad 10 16 > se divide por 2 a ambos miembros, entonces, se cumple que
2
10
2
16

<

, ya que 5 8 <
Pgina del Colegio de Matemticas de la ENP-UNAM Ecuaciones y desigualdades Autor: Dr. Jos Manuel Becerra Espinosa
16

Consecuencia de la propiedad anterior pueden cambiarse todos los signos de una desigualdad, con tal
que se cambie el sentido de la misma; porque esto equivale a multiplicar sus dos miembros por -1.

Ejemplo.
x x 4 2 18 6 < +
x x 4 2 18 6 + >


VII.3.1 INECUACIONES ENTERAS

Las inecuaciones son desigualdades entre expresiones algebraicas. A diferencia de las ecuaciones, que
slo se verifican para algunos valores de la variable, las inecuaciones tienen infinitas soluciones. El
procedimiento para resolverlas es similar al de las ecuaciones, slo que deben tenerse en cuenta las
propiedades de las desigualdades.

Para resolver una inecuacin de primer grado se transponen los trminos (pasar los trminos de un
miembro a otro cambiando el signo equivale a aplicar la propiedad I) para que aquellos que contienen a
la incgnita queden en el primer miembro y los trminos independientes en el otro. Finalmente, para
despejar la incgnita se divide por el valor del coeficiente, teniendo en cuenta la segunda o tercera
propiedad de las desigualdades, segn el signo del coeficiente.

Ejemplos.
Resolver las siguientes inecuaciones enteras:

1) 8 2 6 4 > + x x
Solucin.
Se transponen trminos:
6 8 2 4 > x x
se reducen los trminos semejantes:
14 2 > x
dividiendo por 2 :
7
2
14
>

> x x

2) 6 2 10 5 2 3 13 + + + x x x x
Solucin.
Se transponen trminos:
2 6 10 2 5 3 13 + x x x x
se reducen los trminos semejantes:
6 3 x
dividiendo por 3 :
2
3
6

x x

3) 10 8 34 3 6 5 + > + x x x
Solucin.
Se transponen trminos:
6 10 34 8 3 5 > x x x
se reducen los trminos semejantes:
18 6 > x
dividiendo por 6 y aplicando la tercera propiedad, la desigualdad cambia de sentido:
Pgina del Colegio de Matemticas de la ENP-UNAM Ecuaciones y desigualdades Autor: Dr. Jos Manuel Becerra Espinosa
17

3
6
18
<

< x x

4) x x x x x 4 23 4 8 13 6 10 5 2 3 + + + >
Solucin.
Se transponen trminos:
6 2 23 4 13 4 8 10 5 3 + + + + > + x x x x x
se reducen los trminos semejantes:
48 8 > x
dividiendo por 8 y aplicando la tercera propiedad, la desigualdad cambia de sentido:
6
8
48
<

< x x

5) ( ) ( ) ( ) ( ) x x x x x + + + + + + 8 2 4 10 3 5 3 4 1 3 2 5
Solucin.
Eliminando parntesis:
x x x x x + + + + + + 32 8 30 3 20 12 1 15 10
Se transponen trminos:
20 1 15 32 30 8 3 12 10 + + + + x x x x x
se reducen los trminos semejantes:
96 10 x
dividiendo por 10:
5
48
10
96
x x

Una inecuacin de primer grado literal es aquella que contiene otras expresiones literales aparte de la
incgnita, las cuales deben considerarse como valores constantes.

Para resolver inecuaciones literales se efecta el mismo procedimiento aplicado en los ejemplos
anteriores. La variante es que cuando se tengan todos los trminos que contengan a la incgnita en el
primer miembro de la inecuacin, se factoriza para poder despejarla.

6) ( ) ( ) ab a x abx x b ax + > 5 6 4 3 2
Eliminando parntesis:
ab a x abx b bx ax + > + 5 5 6 12 3 2
Se transponen trminos:
b ab a x abx bx ax 12 5 5 6 3 2 >
factorizando x :
( ) b ab a ab b a x 12 5 5 6 3 2 >
si ( ) 0 5 6 3 2 > ab b a , entonces la solucin es
5 6 3 2
12 5


>
ab b a
b ab a
x
si ( ) 0 5 6 3 2 < ab b a , entonces la solucin es
5 6 3 2
12 5


<
ab b a
b ab a
x


VII.3.2 INECUACIONES FRACCIONARIAS

Para resolver una inecuacin fraccionaria de primer grado, se multiplican sus dos miembros por el
mnimo comn mltiplo de los denominadores con el objeto de eliminarlos y se reduce para convertirla en
Pgina del Colegio de Matemticas de la ENP-UNAM Ecuaciones y desigualdades Autor: Dr. Jos Manuel Becerra Espinosa
18

una inecuacin entera. Cuando el denominador contiene la incgnita, tiene que analizarse cuando es
tanto positiva como negativa. Para ambos casos debe obtenerse la respectiva interseccin de las
restricciones. La solucin de la inecuacin, es la unin de los dos intervalos obtenidos.

Ejemplos.
Resolver las siguientes inecuaciones fraccionarias:

1)
3
7
5
4
3
1
5
2
> + x x
Se multiplican ambos miembros por el mnimo comn mltiplo de los denominadores, que es 15:
|

\
|
>
|

\
|
+
3
7
5
4
15
3
1
5
2
15 x x
se efectan las operaciones para cada trmino:
35 12 5 6 > + x x
se transponen trminos:
6 35 12 5 > x x
Se reducen los trminos semejantes:
41 7 > x
dividiendo por 7 y aplicando la tercera propiedad, la desigualdad cambia de sentido:
7
41
7
41
<

< x x

2) x x x 3
2
1
5
2
8
3
2
4
5
+
Se multiplican ambos miembros por el mnimo comn mltiplo de los denominadores, que es 60 :
|

\
|

|

\
|
+ x x x 3
2
1
5
2
60 8
3
2
4
5
60
se efectan las operaciones para cada trmino:
x x x 180 30 24 480 40 75 +
se transponen trminos:
480 75 30 180 24 40 + + x x x
Se reducen los trminos semejantes:
375 196 x
dividiendo por 196:
196
375
x

3) x x x
6
8
4
10
6
2
4
3
5
4
9
+ + > +
Se multiplican ambos miembros por el mnimo comn mltiplo de los denominadores, que es 12:
|

\
|
+ + >
|

\
|
+ x x x
6
8
4
10
6
2
12 4
3
5
4
9
12
se efectan las operaciones para cada trmino:
x x x 16 30 4 48 20 27 + + > +
se transponen trminos:
48 27 30 16 4 20 + > x x x
Se reducen los trminos semejantes:
Pgina del Colegio de Matemticas de la ENP-UNAM Ecuaciones y desigualdades Autor: Dr. Jos Manuel Becerra Espinosa
19

51 0 > x
como la divisin por cero no est definida, entonces la expresin presenta un enunciado falso. Ntese
que simplificando la inecuacin se llega a
2
5
3
5
4
7
3
5
+ > x x , expresin que es imposible que se cumpla.

4)
x x 4
1
6
8
3
5
6
7
> +
Se multiplican ambos miembros por el mnimo comn mltiplo de los denominadores, que es x 12 :
Si 0 > x se tiene:
|

\
|
>
|

\
|
+
x
x
x
x
4
1
6
8
12
3
5
6
7
12
se efectan las operaciones para cada trmino:
3 16 20 14 > + x x
se transponen trminos:
14 3 16 20 > x x
Se reducen los trminos semejantes:
17 4 > x
dividiendo por 4 :
4
17
> x
dadas las restricciones 0 > x y
4
17
> x , su interseccin es 0 > x
Si 0 < x entonces el resultado de la desigualdad cambia de sentido
4
17
< x
dadas las restricciones 0 < x y
4
17
< x , su interseccin es
4
17
< x
la solucin est dada por: ( )
|

\
|
, , 0
4
17
U

5)
x x x x 2
7
3
1
2
2
1
3
2
5
4

Se multiplican ambos miembros por el mnimo comn mltiplo de los denominadores, que es x 30 :
Si 0 > x se tiene:
|

\
|

|

\
|

x x
x
x x
x
2
7
3
1
2 30
2
1
3
2
5
4
30
se efectan las operaciones para cada trmino:
105 10 60 15 20 24 x x
se transponen trminos:
15 24 105 10 60 20 + x x
Se reducen los trminos semejantes:
124 80 x
dividiendo por 80 y aplicando la tercera propiedad, la desigualdad cambia de sentido:
20
31
80
124

x x
Pgina del Colegio de Matemticas de la ENP-UNAM Ecuaciones y desigualdades Autor: Dr. Jos Manuel Becerra Espinosa
20

dadas las restricciones 0 > x y
20
31
x , su interseccin es
20
31
> x
Si 0 < x entonces el resultado de la desigualdad cambia de sentido:
20
31
x
dadas las restricciones 0 < x y
20
31
< x , su interseccin es 0 < x
Por lo tanto, la solucin est dada por: ( )
|

\
|
, ,
20
31
0 U

6) 0 3
5
2
> +
x

Se multiplican ambos miembros por el mnimo comn mltiplo de los denominadores, que es x 5 :
( ) ( )0 5 3
5
2
5 x
x
x >
|

\
|
+


Si 0 5 > x , que implica 5 < x se tiene:
( ) 0 3 5 2 > + x
se efectan las operaciones para cada trmino:
0 3 15 2 > + x
se transponen trminos:
15 2 3 > x
Se reducen los trminos semejantes:
17 3 > x
dividiendo por 3 y aplicando la tercera propiedad, la desigualdad cambia de sentido:
3
17
3
17
<

< x x
dadas las restricciones 5 < x y
3
17
< x , su interseccin es 5 < x
Si 0 5 < x , que implica 5 > x entonces el resultado de la desigualdad cambia de sentido
3
17
> x
dadas las restricciones 5 > x y
3
17
> x , su interseccin es
3
17
> x
Por lo tanto, la solucin est dada por: ( )
|

\
|
, ,
3
17
5 U

7) 12 18
6 2
5
< +
x

Se multiplican ambos miembros por el mnimo comn mltiplo de los denominadores, que es 6 2 x :
( ) ( )( ) 12 6 2 18
6 2
5
6 2 <
|

\
|
+

x
x
x
Si 0 6 2 > x , que implica 3 > x se tiene:
( ) ( )( ) 12 6 2 18 6 2 5 < + x x
se efectan las operaciones para cada trmino:
72 24 108 36 5 + < + x x
se transponen trminos:
Pgina del Colegio de Matemticas de la ENP-UNAM Ecuaciones y desigualdades Autor: Dr. Jos Manuel Becerra Espinosa
21

x
2 1 -1 -2
2 > x
x
2 1 -1 -2
2 > x
x
0 -2 -6 -8
21
125
x
-4 -10
x
0 -2 -6 -8
21
125
x
-4 -10
108 5 72 24 36 + < + x x
Se reducen los trminos semejantes:
175 60 < x
dividiendo por 60 :
12
35
60
175
< < x x
dadas las restricciones 3 > x y
12
35
< x , su interseccin es 3
12
35
< < x
Si 0 6 2 < x , que implica 3 < x entonces el resultado de la desigualdad cambia de sentido
12
35
> x
dadas las restricciones 3 < x y
12
35
> x , no existe interseccin
Por lo tanto, la solucin est dada por: 3
12
35
< < x .


VII.3.3 GRFICA DE UNA INECUACIN DE PRIMER GRADO

Resolver una inecuacin es encontrar el conjunto de valores de x que cumplan la desigualdad. Grficamente, la
solucin de una inecuacin de primer grado est representada por un intervalo del eje de las abscisas a partir de
un valor lmite a . Si la solucin es de la forma a x > , entonces la regin ser todos los nmeros que estn a la
derecha de a sin incluirlo. Si la solucin es de la forma a x , la regin incluye al valor a . De la misma forma,
si la solucin es de la forma a x < , entonces la regin ser todos los nmeros que estn a la izquierda de a sin
incluirlo. Si la solucin es de la forma a x , la regin incluye al valor a . Dependiendo del tipo de desigualdad
el conjunto solucin puede ser uno o dos intervalos, la totalidad de los nmeros reales o el conjunto vaco.

Ejemplos.
Representar grficamente la solucin de las siguientes inecuaciones de primer grado:

1) ( ) ( ) 6 2 3 2 1 4 + > + x x
Solucin.
18 6 2 4 4 > + x x
4 18 2 6 4 > + x x
20 10 > x
10
20
> x
2 > x

2) 5
4
11
8
3
5
2
9
7
4
3
+ + x x x
|

\
|
+ +
|

\
|
5
4
11
8
3
5
12
2
9
7
4
3
12 x x x
60 33 96 20 54 84 9 + + x x x
54 9 60 20 33 96 84 + + + x x x
125 21 x
21
125
x
Pgina del Colegio de Matemticas de la ENP-UNAM Ecuaciones y desigualdades Autor: Dr. Jos Manuel Becerra Espinosa
22

3)
x x 8
5
4
13
2
7 3
< +
si 0 > x
|

\
|
<
|

\
|
+
x
x
x
x
8
5
4
13
8
2
7 3
8
5 26 28 24 < + x x
24 5 26 28 + < x x
19 2 < x
2
19
< x
dadas las restricciones 0 > x y
2
19
< x , su interseccin es
2
19
0 < < x
Si 0 < x entonces el resultado de la desigualdad cambia de sentido
2
19
> x
dadas las restricciones 0 < x y
2
19
> x , no hay interseccin.
Por lo tanto, la solucin est dada por:
2
19
0 < < x











VII.4 ECUACIONES DE SEGUNDO GRADO EN UNA VARIABLE

Una ecuacin de segundo grado en una variable es aquella que, una vez realizadas todas las
reducciones posibles, el mximo exponente es dos.

Una ecuacin de este tipo tambin es llamada ecuacin cuadrtica y tiene la forma general:

0
2
= + + c bx ax

donde 0 a , b y c son nmeros reales; y x es la incgnita. El monomio
2
ax recibe el nombre de
trmino cuadrtico, bx se conoce como trmino lineal y c es el trmino independiente.

Ejemplos de ecuaciones de segundo grado en una variable:
1) 0 4 2 5
2
= + x x
2)
9
4
11
6
8
3
2
= x x
3) 0 57 8 46 3
2
= x . x .
4) 0 28 7
2
= x
x
7 5 3 1 9
2
19
0 < < x
11 -1 x
7 5 3 1 9
2
19
0 < < x
11 -1
Pgina del Colegio de Matemticas de la ENP-UNAM Ecuaciones y desigualdades Autor: Dr. Jos Manuel Becerra Espinosa
23

Una ecuacin de segundo grado tiene siempre dos respuestas (algunas veces repetidas). El objetivo de
resolverla es obtener las races
1
x y
2
x , si existen, para los que la igualdad de la ecuacin es cierta.

Una ecuacin cuadrtica puede ser de dos tipos:

Ecuacin completa si 0 b y 0 c
Ecuacin incompleta si 0 = b 0 = c .

En la vida prctica, cuando se tiene que resolver una ecuacin cuadrtica que surge de un problema
concreto, la mayora de las veces sta no tiene un formato sencillo, sin embargo, puede reducirse a
alguna de estas formas para decidir el mtodo que se usar para resolverla.

Ejemplos.
1) 0 1 8 3
2
= + x x es una ecuacin completa
2) 0 12 4
2
= x x es una ecuacin incompleta ya que no tiene el trmino independiente
3) 0 28 7
2
= x es una ecuacin incompleta porque carece del trmino lineal.


VII.4.1 ECUACIONES INCOMPLETAS

Sea una ecuacin incompleta de la forma

trasponiendo el trmino independiente: c ax =
2

dividiendo la ecuacin por a :
a
c
x =
2

Para despejar x de esta ecuacin, se busca un nmero que elevado al cuadrado sea igual a
a
c
.
Como
2
2
c
a
c
=
|
|

\
|
si 0 >
a
c
y tambin
2
2
c
a
c
=
|
|

\
|
si 0 >
a
c
, entonces estos dos nmeros
se encuentran en la recta numrica a un lado y al otro del cero y su distancia al origen es
a
c
.
Lo anterior significa que:
a
c
x = , lo cual implica que
a
c
x = o
a
c
x = .
Por lo tanto, las races de la ecuacin

estn dadas por:
a
c
x =
1

a
c
x =
2


Ntese como las races de la ecuacin existirn siempre y cuando los coeficientes a y c tengan signos
opuestos.

Ejemplos.
Resolver las siguientes ecuaciones de segundo grado incompletas:

1)
0
2
= +c ax
0
2
= +c ax
0 12 3
2
= x
Pgina del Colegio de Matemticas de la ENP-UNAM Ecuaciones y desigualdades Autor: Dr. Jos Manuel Becerra Espinosa
24

Solucin.
2 2 4 4
3
12
12 3
2 1
2 2
= = = = = = x , x x x x
Comprobacin:
( ) ( ) 0 12 12 12 4 3 12 2 3
2
= = =
( ) ( ) 0 12 12 12 4 3 12 2 3
2
= = =

2) 0 54 6
2
= x
Solucin.
3 3 9 9
6
54
54 6
2 1
2 2
= = = = = = x , x x x x
Comprobacin:
( ) ( ) 0 54 54 54 9 6 54 3 6
2
= = =
( ) ( ) 0 54 54 54 9 6 54 3 6
2
= = =

3) 0 5
5
1
2
= + x
Solucin.
Multiplicando por 5 :
( ) 0 5 5
5
1
5
2
= |

\
|
+ x
25 25
1
25
25 0 25
2 2 2
= =

= = = + x x x x
5 5
2 1
= = x , x
Comprobacin:
( ) 0 5 5 5
5
25
5 5
5
1
2
= + = + = +
( ) 0 5 5 5
5
25
5 5
5
1
2
= + = + = +

4) 4 7 8 36 2 14 10 6
2 2 2 2 2
+ = + + + x x x x x
Solucin.
Reduciendo trminos semejantes se tiene: 0 14 2
2
= x
7 7 7 7
2
14
14 2
2 1
2 2
= = = = = = x , x x x x
Comprobacin:
( ) ( ) ( ) = + + +
2
2 2 2
36 7 2 7 14 10 7 6 ( ) ( ) ( ) 44 36 14 98 10 42 36 7 2 7 14 10 7 6 = + + + = + + +
( ) ( ) ( ) 44 4 48 49 8 7 7 7 8 7 7 7 8 7
2 2
= = + = + = +
44 44
( ) ( ) ( ) = + + +
2
2 2 2
36 7 2 7 14 10 7 6 ( ) ( ) ( ) 44 36 14 98 10 42 36 7 2 7 14 10 7 6 = + + + = + + +
( ) ( ) ( ) 44 4 48 49 8 7 7 7 8 7 7 7 8 7
2 2
= = + = + = +
44 44

Pgina del Colegio de Matemticas de la ENP-UNAM Ecuaciones y desigualdades Autor: Dr. Jos Manuel Becerra Espinosa
25

5) 0 32 8
2
= + x
Solucin.
4 4
8
32
32 8
2 2
= =

= = x x x , por lo tanto no existen soluciones reales.



Sea una ecuacin incompleta de la forma 0
2
= +bx ax

factorizando el primer miembro: ( ) 0 = +b ax x
aplicando la propiedad cero de los nmeros reales
3
: 0 = x y 0 = + b ax
despejando x de la segunda ecuacin se obtiene:
a
b
x =
Por lo tanto, las races de esta ecuacin estn dadas por:
0
1
= x
a
b
x =
2

Ntese como una raz siempre ser cero y la otra siempre existe.

Es comn que en muchos ejercicios el factor comn es de la forma kx , donde k es el mximo comn
divisor de a y b , entonces si 0
2
= +bx ax , se tiene que 0 =
|

\
|
+
k
b
x
k
a
kx

Ejemplos.
Resolver las siguientes ecuaciones de segundo grado incompletas:
1) 0 8 2
2
= x x
Solucin.
( )

= =
= =
= =
4 0 4
0 0 2
0 4 2 0 8 2
2
1 2
x x
x x
x x x x
Comprobacin:
( ) ( ) 0 0 8 0 2
2
=
( ) ( ) ( ) 0 32 32 32 16 2 4 8 4 2
2
= = =

2) 0 10 5
2
= + x x
Solucin.
( )

= = +
= =
= + = +
2 0 2
0 0 5
0 2 5 0 10 5
2
1 2
x x
x x
x x x x
Comprobacin:
( ) ( ) 0 0 10 0 5
2
= +
( ) ( ) ( ) 0 20 20 20 4 5 2 10 2 5
2
= = = +

3) 0 28 6
2
= + x x
Solucin.

3
Esta propiedad establece que si el producto de dos nmeros es cero, entonces uno de ellos o ambos es cero.
Pgina del Colegio de Matemticas de la ENP-UNAM Ecuaciones y desigualdades Autor: Dr. Jos Manuel Becerra Espinosa
26

( )

= = =
= =
= = +
3
14
14 3 0 14 3
0 0 2
0 14 3 2 0 28 6
2
1
2
x x x
x x
x x x x

4) x x x x x x x x x 3 9 12 2 4 8 3 7 5
2 2 2 2
+ = +
Solucin.
Reduciendo trminos semejantes se tiene: 0 12 9
2
= x x
( )

= = =
= =
= =
3
4
4 3 0 4 3
0 0 3
0 4 3 3 0 12 9
2
1
2
x x x
x x
x x x x
Comprobacin:
( ) ( ) ( ) ( ) ( ) 0 0 4 0 8 0 3 0 7 0 5
2 2
= +
( ) ( ) ( ) ( ) 0 0 3 0 9 0 12 0 2
2 2
= +
0 0
Comprobacin:
9
4
9
48
9
96
9
48
3
84
9
80
3
16
3
32
9
48
3
28
9
80
3
4
4
3
4
8
3
4
3
3
4
7
3
4
5
2 2
= + = + = |

\
|
|

\
|
+ |

\
|
|

\
|
|

\
|

9
4
9
36
9
144
9
144
9
32
4 16 16
9
32
3
4
3
3
4
9
3
4
12
3
4
2
2 2
= + = + = |

\
|
|

\
|
|

\
|
+ |

\
|

9
4
9
4


5) 0
2
7
5
3
2
= x x
Solucin.
Multiplicando por 10:
( ) 0 10
2
7
5
3
10
2
= |

\
|
x x 0 35 6
2
= x x
( )

= = = +
= =
= + =
6
35
35 6 0 35 6
0 0
0 35 6 0 35 6
2
1
2
x x x
x x
x x x x
Comprobacin:
0
12
245
12
245
12
245
36
1225
5
3
6
35
2
7
6
35
5
3
2
= + = + |

\
|
= |

\
|
|

\
|



VII.4.2 ECUACIONES COMPLETAS UTILIZANDO FRMULA GENERAL

Existe una frmula general que puede aplicarse a cualquier ecuacin de segundo grado en una variable y
que permite conocer la naturaleza de las races.

Para resolver la ecuacin de segundo grado en el caso general, se necesita que el primer miembro sea
un cuadrado perfecto:

Pgina del Colegio de Matemticas de la ENP-UNAM Ecuaciones y desigualdades Autor: Dr. Jos Manuel Becerra Espinosa
27

Sea la ecuacin: 0
2
= + + c bx ax
se traspone el trmino independiente al segundo miembro: c bx ax = +
2

dividiendo por a :
a
c
x
a
b
x = +
2

sumando
2
2
|

\
|
a
b
para que el primer miembro sea un trinomio cuadrado perfecto:
2 2
2
2 2
|

\
|
+ = |

\
|
+ +
a
b
a
c
a
b
x
a
b
x
expresin que equivale a:
2
2
2
2
4 2 a
b
a
c
a
b
x
a
b
x + = |

\
|
+ +
acomodando el segundo miembro:
a
c
a
b
a
b
x
a
b
x = |

\
|
+ +
2
2
2
2
4 2

expresin que equivale a:
2
2
2
2
4
4
2 a
ac b
a
b
x
a
b
x

= |

\
|
+ +
factorizando el trinomio cuadrado perfecto:
2
2
2
4
4
2 a
ac b
a
b
x

= |

\
|
+
extrayendo raz cuadrada en ambos miembros:
2
2
4
4
2 a
ac b
a
b
x

= +
aplicando propiedades de los radicales:
a
ac b
a
b
x
2
4
2
2

= +
se traspone el trmino
a
b
2
al segundo miembro:
a
b
a
ac b
x
2 2
4
2


=
acomodando convenientemente se llega a:

a
ac b b
x
2
4
2

=

expresin conocida como frmula general para resolver una ecuacin de segundo grado.

En la frmula general, la cantidad: ac b 4
2
es llamada discriminante de la ecuacin y determina la
naturaleza de las races, de acuerdo a lo siguiente:

Si 0 4
2
> ac b , las races son reales y diferentes.
Si 0 4
2
= ac b , las races son reales e iguales.
Si 0 4
2
< ac b , las races son complejas conjugadas.

Ejemplos.
Aplicando la frmula general, resolver las siguientes ecuaciones de segundo grado:

1) 0 30 21 3
2
= + + x x
Simplificando la ecuacin para que la sustitucin sea ms sencilla: 0 10 7
2
= + + x x
Pgina del Colegio de Matemticas de la ENP-UNAM Ecuaciones y desigualdades Autor: Dr. Jos Manuel Becerra Espinosa
28

10 7 1 = = = c , b , a
Sustituyendo en la frmula general se tiene:
( ) ( )( )
( ) 2
3 7
2
9 7
2
40 49 7
1 2
10 1 4 7 7
2

=

=

=

= x
2
2
4
2
3 7
1
=

=
+
= x
5
2
10
2
3 7
2
=

=

= x
Comprobacin:
( ) ( ) ( ) 0 30 42 12 30 42 4 3 30 2 21 2 3
2
= + = + = + +
( ) ( ) ( ) 0 30 105 75 30 105 25 3 30 5 21 5 3
2
= + = + = + +

2) 0 24 14 2
2
= + x x
Simplificando la ecuacin para que la sustitucin sea ms sencilla: 0 12 7
2
= + x x
12 7 1 = = = c , b , a
Sustituyendo en la frmula general se tiene:
( ) ( ) ( )( )
( ) 2
1 7
2
1 7
2
48 49 7
1 2
12 1 4 7 7
2

=

=

= x
4
2
8
2
1 7
1
= =
+
= x
3
2
6
2
1 7
2
= =

= x
Comprobacin:
( ) ( ) ( ) 0 24 56 32 24 56 16 2 24 4 14 4 2
2
= + = + = +
( ) ( ) ( ) 0 24 42 18 24 42 9 2 24 3 14 3 2
2
= + = + = +

3) 4 8 5 3 19 7 11
2 2
+ + = + + x x x x
Reduciendo trminos semejantes se tiene:

Simplificando la ecuacin para que la sustitucin sea ms sencilla: 0 4 4
2
= + + x x
4 4 1 = = = c , b , a
Sustituyendo en la frmula general se tiene:
( ) ( )( )
( ) 2
0 4
2
0 4
2
16 16 4
1 2
4 1 4 4 4
2

=

=

=

= x
2
2
4
2
0 4
1
=

=
+
= x
2
2
4
2
0 4
2
=

=

= x
Comprobacin:
( ) ( ) ( ) 49 19 14 44 19 14 4 11 19 2 7 2 11
2
= + = + = + +
( ) ( ) ( ) 49 4 32 10 3 4 4 8 10 3 4 2 8 2 5 3
2
= + + + = + + + = + +
49 49
0 12 12 3
2
= + + x x
Pgina del Colegio de Matemticas de la ENP-UNAM Ecuaciones y desigualdades Autor: Dr. Jos Manuel Becerra Espinosa
29

4) 0
4
10
3
4
6
1
2
= + x x
Multiplicando por 12:
( ) 0 12
4
10
3
4
6
1
12
2
= |

\
|
+ x x
0 30 16 2
2
= + x x
Simplificando la ecuacin para que la sustitucin sea ms sencilla: 0 15 8
2
= + x x
15 8 1 = = = c , b , a
Sustituyendo en la frmula general se tiene:
( ) ( ) ( )( )
( ) 2
2 8
2
4 8
2
60 64 8
1 2
15 1 4 8 8
2

=

=

= x
5
2
20
2
2 8
1
= =
+
= x
3
2
6
2
2 8
2
= =

= x
Comprobacin:
( ) ( ) 0
12
30
12
80
12
50
4
10
3
20
6
25
4
10
5
3
4
5
6
1
2
= + = + = +
( ) ( ) 0
12
30
12
48
12
18
4
10
4
6
9
4
10
3
3
4
3
6
1
2
= + = + = +

5) 0 7 8 5
2
= + + x x
7 8 5 = = = c , b , a
Sustituyendo en la frmula general se tiene:
( )( )
( ) 10
76 8
10
140 64 8
5 2
7 5 4 8 8
2

=

=

= x , por lo tanto no existen soluciones reales.


VII.4.3 ECUACIONES COMPLETAS UTILIZANDO FACTORIZACIN

Toda ecuacin cuadrtica 0
2
= + + c bx ax es una ecuacin en la cual uno de sus miembros es un
trinomio de segundo grado y el otro es cero. Muchos trinomios de segundo grado, pueden factorizarse
como el producto de dos binomios que tienen un trmino en comn
4
.

El trmino comn de los binomios es de grado uno ya que es raz del trmino cuadrtico. Para encontrar
las races se resuelven las dos ecuaciones de primer grado.

Este mtodo aplica nica y exclusivamente si el miembro de la derecha es cero y si el primer miembro es
factorizable de acuerdo a la forma que se expuso en los subtemas V.2.6 y V.2.7.




4
De acuerdo a lo expuesto en la seccin V.2.7, el ltimo paso de la factorizacin de un trinomio de la forma c bx ax + +
2
consiste
en dividir por a y el resultado final puede no ser el producto de dos binomios por un trmino comn. Para resolver ecuaciones del
tipo 0
2
= + + c bx ax no es necesario dividir por a , as que el resultado ser el producto de dos binomios por un trmino comn
porque estrictamente no se completa la factorizacin.
Pgina del Colegio de Matemticas de la ENP-UNAM Ecuaciones y desigualdades Autor: Dr. Jos Manuel Becerra Espinosa
30

Ejemplos.
Obtener las races de las siguientes ecuaciones de segundo grado por factorizacin:
1) 0 8 6
2
= + + x x
( )( ) 0 2 4 = + + x x
4 0 4
1
= = + x x
2 0 2
2
= = + x x
Comprobacin:
( ) ( ) 0 8 24 16 8 4 6 4
2
= + = + +
( ) ( ) 0 8 12 4 8 2 6 2
2
= + = + +

2) 0 6 5
2
= + x x
( )( ) 0 3 2 = x x
2 0 2
1
= = x x
3 0 3
2
= = x x
Comprobacin:
( ) ( ) 0 6 10 4 6 2 5 2
2
= + = +
( ) ( ) 0 6 15 9 6 3 5 3
2
= + = +

3) 0 35 2
2
= + x x
( )( ) 0 5 7 = + x x
7 0 7
1
= = + x x
5 0 5
2
= = x x
Comprobacin:
( ) ( ) 0 35 14 49 35 7 2 7
2
= = +
( ) ( ) 0 35 10 25 35 5 2 5
2
= + = +

4) 0 12
2
= x x
( )( ) 0 3 4 = + x x
4 0 4
1
= = x x
3 0 3
2
= = + x x
Comprobacin:
( ) ( ) 0 12 4 16 12 4 4
2
= =
( ) ( ) 0 12 3 9 12 3 3
2
= + =

5) 0 52 9
2
= x x
( )( ) 0 4 13 = + x x
13 0 13
1
= = x x
4 0 4
2
= = + x x
Comprobacin:
( ) ( ) 0 52 117 169 52 13 9 13
2
= =
( ) ( ) 0 52 36 16 52 4 9 4
2
= + =
Pgina del Colegio de Matemticas de la ENP-UNAM Ecuaciones y desigualdades Autor: Dr. Jos Manuel Becerra Espinosa
31

6) 0 2 3 2
2
= + x x
( ) ( ) ( ) ( ) 0 2 2 2 3 2 2 2
2
= + x x
( ) ( ) 0 4 2 3 2
2
= + x x
( )( ) 0 1 2 4 2 = + x x

2
1
1 2 0 1 2
2
= = = x x x
Comprobacin:
( ) ( ) ( ) 0 2 6 8 2 6 4 2 2 2 3 2 2
2
= = = +
0
2
4
2
3
2
1
4
4
2
3
4
1
2 2
2
1
3
2
1
2
2
= + = +
|

\
|
=
|

\
|
+
|

\
|


7) 0 10 17 3
2
= + x x
( ) ( ) ( ) ( ) 0 3 10 3 17 3 3 3
2
= + x x
( ) ( ) 0 30 3 17 3
2
= + x x
( )( ) 0 2 3 15 3 = x x
5
3
15
15 3 0 15 3
1
= = = = x x x
3
2
2 3 0 2 3
2
= = = x x x
Comprobacin:
( ) ( ) ( ) 0 10 85 75 10 85 25 3 10 5 17 5 3
2
= + = + = +
0
3
30
3
34
3
4
10
3
34
9
4
3 10
3
2
17
3
2
3
2
= + = +
|

\
|
= +
|

\
|

\
|


8) 0 8 4 4
2
= x x
( ) ( ) ( ) ( ) 0 4 8 4 4 4 4 4
2
= x x
( ) ( ) 0 32 4 4 4
2
= x x
( )( ) 0 4 4 8 4 = + x x
2
4
8
8 4 0 8 4
1
= = = = x x x
1
4
4
4 4 0 4 4
2
= = = = + x x x
Comprobacin:
( ) ( ) ( ) 0 8 8 16 8 8 4 4 8 2 4 2 4
2
= = =
( ) ( ) ( ) 0 8 4 4 8 4 1 4 8 1 4 1 4
2
= + = + =

9) 0 15 10 5
2
= + x x
( ) ( ) ( ) ( ) 0 5 15 5 10 5 5 5
2
= + x x
2
2
4
4 2 0 4 2
1
= = = = + x x x
Pgina del Colegio de Matemticas de la ENP-UNAM Ecuaciones y desigualdades Autor: Dr. Jos Manuel Becerra Espinosa
32

( ) ( ) 0 75 5 10 5
2
= + x x
( )( ) 0 5 5 15 5 = + x x
3
5
15
15 5 0 15 5
1
= = = = + x x x
1
5
5
5 5 0 5 5
2
= = = = x x x
Comprobacin:
( ) ( ) ( ) 0 15 30 45 15 30 9 5 15 3 10 3 5
2
= = = +
( ) ( ) ( ) 0 15 10 5 15 10 1 5 15 1 10 1 5
2
= + = + = +

10) 0 36 15 6
2
= + x x
( ) ( ) ( ) ( ) 0 6 36 6 15 6 6 6
2
= + x x
( ) ( ) 0 216 6 15 6
2
= + x x
( )( ) 0 9 6 24 6 = + x x
4
6
24
24 6 0 24 6
1
= = = = + x x x
2
3
6
9
9 6 0 9 6
2
= = = = x x x
Comprobacin:
( ) ( ) ( ) 0 36 60 96 36 60 16 6 36 4 15 4 6
2
= = = +
0
2
72
2
45
2
27
36
2
45
4
9
6 36
2
3
15
2
3
6
2
= + = +
|

\
|
=
|

\
|
+
|

\
|



VII.4.4 PROBLEMAS DE APLICACIN

Al momento de plantear un problema que se modele como una ecuacin de segundo grado, al resolverla
se deben aceptar slo los valores de la incgnita que cumplan las condiciones del problema y rechazar
los que no los cumplan.

1) La suma de dos nmeros es y su producto 204, cules son los nmeros?
Solucin.
El primer nmero es: x
El segundo nmero es: x 29
( ) 204 29 = x x
0 204 29 204 29
2 2
= + = x x x x
( )( ) 0 17 12 = x x
12 0 12
1
= = x x
17 0 17
2
= = x x

2) Hallar tres nmeros impares consecutivos positivos, tales que si al cuadrado del mayor se le restan los
cuadrados de los otros dos se obtiene como resultado 7 .
Solucin.

29
Pgina del Colegio de Matemticas de la ENP-UNAM Ecuaciones y desigualdades Autor: Dr. Jos Manuel Becerra Espinosa
33

x
2 + x
2
24 cm rea =
El primer nmero impar es: x
El segundo nmero impar es: 2 + x
El tercer nmero impar es: 4 + x
( ) ( ) 7 2 4
2 2 2
= + + x x x
( ) 7 4 4 16 8
2 2 2
= + + + + x x x x x
7 4 4 16 8
2 2 2
= + + x x x x x
0 5 4 0 5 4
2 2
= = + + x x x x
( )( ) 0 1 5 = + x x

1 0 1
2
= = + x x
Se rechaza la segunda raz por ser negativa.
Los nmeros son: 4 5 2 5 5 + + , , , es decir:

3) La edad de un padre es el cuadrado de la de su hijo. Dentro de 24 aos la edad del padre ser el
doble de la del hijo. Cuntos aos tiene ahora cada uno?
Solucin.
La edad del hijo es:
La edad del padre es:
2
x
( ) 24 2 24
2
+ = + x x
48 2 24
2
+ = + x x
0 24 2
2
= x x
( )( ) 0 4 6 = + x x
6 0 6
1
= = x x
4 0 4
2
= = + x x
Se rechaza la segunda raz por ser negativa.
36
2
= x , por lo tanto, la edad del hijo es seis aos y la del padre .

4) Un tringulo tiene un rea de 24 cm
2
y la altura mide 2 cm. ms que la base correspondiente.
Cunto miden la base y la altura?
Solucin.
La longitud de la base: x

La longitud de la altura: 2 + x
El rea del tringulo es:
( )
24
2
2
=
+ x x

( ) 48 2 = + x x
48 2
2
= + x x
0 48 2
2
= + x x
( )( ) 0 6 8 = + x x
8 0 8
1
= = + x x
6 0 6
2
= = x x
Se rechaza la primera raz por ser negativa.
La longitud de la altura es: 8 2 6 2 = + = + x
Por lo tanto, la base mide 6 cm. y la altura mide 8 cm.

5 0 5
1
= = x x
9 7 5 , ,
x
36
Pgina del Colegio de Matemticas de la ENP-UNAM Ecuaciones y desigualdades Autor: Dr. Jos Manuel Becerra Espinosa
34

5) Una persona tiene 52 aos de edad y su nieto 2 . Despus de cuntos aos la razn entre la edad
del abuelo y del nieto ser igual a los tres cuartos del tiempo transcurrido para que eso suceda?
Solucin.
El tiempo transcurrido es: x
La edad del nieto despus de x aos es: x + 2
La edad del abuelo despus de x aos es: x + 52
x
x
x
4
3
2
52
=
+
+

( ) x x x + = + 2
4
3
52
( ) ( )
|

\
|
+ = + x x x 2
4
3
4 52 4
2
3 6 4 208 x x x + = +
0 208 2 3
2
= + x x
208 2 3 = = = c , b , a
Sustituyendo en la frmula general se tiene:
( )( )
( ) 6
50 2
6
2500 2
6
2496 4 2
3 2
208 3 4 2 2
2

=

=
+
=

= x
8
6
48
6
50 2
1
= =
+
= x
3
26
6
52
6
50 2
2
=

=

= x
Se rechaza la segunda raz por ser negativa.
Por lo tanto, el tiempo que deber transcurrir sern 8 aos.

6) Un conjunto de personas alquil un microbs en 200 1, pesos. Como tres personas no fueron, las
dems debieron pagar 20 pesos ms de lo acordado. Cuntas viajaban originalmente?
Solucin.
El nmero de personas es: x
Cada persona debi pagar originalmente:
x
,200 1
pesos.
( ) 200 1 20
200 1
3 ,
x
,
x =
|

\
|
+
( ) ( ) =
|

\
|
+ 200 1 20
200 1
3 , x
x
,
x x ( )( ) x , x , x 200 1 20 200 1 3 = +
= + x , x , x x , 200 1 60 600 3 20 200 1
2
0 600 3 60 20
2
= , x x
0 180 3
2
= x x
( )( ) 0 12 15 = + x x
15 0 15
1
= = x x
12 0 12
2
= = + x x
Se rechaza la segunda raz por ser negativa, se tiene que originalmente viajaban 15 personas.


Pgina del Colegio de Matemticas de la ENP-UNAM Ecuaciones y desigualdades Autor: Dr. Jos Manuel Becerra Espinosa
35

x
2 + x
1 + x
7) Calcular la hipotenusa de un tringulo rectngulo, sabiendo que las medidas de sus lados son tres
nmeros consecutivos. Las medidas estn en cm.
Solucin.
El cateto menor es: x
El cateto mayor es: 1 + x
La hipotenusa es: 2 + x
Aplicando el teorema de Pitgoras:
( ) ( )
2 2 2
2 1 + = + + x x x
4 4 1 2
2 2 2
+ + = + + + x x x x x
0 3 2
2
= x x
( )( ) 0 1 3 = + x x
3 0 3
1
= = x x
1 0 1
2
= = + x x
Se rechaza la segunda raz por ser negativa.
4 1 3 1 = + = + x , 5 2 3 2 = + = + x
Las longitudes de los catetos son: 3 cm. y 4 cm., la longitud de la hipotenusa es 5 cm.

8) La diferencia de dos nmeros naturales es 7 y su suma multiplicada por el nmero menor es 184.
Hallar los nmeros.
Solucin.
El nmero menor es x
El nmero mayor es x + 7
( ) 184 7 = + + x x x
184 7
2 2
= + + x x x
0 184 7 2
2
= + x x
184 7 2 = = = c , b , a
Sustituyendo en la frmula general se tiene:
( )( )
( ) 4
39 7
4
521 1 7
4
472 1 49 7
2 2
184 2 4 7 7
2

=

=
+
=

=
, ,
x
8
4
32
4
39 7
1
= =
+
= x
2
23
4
46
4
39 7
2
=

=

= x
Se rechaza la segunda raz por ser negativa.
15 8 7 7 = + = + x
Por lo tanto, los nmeros son 8 y 15.

9) Los tiempos empleados por dos pintores para pintar cada uno un metro cuadrado difieren entre s en
un minuto. Trabajando conjuntamente emplean una hora en pintar 27 metros cuadrados. En cunto
tiempo pinta cada uno un metro cuadrado?
Solucin.
El nmero de minutos que necesita el pintor ms rpido para pintar un metro cuadrado es: x
El nmero de minutos empleados por el otro pintor es: 1 + x
La fraccin de metro cuadrado que pinta el ms rpido en un minuto es:
x
1

Pgina del Colegio de Matemticas de la ENP-UNAM Ecuaciones y desigualdades Autor: Dr. Jos Manuel Becerra Espinosa
36

La fraccin de metro cuadrado que pinta el otro en un minuto es:
1
1
+ x

La fraccin de metro cuadrado que pintan entre los dos en un minuto es:
1
1 1
+
+
x x

Trabajando juntos pintan 27 metros cuadrados en una hora, as que en un minuto pintan:
2
60
27
m
Por tanto:
60
27
1
1 1
=
+
+
x x

( ) ( )
60
27
1 60
1
1 1
1 60 + =
|

\
|
+
+ + x x
x x
x x
( ) ( ) 1 27 60 1 60 + = + + x x x x
x x x x 27 27 60 60 60
2
+ = + +
0 60 93 27
2
= x x
60 93 27 = = = c , b , a
Sustituyendo en la frmula general se tiene:
( ) ( ) ( )( )
( ) 54
123 93
54
129 15 93
54
480 6 649 8 93
27 2
60 27 4 93 93
2

=
+
=

=
, , ,
x
4
54
216
54
123 93
1
= =
+
= x
9
5
54
30
54
123 93
2
=

= x
Se rechaza la segunda raz por ser negativa.
5 1 4 1 = + = + x , as que los pintores emplean 4 y 5 minutos, respectivamente para pintar un metro cuadrado.


VII.4.5 GRFICA DE UNA ECUACIN DE SEGUNDO GRADO

Para graficar una ecuacin de segundo grado, se establece la ecuacin c bx ax y + + =
2
. La solucin de
0
2
= + + c bx ax son los valores x que hacen 0 = y , es decir los puntos
|
|

\
|
+
0
2
4
2
,
a
ac b b
y
|
|

\
|

0
2
4
2
,
a
ac b b
donde la curva c bx ax y + + =
2
cruza el eje x .

El resultado grfico siempre es una curva que recibe el nombre de parbola, cuyas caractersticas son:

1) Si 0 > a , la parbola se abre hacia arriba:
2) Si 0 < a , la parbola se abre hacia abajo:
3) La interseccin con el eje y es el punto ( ) c , 0
4) Como las soluciones dependen del signo del discriminante ac b 4
2
= , se tiene que:
Si 0 > , la ecuacin tiene soluciones reales y distintas, por lo tanto la parbola corta en dos
puntos al eje x .
Si 0 = , la ecuacin tiene soluciones reales iguales, por lo tanto la parbola es tangente al eje x .
Si 0 < , la ecuacin no tiene soluciones reales, por lo tanto la parbola no corta el eje x .
Pgina del Colegio de Matemticas de la ENP-UNAM Ecuaciones y desigualdades Autor: Dr. Jos Manuel Becerra Espinosa
37

x 1 4 3 2 5 -1 -2 -3 -4 -5
5
25
35
40
y
20
15
10
30
6 7 -6 -7
45
-10
x 1 4 3 2 5 -1 -2 -3 -4 -5
5
25
35
-15
y
20
15
10
6 -6 -7
-10
-8
La ecuacin c bx ax y + + =
2
puede evaluarse para todo x R y por ello se unen los puntos obtenidos
para obtener sus grficas.

Para fines prcticos, tabulando valores diferentes de x se pueden obtener los valores de y , generando
puntos de coordenadas ( ) y , x que se localizan en el plano coordenado y que al unirse conforman la parbola.

Si las coordenadas de los puntos son grandes puede ser necesario modificar la escala en los ejes x y y
, lo que provoca que las grficas se deformen. Esto significa que su aspecto es diferente al que realmente
tienen.

Ejemplos.
Graficar las siguientes ecuaciones de segundo grado:

1) 6 4 2
2
= x x y
Solucin.

x
y
-4 42
-3 24
-2 10
-1 0
0 -6
1 -8
2 -6
3 0
4 10
5 24
6 42



La parbola se abre hacia arriba y las races de la ecuacin son diferentes:
3
1
= x
1
2
= x


2) 28 3
2
+ = x x y

Solucin.

x
y
-8 -12
-7 0
-6 10
-5 18
-4 24
-3 28
-2 30
-1 30
0 28
1 24
2 18
3 10
4 0
5 -12
Pgina del Colegio de Matemticas de la ENP-UNAM Ecuaciones y desigualdades Autor: Dr. Jos Manuel Becerra Espinosa
38

x
1 4 3 2 5 -1 -2 -3 -4
20
40
50
10
y
35
30
25
6
15
5
45
x
1 4 3 2 5 -1 -2 -3 -4
20
40
50
10
y
35
30
25
6
15
45
-5 -6 -7 -8
La parbola se abre hacia abajo y las races de la ecuacin son diferentes:
7
1
= x
4
2
= x


3) 12 12 3
2
+ = x x y

Solucin.





x
y
-2 48
-1 27
0 12
1 3
2 0
3 3
4 12
5 27
6 48




La parbola se abre hacia arriba y las races de la ecuacin son iguales:
2
1
= x
2
2
= x


4) 6 2
2
+ + = x x y

Solucin.


x
y
-7 41
-6 30
-5 21
-4 14
-3 9
-2 6
-1 5
0 6
1 9
2 14
3 21
4 30
5 41



La parbola se abre hacia arriba y las races de la ecuacin no son reales.
Pgina del Colegio de Matemticas de la ENP-UNAM Ecuaciones y desigualdades Autor: Dr. Jos Manuel Becerra Espinosa
39

VII.5 DESIGUALDADES DE SEGUNDO GRADO EN UNA VARIABLE

Una desigualdad de segundo grado o desigualdad cuadrtica, tiene la forma:

0
2
> + + c bx ax o 0
2
+ + c bx ax o 0
2
< + + c bx ax o 0
2
+ + c bx ax

donde b , a y c son nmeros reales y 0 a . Su solucin generalmente representa un intervalo o la
unin de dos intervalos de nmeros reales.

Para resolver una desigualdad cuadrtica se usan los conceptos de nmero crtico y nmero de prueba.

Un nmero crtico de la desigualdad mencionada es una raz real de la ecuacin cuadrtica
0
2
= + + c bx ax .

Si
1
r y
2
r son nmeros crticos y
2 1
r r < , entonces el polinomio c bx ax + +
2
slo puede cambiar de
signo algebraico en
1
r y
2
r por la tanto el signo ms o menos de c bx ax + +
2
ser constante en cada
uno de los intervalos ( )
1
r , , ( )
2 1
r , r , ( ) , r
2
.

Para determinar si estos intervalos son o no solucin de la inecuacin, se evala con un nmero x de
prueba arbitrario en c bx ax + +
2

para cada intervalo. Los resultados obtenidos sirven para ubicar el
conjunto de soluciones de la desigualdad.

Un procedimiento sistemtico para la resolucin de inecuaciones cuadrticas es el siguiente:

1. Se trasladan todos los trminos de la inecuacin al miembro de la izquierda.
2. Se hallan los nmeros crticos
1
r y
2
r de la ecuacin cuadrtica y se forman los intervalos ( )
1
r , ,
( )
2 1
r , r , ( ) , r
2
.
3. Se prueban con valores de fcil sustitucin localizados en dichos intervalos para determinar cules
son los que satisfacen la desigualdad.

Ejemplos.
Resolver las siguientes inecuaciones:

1) 0 9
2
> x
Solucin.
9
2
= x
9 = x
3 = x
Los nmeros crticos son:
3
1
= r y 3
2
= r
los intervalos solucin pueden ser ( ) 3 , , ( ) 3 3, y ( ) , 3
probando con tres nmeros ubicados en esos intervalos para saber si cumplen la desigualdad :
para 4 = x del intervalo ( ) 3 , se tiene: ( ) 0 7 9 16 9 4
2
> = =
para 0 = x del intervalo ( ) 3 3, se tiene: 0 9 9 0 9 0
2
< = =
para 4 = x del intervalo ( ) , 3 se tiene: ( ) 0 7 9 16 9 4
2
> = =
Los valores que cumplen la desigualdad son el primero y el tercero, por lo que la solucin es:
( ) ( ) , , 3 3 U .
0 9
2
> x
Pgina del Colegio de Matemticas de la ENP-UNAM Ecuaciones y desigualdades Autor: Dr. Jos Manuel Becerra Espinosa
40

La grfica de la parbola se ubica por arriba del eje x en los intervalos solucin de la desigualdad porque
sus ordenadas son mayores que cero:


x 1 4 3 2 5 -1 -2 -3 -4 -5
5
25
35
y
20
15
10
30
6 7 -6 -7
-10
( ] [ ) , , 3 3 U



2) 0 4
2
< x
Solucin.
4
2
= x
2 = x
2 = x
Los nmeros crticos son:
2
1
= r y 2
2
= r
los intervalos solucin pueden ser ( ) 2 , , ( ) 2 2, y ( ) , 2
probando con tres nmeros ubicados en esos intervalos para saber si cumplen la desigualdad 0 4
2
< x :
para 5 = x del intervalo ( ) 2 , se tiene: ( ) 0 21 4 25 4 5
2
> = =
para 0 = x del intervalo ( ) 2 2,

se tiene: 0 4 4 0 4 0
2
< = =
para 5 = x del intervalo ( ) , 2 se tiene: 0 21 4 25 4 5
2
> = =
El valor que cumple la desigualdad es el segundo, por lo que la solucin es: ( ) 2 2, .
La grfica de la parbola se ubica por abajo del eje x en los intervalos solucin de la desigualdad porque
sus ordenadas son menores que cero:

x 1 4 3 2 5 -1 -2 -3 -4 -5
5
25
35
40
y
20
15
10
30
6 7 -6 -7
( ) 2 2,

Pgina del Colegio de Matemticas de la ENP-UNAM Ecuaciones y desigualdades Autor: Dr. Jos Manuel Becerra Espinosa
41

3) x x 10 2
2

Solucin.
0 10 2
2
x x
0 10 2
2
= x x
( ) 0 10 2 = x x
Los nmeros crticos son:
0
1
= r
5
2
10
10 2 0 10 2
2
= = = = r x x
los intervalos solucin pueden ser: ( ] 0 , , [ ] 5 0, y [ ) , 5
probando con tres nmeros ubicados en esos intervalos para saber si cumplen la desigualdad 0 10 2
2
> x x :
para 1 = x del intervalo ( ] 0 , se tiene: ( ) ( ) 0 12 10 2 1 10 1 2
2
> = + =
para 3 = x del intervalo [ ] 5 0, se tiene: ( ) ( ) 0 12 30 18 3 10 3 2
2
< = =
para 6 = x del intervalo [ ) , 5 se tiene: ( ) ( ) 0 12 60 72 6 10 6 2
2
> = =
Los valores que cumplen la desigualdad son el primero y el tercero, por lo que la solucin es:
( ] [ ) , , 5 0 U .
La grfica de la parbola se ubica por arriba del eje x en los intervalos solucin de la desigualdad porque
sus ordenadas son mayores o iguales que cero:


x 1 4 3 2 5 -1 -2 -3 -4 -5
5
25
35
y
20
15
10
30
6 7 -6 -7
-15
( ) ( ) , , 5 0 U




4) x x 12 3
2

Solucin.
0 12 3
2
+ x x
0 12 3
2
= + x x
( ) 0 12 3 = + x x
Los nmeros crticos son:
0
1
= r
4
3
12
12 3 0 12 3
2
=

= = = + r x x
Pgina del Colegio de Matemticas de la ENP-UNAM Ecuaciones y desigualdades Autor: Dr. Jos Manuel Becerra Espinosa
42

los intervalos solucin pueden ser: ( ] 4 , , [ ] 0 4, y [ ) , 0
probando con tres nmeros ubicados en esos intervalos para saber si cumplen la desigualdad :
para 5 = x del intervalo ( ] 4 , se tiene: ( ) ( ) 0 15 60 75 5 12 5 3
2
> = = +
para 2 = x del intervalo [ ] 0 4, se tiene: ( ) ( ) 0 12 24 12 2 12 2 3
2
< = = +
para 1 = x del intervalo [ ) , 0 se tiene: ( ) ( ) 0 15 12 3 1 12 1 3
2
> = + = +
El valor que cumple la desigualdad es el segundo, por lo que la solucin es: [ ] 0 4, .
La grfica de la parbola se ubica por abajo del eje x en los intervalos solucin de la desigualdad porque
sus ordenadas son menores que cero:


x 1 4 3 2 5 -1 -2 -3 -4 -5
5
25
y
20
15
10
30
6 7 -6 -7
-15
[ ] 0 4,



5) x x 2 8
2
<
Solucin.
Trasponiendo trminos: 0 8 2
2
< x x
0 8 2
2
= x x
8 2 1 = = = c , b , a
Sustituyendo en la frmula general se tiene:
( ) ( ) ( )( )
( ) 2
6 2
2
36 2
2
32 4 2
1 2
8 1 4 2 2
2

=
+
=

= x
Los nmeros crticos son:
4
2
8
2
6 2
1
= =
+
= r
2
2
4
2
6 2
2
=

= r
Ntese que la ecuacin tambin puede factorizarse y los nmeros crticos pueden obtenerse ms rpidamente:
( )( ) 0 2 4 = + x x
4 0 4
1
= = r x
2 0 2
2
= = + r x
los intervalos solucin pueden ser: ( ) 2 , , ( ) 4 2, y ( ) , 4


0 12 3
2
< + x x
Pgina del Colegio de Matemticas de la ENP-UNAM Ecuaciones y desigualdades Autor: Dr. Jos Manuel Becerra Espinosa
43

probando con tres nmeros ubicados en esos intervalos para saber si cumplen la desigualdad 0 8 2
2
< x x :
para 3 = x del intervalo ( ) 2 , se tiene: ( ) ( ) 0 7 8 6 9 8 3 2 3
2
> = + =
para 0 = x del intervalo ( ) 4 2, se tiene: ( ) 0 8 8 0 0 8 0 2 0
2
< = + =
para 5 = x del intervalo ( ) , 4 se tiene: ( ) 0 7 8 10 25 8 5 2 5
2
> = =
Por lo tanto, el valor que cumple la desigualdad es el segundo, por lo que la solucin es: ( ) 4 2, .
La grfica de la parbola se ubica por abajo del eje x en los intervalos solucin de la desigualdad porque
sus ordenadas son menores que cero:


x 1 4 3 2 5 -1 -2 -3 -4 -5
5
25
35
y
20
15
10
30
6 7 -6 -7
( ) 4 2,



6) 30 4 2
2
+ x x
Solucin.
Trasponiendo trminos: 0 30 4 2
2
+ x x

Simplificando:
0 15 2
2
+ x x
0 15 2
2
= + x x
( )( ) 0 3 5 = + x x
5 0 5
1
= = + r x
3 0 3
2
= = r x
los intervalos solucin pueden ser ( ] 5 , , [ ] 3 5, y [ ) , 3
probando con tres nmeros ubicados en esos intervalos para saber si cumplen la desigualdad
0 30 4 2
2
+ x x :
para 6 = x del intervalo ( ] 5 , se tiene: ( ) ( ) 0 18 30 24 72 30 6 4 6 2
2
> = = +
para 0 = x del intervalo [ ] 3 5, se tiene: ( ) ( ) 0 30 30 0 0 30 0 4 0 2
2
< = = +
para 4 = x del intervalo [ ) , 3 se tiene: ( ) ( ) 0 18 30 16 32 30 4 4 4 2
2
> = + = +
Los valores que cumplen la desigualdad son el primero y el tercero, por lo que la solucin es:
( ] [ ) , , 3 5 U .
La grfica de la parbola se ubica por ariba del eje x en los intervalos solucin de la desigualdad porque
sus ordenadas son mayores que cero:

Pgina del Colegio de Matemticas de la ENP-UNAM Ecuaciones y desigualdades Autor: Dr. Jos Manuel Becerra Espinosa
44

x 1 4 3 2 5 -1 -2 -3 -4 -5
-10
y
5
10
15
6 7 -6 -7
-30
-25
-15
-20
( ] [ ) , , 3 5 U



7) 4
3
1
6
1
2
< + x x
Solucin.
( ) 4 6
3
1
6
1
6
2
< |

\
|
+ x x
24 2
2
< + x x
Trasponiendo trminos:
0 24 2
2
< + x x
24 2 1 = = = c , b , a
Sustituyendo en la frmula general se tiene:
( ) ( )( )
( ) 1 2
24 1 4 2 2
2

= x

2
96 4 2 +
=

2
100 2
=

2
10 2
=
Los nmeros crticos son:
4
2
8
2
10 2
1
= =
+
= r
6
2
12
2
10 2
2
=

=

= r
los intervalos solucin pueden ser ( ) 6 , , ( ) 4 6, y ( ) , 4
probando con tres nmeros ubicados en esos intervalos para saber si cumplen la desigualdad
0 24 2
2
< + x x :
para 7 = x del intervalo ( ) 6 , se tiene: ( ) ( ) 0 11 24 14 49 24 7 2 7
2
> = = +
para 0 = x del intervalo ( ) 4 6, se tiene: ( ) 0 24 24 0 0 24 0 2 0
2
< = = +
para 5 = x del intervalo ( ) , 4 se tiene: ( ) ( ) 0 11 24 10 25 24 5 2 5
2
> = + = +
Pgina del Colegio de Matemticas de la ENP-UNAM Ecuaciones y desigualdades Autor: Dr. Jos Manuel Becerra Espinosa
45

Por lo tanto, el valor que cumple la desigualdad es el segundo, por lo que la solucin es: ( ) 4 6, .
La grfica de la parbola se ubica por abajo del eje x en los intervalos solucin de la desigualdad porque
sus ordenadas son menores que cero:


x 1 4 3 2 5 -1 -2 -3 -4 -5
-10
20
y
5
10
15
6 7 -6 -7
-15
-20
( ) 4 6,



8) x x x x 2 1 2 5
2 2
+ < +
Solucin.
Trasponiendo trminos: 0 1 4 4
2
< + x x
0 1 4 4
2
= + x x
1 4 4 = = = c , b , a
Sustituyendo en la frmula general se tiene:
( ) ( ) ( )( )
( ) 8
0 4
8
0 4
8
16 16 4
4 2
1 4 4 4 4
2

=

=

= x
Los nmeros crticos son:
2
1
8
4
8
0 4
1
= =
+
= r
2
1
8
4
8
0 4
2
= =

= r
los intervalos solucin pueden ser
|

\
|

2
1
, y
|

\
|
,
2
1

probando con dos nmeros ubicados en esos intervalos para saber si cumplen la desigualdad 0 1 4 4
2
< + x x :
para 0 = x del intervalo
|

\
|

2
1
, se tiene: ( ) ( ) 0 1 0 0 1 0 4 0 4
2
> + = +
para 1 = x del intervalo
|

\
|
,
2
1
se tiene: ( ) ( ) 0 1 1 4 4 1 1 4 1 4
2
> = + = +
Ninguno de los valores que cumplen la desigualdad, por lo que no tiene solucin.

Ntese como la desigualdad 0 1 4 4
2
< + x x se puede expresar como:
( ) ( ) 0 1 2 2 2
2
< + x x


Pgina del Colegio de Matemticas de la ENP-UNAM Ecuaciones y desigualdades Autor: Dr. Jos Manuel Becerra Espinosa
46

Factorizando:
( ) 0 1 2
2
< x
Puesto que el cuadrado de cualquier nmero real siempre es mayor o igual a cero, entonces se
comprueba que esta inecuacin no tiene solucin.
Toda la parbola se localiza por arriba del eje x , por eso no hay solucin:


x 2 1 -1 -2
4
8
2
y
7
6
5
3
3
9
-3
1



9) 5 4 3 1 8 6
2 2
+ + < + + x x x x

Solucin.
Trasponiendo trminos: 0 4 4 3
2
< + x x
Convirtiendo esta desigualdad a un trinomio cuadrado perfecto, se tiene:
4
3
4
3 4 4 3 0 4 4 3 0 4 4 3
2 2 2 2
> |

\
|
> + > + < + x x x x x x x x
9
8
3
2
3
8
3
2
3
3
4
4
9
4
3
4
3
2 2
2
> |

\
|
> |

\
|
+ > |

\
|
+ x x x x
Puesto que el cuadrado de cualquier nmero real siempre es mayor o igual a cero, entonces se trata de
una desigualdad absoluta.
Toda la parbola se localiza por abajo del eje x y su solucin es cualquier nmero real:


-30
-10
y
-15
-20
-25
-45
x 1 4 3 2 5 -1 -2 -3 -4 -5 6 -6 -7
-35
-40

Pgina del Colegio de Matemticas de la ENP-UNAM Sistemas de ecuaciones y de desigualdades Autor: Dr. Jos Manuel Becerra Espinosa
1

SISTEMAS DE ECUACIONES Y DE DESIGUALDADES

UNIDAD VIII


VIII.1 SISTEMAS DE ECUACIONES

Una ecuacin lineal con dos incgnitas x y y es una expresin de la forma c by ax = + , donde
c , b , a R y a y b son diferentes de cero.

Toda ecuacin lineal con dos incgnitas tiene un nmero ilimitado de soluciones de la forma ( ) y , x y su
grfica determina una recta.

Ejemplos.

1) La ecuacin lineal 20 4 2 = + y x tiene entre sus ilimitadas soluciones a los valores: ( ) 6 2, , ( ) 5 0, ,
( ) 1 8, y ( ) 1 12 ,

2) La ecuacin lineal 15 3 = y x tiene entre sus ilimitadas soluciones a los valores: ( ) 0 5, , ( ) 9 2, ,
( ) 18 1, y ( ) 6 3,

Un sistema de ecuaciones es un conjunto de ecuaciones que poseen incgnitas. Para indicar que varias
ecuaciones forman un sistema, se abarca el conjunto de todas ellas con una llave.

Un sistema de dos ecuaciones lineales con incgnitas x y y , tambin llamado ecuaciones simultneas
de dos por dos es de la forma:

)
`

= +
= +
2 22 21
1 12 11
b y a x a
b y a x a


donde
22 21 12 11
a , a , a , a son coeficientes reales y
2 1
b , b son trminos independientes. En cada una de
las ecuaciones, por lo menos uno de los coeficientes de las incgnitas es diferente de cero.

Los sistemas de dos ecuaciones lineales con dos incgnitas que surgen del planteamiento de un
problema, generalmente no tienen la forma estndar, sin embargo, debe obtenerse.

Resolver un sistema de este tipo es encontrar los pares de nmeros x y y que satisfacen ambas
ecuaciones, si existen. Grficamente, una solucin del sistema es un punto comn a ambas rectas
( ) y , x P .

En un sistema de dos ecuaciones lineales:

Si las dos rectas que se cruzan en un punto, ste representa la solucin del sistema. En este caso el
sistema es compatible determinado.
Si las dos rectas coinciden en todos sus puntos, tiene infinitas soluciones. En este caso el sistema es
compatible indeterminado.
Si las dos rectas son paralelas, no tienen ningn punto comn. En este caso el sistema es
incompatible y no tiene solucin.
Pgina del Colegio de Matemticas de la ENP-UNAM Sistemas de ecuaciones y de desigualdades Autor: Dr. Jos Manuel Becerra Espinosa
2


VIII.2 MTODOS DE SOLUCIN DE SISTEMAS DE DOS ECUACIONES Y DOS
INCGNITAS

Existen cinco mtodos para resolver sistemas de ecuaciones:

Igualacin
Suma y resta (eliminacin)
Sustitucin
Determinantes
Grfico


VIII.2.1 MTODO DE IGUALACIN

El mtodo de igualacin consiste en realizar los siguientes pasos:

Se despeja la misma incgnita en las dos ecuaciones.
Se igualan las expresiones despejadas y se obtiene una ecuacin lineal para la otra incgnita.
Se resuelve la ecuacin lineal.
Se sustituye este valor en cualquiera de las dos expresiones despejadas a fin de obtener el valor de
la otra.
Se realiza la comprobacin.

Ejemplos.
Aplicando el mtodo de igualacin, resolver los siguientes sistemas de ecuaciones:

1)
)
`

= +
=
14 5 3
10 2 4
y x
y x

Solucin.
De la primera ecuacin se despeja x :
2
5
4
2 10 y y
x
+
=
+
=
de la segunda ecuacin tambin se despeja x :
3
5 14 y
x

=
se igualan estas dos ltimas ecuaciones:
3
5 14
2
5 y y
=
+

resolviendo para y :
( ) ( ) y y 5 14 2 5 3 = +
y y 10 28 3 15 = +
15 28 10 3 = + y y
1
13
13
13 13 = = = y y
sustituyendo en la primera ecuacin despejada, se obtiene el valor de la otra incgnita:
3
2
6
2
1 5
= =
+
= x
Por lo tanto: 3 = x y 1 = y . Comprobacin:
( ) ( )
( ) ( )
)
`

= + = +
= =
14 5 9 1 5 3 3
10 2 12 1 2 3 4


Pgina del Colegio de Matemticas de la ENP-UNAM Sistemas de ecuaciones y de desigualdades Autor: Dr. Jos Manuel Becerra Espinosa
3

2)
)
`

= +
=
48 8 2
18 3 9
y x
y x

Solucin.
De la primera ecuacin se despeja x :
3
6
9
3 18 y y
x
+
=
+
=
de la segunda ecuacin tambin se despeja x : y
y
x 4 24
2
8 48
=

=
se igualan estas dos ltimas ecuaciones: y
y
4 24
3
6
=
+

resolviendo para y :
( ) ( ) = + y y 4 24 3 6 2 = + y y 12 72 2 12 12 72 12 2 = + y y
6
14
84
84 14 =

= = y y
sustituyendo en la primera ecuacin despejada, se obtiene el valor de la otra incgnita:
( )
0
3
0
3
6 6
= =
+
= x
Por lo tanto: 0 = x y 6 = y . Comprobacin:
( ) ( )
( ) ( )
)
`

= = +
= + =
48 48 0 6 8 0 2
18 18 0 6 3 0 9


3)

+
=
+

=
+

10
18
7
2 3
3
5 2
9
1 4
x y
y
y x
x

Solucin.
La primera ecuacin, se multiplica por 9 :
14 6 5 15 6 1 4 9
3
5 2
9
9
1 4
9 = =
|

\
|
=
|

\
| +
y x y x x
y x
x
la segunda ecuacin, se multiplica por 70 :
146 40 7 126 7 20 30 70
10
18
70
7
2 3
70 = + + =
|

\
| +
=
|

\
| +
y x x y y
x y
y
el sistema se convierte a su forma estndar:
)
`

= +
=
146 40 7
14 6 5
y x
y x

de la primera ecuacin se despeja y :
6
5 14


=
x
y
de la segunda ecuacin tambin se despeja y :
40
7 146 x
y
+
=
se igualan estas dos ltimas ecuaciones:
40
7 146
6
5 14 x x +
=



resolviendo para x :
( ) ( ) + = x x 7 146 6 5 14 40 = x x 42 876 200 560 560 876 42 200 + = + x x
2
158
316
316 158 =

= = x x
Pgina del Colegio de Matemticas de la ENP-UNAM Sistemas de ecuaciones y de desigualdades Autor: Dr. Jos Manuel Becerra Espinosa
4

sustituyendo en la primera ecuacin despejada, se obtiene el valor de la otra incgnita:
( )
4
40
160
40
14 146
40
2 7 146
= =
+
=
+
= y
Por lo tanto: 2 = x y 4 = y . Comprobacin:
( )
1 1 2
9
9
2
9
1 8
2
9
1 2 4
2 = = =
+
=
+

( )
1
3
3
3
5 8
3
5 4 2
= =


1 1
( )
2 2 4
7
14
4
7
2 12
4
7
2 4 3
4 = = =
+
=
+

2
10
20
10
18 2
10
18 2
= =
+
=
+

2 2


VIII.2.2 MTODO DE SUMA Y RESTA (ELIMINACIN)

El mtodo de suma y resta, tambin llamado de eliminacin consiste en efectuar el procedimiento siguiente:

Se multiplica cada ecuacin por constantes de modo que los coeficientes de la variable a eliminar
resulten iguales en valor absoluto pero con signos opuestos.
Se suman ambas ecuaciones para obtener una nueva ecuacin en trminos solamente de la otra variable.
Se resuelve la ecuacin lineal.
Se despeja la otra variable de cualquiera de las ecuaciones del sistema.
Se sustituye el valor obtenido en la expresin despejada para obtener el valor de la otra.
Se realiza la comprobacin.

Ejemplos.
Resolver los siguientes sistemas de ecuaciones por el mtodo de eliminacin:
1)
)
`

= +
=
13 4 5
2 2 4
y x
y x

Solucin.
Se multiplica la primera ecuacin por 2 y se suma a la segunda:
9 3
13 4 5
4 4 8
=
)
`

= +
=
x
y x
y x

3
3
9
=

= x
de la primera ecuacin se despeja la otra incgnita y se sustituye el valor obtenido:
( ) 7 6 1 3 2 1 2 1
2
4 2
= = + = + =

= x
x
y
Por lo tanto: 3 = x y 7 = y . Comprobacin:
( ) ( )
( ) ( )
)
`

= = +
= + =
13 28 15 7 4 3 5
2 14 12 7 2 3 4


2)
)
`

= +
= +
16 7 5
20 14 8
y x
y x

Solucin.
Pgina del Colegio de Matemticas de la ENP-UNAM Sistemas de ecuaciones y de desigualdades Autor: Dr. Jos Manuel Becerra Espinosa
5

Se multiplica la segunda ecuacin por 2 y se suma a la primera:
12 2
32 14 10
20 14 8
=
)
`

=
= +
x
y x
y x

6
2
12
= = x
de la primera ecuacin se despeja la otra incgnita y se sustituye el valor obtenido:
( )
2
7
14
7
24 10
7
6 4 10
7
4 10
14
8 20
= =
+
=
+
=
+
=
+
=
x x
y
Por lo tanto: 6 = x y 2 = y . Comprobacin:
( ) ( )
( ) ( )
)
`

= + = +
= + = +
16 14 30 2 7 6 5
20 28 48 2 14 6 8


3)
)
`

= +
=
98 2 15
139 9 5
y x
y x

Solucin.
Se multiplica la primera ecuacin por 3 y se suma a la segunda:
319 29
98 2 15
417 27 15
=
)
`

= +
= +
y
y x
y x

11
29
319
=

= y
de la primera ecuacin se despeja la otra incgnita y se sustituye el valor obtenido:
( )
8
5
40
5
99 139
5
11 9 139
5
9 139
= =

=
+
=
+
=
y
x
Por lo tanto: 8 = x y 11 = y . Comprobacin:
( ) ( )
( ) ( )
)
`

= = +
= + =
98 22 120 11 2 8 15
139 99 40 11 9 8 5



VIII.2.3 MTODO DE SUSTITUCIN

El mtodo de sustitucin consiste en efectuar los siguientes pasos:

Despejar una de las incgnitas de una de las ecuaciones.
Sustituir la expresin despejada en la otra ecuacin.
Se resuelve la ecuacin lineal, generalmente fraccionaria.
Se sustituye este valor en la expresin despeja a fin de obtener el valor de la otra.
Se realiza la comprobacin.

Ejemplos.
Mediante el mtodo de sustitucin, resolver los siguientes sistemas de ecuaciones:

1)
)
`

= +
= +
12 2 4
17 7 9
y x
y x

Solucin.
De la primera ecuacin se despeja x :
9
7 17 y
x

=
Pgina del Colegio de Matemticas de la ENP-UNAM Sistemas de ecuaciones y de desigualdades Autor: Dr. Jos Manuel Becerra Espinosa
6

se sustituye en la segunda ecuacin: 12 2
9
7 17
4 = +
|

\
|
y
y

multiplicando por 9 : ( ) ( ) 108 18 7 17 4 12 9 2
9
7 17
4 9 = + =
(

+ |

\
|
y y y
y

40 10 68 108 18 28 108 18 28 68 = + = + = + y y y y y
4
10
40
=

= y
sustituyendo en la ecuacin despejada:
( )
5
9
45
9
28 17
9
4 7 17
9
7 17
=

=

=

=

=
y
x
Por lo tanto: 5 = x y 4 = y . Comprobacin:
( ) ( )
( ) ( )
)
`

= + = +
= + = +
12 8 20 4 2 5 4
17 28 45 4 7 5 9


2)
)
`

=
= +
31 9 7
9 3 2
y x
y x

Solucin.
De la primera ecuacin se despeja x :
2
3 9

=
y
x
se sustituye en la segunda ecuacin: 31 9
2
3 9
7 =
|

\
|

y
y

multiplicando por 2 : ( ) ( )( ) ( ) 62 18 3 9 7 31 2 9
2
3 9
7 2 = + =
(

\
|

y y y
y

1 3 63 62 18 21 62 18 21 63 = = + = + y y y y y
3
1
3
1
=

= y
sustituyendo en la ecuacin despejada: 4
2
8
2
1 9
2
3
1
3 9
2
3 9
=

\
|

=
y
x
Por lo tanto: 4 = x y
3
1
= y . Comprobacin:
( )
( )

= =
|

\
|

= + = |

\
|
+
31 3 28
3
1
9 4 7
9 1 8
3
1
3 4 2


3)
)
`

= +
= +
13 2 5
34 4 10
y x
y x

Solucin.
De la primera ecuacin se despeja x :
5
2 17
10
4 34 y y
x

=

=
se sustituye en la segunda ecuacin: 13 2
5
2 17
5 = +
|

\
|
y
y

simplificando: ( ) = + + = + 13 2 2 17 13 2 2 17 y y y y 4 4 17 13 2 2 = = + y y y
Pgina del Colegio de Matemticas de la ENP-UNAM Sistemas de ecuaciones y de desigualdades Autor: Dr. Jos Manuel Becerra Espinosa
7

1
4
4
=

= y
sustituyendo en la ecuacin despejada:
( )
3
10
30
10
4 34
10
1 4 34
10
4 34
=

=
+
=

=

=
y
x
Por lo tanto: 3 = x y 1 = y .
Comprobacin:
( ) ( )
( ) ( )
)
`

= = +
= = +
13 2 15 1 2 3 5
34 4 30 1 4 3 10



VIII.2.4 MTODO DE DETERMINANTES

Dado un arreglo de nmeros de la forma:
(

22 21
12 11
a a
a a
, su determinante:

22 21
12 11
a a
a a


denotado por , es el resultado de la operacin:
12 21 22 11
a a a a y representa el producto de nmeros
que conforman su diagonal principal (la que se dirige hacia abajo) menos el producto de nmeros que
conforman su diagonal secundaria (la que se dirige hacia arriba).

Ejemplos.
Calcular los siguientes determinantes:

1) ( ) ( ) 14 6 20 2 3 4 5
4 3
2 5
= = =
2) ( ) ( ) 7 5 12 5 1 6 2
6 1
5 2
= + = =


3) ( )( ) ( )( ) 37 28 9 7 4 1 9
1 4
7 9
= + = =


4) ( ) ( )( ) 4 0 4 0 3 10
5
2
10 3
0
5
2
= + = =



Dado un sistema de la forma:

)
`

= +
= +
2 22 21
1 12 11
b y a x a
b y a x a


El determinante del Sistema es el determinante del arreglo formado por los coeficientes de las incgnitas.
El determinante de la incgnita x es el que se obtiene sustituyendo en el arreglo del sistema la
columna de los coeficientes de la incgnita x por la columna de los trminos independientes.
El determinante de la incgnita y es el que se obtiene sustituyendo en el arreglo del sistema la
columna de los coeficientes de la incgnita y por la columna de los trminos independientes.
Pgina del Colegio de Matemticas de la ENP-UNAM Sistemas de ecuaciones y de desigualdades Autor: Dr. Jos Manuel Becerra Espinosa
8

La Regla de Cramer establece que dado un sistema de ecuaciones lineales cuyos trminos
independientes no son cero, el valor de cada incgnita se obtiene dividiendo el determinante de la
incgnita por el determinante del sistema. Esto es:

22 21
12 11
22 2
12 1
a a
a a
a b
a b
x
x =

=

22 21
12 11
2 21
1 11
a a
a a
b a
b a
y
y =

=

En este mtodo solo interesan los coeficientes numricos incluyendo su signo y, en ambos casos, el
denominador es el mismo.

Ejemplos.
Por medio de determinantes, resolver los siguientes sistemas de ecuaciones:

1)
)
`

= +
=
14 5 4
12 3 2
y x
y x

Solucin.
( ) ( )( )
( ) ( )( )
9
2
18
12 10
42 60
3 4 5 2
3 14 5 12
5 4
3 2
5 14
3 12
=

=


=

= x

( ) ( )( )
( ) ( )( )
10
2
20
12 10
48 28
3 4 5 2
12 4 14 2
5 4
3 2
14 4
12 2
=

+
=


=


= y
Por lo tanto: 9 = x y 10 = y . Comprobacin:
( ) ( )
( ) ( )
)
`

= = +
= + =
14 50 36 10 5 9 4
12 30 18 10 3 9 2


2)
)
`

=
= +
26 5 4
9 2 3
y x
y x

Solucin.
( ) ( )
( )( ) ( )
1
7
7
8 15
52 45
2 4 5 3
2 26 5 9
5 4
2 3
5 26
2 9
=

=


=

= x
Pgina del Colegio de Matemticas de la ENP-UNAM Sistemas de ecuaciones y de desigualdades Autor: Dr. Jos Manuel Becerra Espinosa
9

( )( ) ( )
( )( ) ( )
6
7
42
8 15
36 78
2 4 5 3
9 4 26 3
5 4
2 3
26 4
9 3
=

+
=


=


= y
Por lo tanto: 1 = x y 6 = y . Comprobacin:
( ) ( )
( ) ( )
)
`

= + =
= = +
26 30 4 6 5 1 4
9 12 3 6 2 1 3


3)
)
`

= +
= +
17 16 9
7 4 6
y x
y x

Solucin.
( ) ( )
( )( ) ( )( ) 3
1
132
44
36 96
68 112
4 9 16 6
4 17 16 7
16 9
4 6
16 17
4 7
= =
+

= x

( ) ( )( )
( )( ) ( )( ) 4
5
132
165
36 96
63 102
4 9 16 6
7 9 17 6
16 9
4 6
17 9
7 6
= =
+
+
=


=

= y
Por lo tanto:
3
1
= x y
4
5
= y . Comprobacin:

= + =
|

\
|
+
|

\
|

= + = |

\
|
+ |

\
|
17 20 3
4
5
16
3
1
9
7 5 2
4
5
4
3
1
6


4)
)
`

=
=
14 6 10
8 3 5
y x
y x

Solucin.
( ) ( )
( )( ) ( ) 0
6
30 30
42 48
3 10 6 5
3 14 6 8
6 10
3 5
6 14
3 8

=
+
+
=


=

= x

( ) ( )
( )( ) ( ) 0
10
30 30
80 70
3 10 6 5
8 10 14 5
6 10
3 5
14 10
8 5

=
+

= y

Al no existir divisin por cero, el sistema es incompatible.

Pgina del Colegio de Matemticas de la ENP-UNAM Sistemas de ecuaciones y de desigualdades Autor: Dr. Jos Manuel Becerra Espinosa
10

x
1 4 3 2 5 -1 -2 -3 -4 -5
4
5
y
2
1
-1
-2
3
-3
9 6 3 = y x
10 4 2 = + y x
x
2 1 -1 -2
2
y
1
-1
3 -3
-2
3 2 3 = + y x
9 14 6 = + y x
VIII.2.5 MTODO GRFICO

Como ya se mencion, cada ecuacin lineal de un sistema representa una recta. Esto implica que la
representacin de un sistema de dos ecuaciones lineales con dos incgnitas consiste en un par de rectas
y recurdese que:

Si se cortan, el sistema es compatible determinado y las coordenadas del punto de corte son la
solucin del sistema.
Si las rectas son coincidentes (son la misma recta), el sistema es compatible indeterminado y sus
soluciones son todos los puntos de la recta.
Si las rectas son paralelas, el sistema es incompatible.

Para fines de graficacin conviene despejar de ambas ecuaciones la variable y . Se puede elaborar una
tabla de valores o se ubican los puntos en que cruzan a los ejes coordenados para cada recta, se trazan
y se analiza su comportamiento.

Ejemplos.
Resolver los siguientes sistemas de ecuaciones aplicando el mtodo grfico:

1)
)
`

=
= +
9 6 3
5 2
y x
y x

Solucin
Para la primera ecuacin:
Si 5 2
2
5
5 2 0 . y y x = = = =
Si 5
2
10
10 2 0 = = = = x x y
la recta pasa por los puntos ( ) 5 2 0 . , y ( ) 0 5,
Para la segunda ecuacin:
Si 5 1
6
9
9 6 0 . y y x =

= = =
Si 3
3
9
9 3 0 =

= = = x x y
la recta pasa por los puntos ( ) 5 1 0 . , y ( ) 0 3,

graficando se obtiene que la solucin es el punto de interseccin ( ) y , x , es decir ( ) 2 1,
comprobacin:
( ) ( )
( ) ( )
)
`

= =
= + = +
9 12 3 2 6 1 3
10 8 2 2 4 1 2


2)
)
`

= +
= +
3 2 3
9 14 6
y x
y x

Solucin
Para la primera ecuacin:
Si 6428 0
14
9
9 14 0 . y y x = = =
Si 5 1
2
3
6
9
9 6 0 . x x y = = = = =
Pgina del Colegio de Matemticas de la ENP-UNAM Sistemas de ecuaciones y de desigualdades Autor: Dr. Jos Manuel Becerra Espinosa
11

x
2 1 -1 -2
2
y
1
-1
3 -3
-2
6 3 3 = + y x
10 10 5 = y x
la recta pasa por los puntos ( ) 6428 0 0 . , y ( ) 0 5 1 , .
Para la segunda ecuacin:
Si 5 1
2
3
3 2 0 . y y x =

= = =
Si 1
3
3
3 3 0 =

= = = x x y
la recta pasa por los puntos ( ) 5 1 0 . , y ( ) 0 1,

graficando se obtiene que la solucin es el punto de interseccin ( ) y , x , es decir ( ) 5 1 2 . ,
comprobacin:
( ) ( )
( ) ( )
)
`

= + = +
= + = +
3 3 6 5 1 2 2 3
9 21 12 5 1 14 2 6
.
.


3)
)
`

=
= +
10 10 5
6 3 3
y x
y x

Solucin
Para la primera ecuacin:
Si 2
3
6
6 3 0 = = = = y y x
Si 2
3
6
6 3 0 = = = = x x y
la recta pasa por los puntos ( ) 2 0, y ( ) 0 2,
Para la segunda ecuacin:
Si 1
10
10
10 10 0 =

= = = y y x
Si 2
5
10
10 5 0 = = = = x x y
la recta pasa por los puntos ( ) 1 0 , y ( ) 0 2,

graficando se obtiene que la solucin es el punto de interseccin ( ) y , x , es decir ( ) 0 2,
comprobacin:
( ) ( )
( ) ( )
)
`

= =
= + = +
10 0 10 0 10 2 5
6 0 6 0 3 2 3



VIII.3 PROBLEMAS CON SISTEMAS DE DOS ECUACIONES Y DOS INCGNITAS

Muchos problemas tcnicos y cientficos requieren la resolucin de sistemas de ecuaciones lineales. Se
trata de un tema fundamental para todas las disciplinas que utilizan las matemticas de una manera u
otra. En muchos problemas existe dependencia entre las diferentes magnitudes o variables que
intervienen, y a menudo, se expresa en forma de ecuacin lineal.

Dentro del proceso de resolucin de problemas con sistemas de ecuaciones lineales, se pueden definir
cinco etapas:

Leer el problema
Definir las incgnitas principales de forma precisa
Pgina del Colegio de Matemticas de la ENP-UNAM Sistemas de ecuaciones y de desigualdades Autor: Dr. Jos Manuel Becerra Espinosa
12

Traduccin matemtica del problema para plantearlo
Resolucin
Interpretacin de las soluciones para contrastar la adecuacin de las soluciones obtenidas.

Ejemplos.

1) En una granja, se tienen cien animales entre puercos y gallinas. Si en total suman 240 patas,
cuntos animales tengo de cada clase?
Solucin.
x es el nmero de puercos
y es el nmero de gallinas
como cada puerco tiene cuatro patas y cada gallina dos, el sistema est dado por:

)
`

= +
= +
240 2 4
100
y x
y x
)
`

= +
= +
120 2
100
y x
y x

resolviendo por eliminacin, se multiplica la primera ecuacin por 2 y se suma a la segunda:

80
120 2
200 2 2
=
)
`

= +
=
y
y x
y x

80
1
80
=

= y
de la primera ecuacin se despeja la otra incgnita y se sustituye el valor obtenido:
20 80 100 100 = = = y x
Por lo tanto, hay 20 puercos y 80 gallinas.
Comprobacin:
( ) ( )
)
`

= + = +
= +
240 160 80 80 2 20 4
100 80 20


2) Una cuerda mide doce metros y se corta en dos partes de tal manera que una es dos metros ms
grande que la otra. Cuales son las nuevas medidas de las cuerdas?
Solucin.
x es la longitud del pedazo ms grande
y es la longitud del pedazo ms pequeo
)
`

+ =
= +
2
12
y x
y x

ordenando:
)
`

=
= +
2
12
y x
y x

resolviendo por eliminacin, se suma la primera ecuacin a la segunda:

14 2
2
12
=
)
`

=
= +
x
y x
y x

7
2
14
= = x
de la primera ecuacin se despeja la otra incgnita y se sustituye el valor obtenido:
5 7 12 12 = = = y x y
Pgina del Colegio de Matemticas de la ENP-UNAM Sistemas de ecuaciones y de desigualdades Autor: Dr. Jos Manuel Becerra Espinosa
13

Por lo tanto, los pedazos miden 7 y 5 metros.
Comprobacin:
)
`

=
= +
2 5 7
12 5 7


3) Seis Kg. de piones y cinco Kg. de nueces costaron 270 2, pesos y cinco Kg. de piones y cuatro de
nueces costaron 880 1, pesos. Hallar el precio de un kilogramo de piones y uno de nueces.
Solucin.
x es el precio en pesos de un Kg. de piones
y es el precio en pesos de un Kg. de nueces
)
`

= +
= +
880 1 4 5
270 2 5 6
, y x
, y x

resolviendo por determinantes:
( ) ( )
( ) ( )
320
1
320
25 24
400 9 080 9
5 5 4 6
5 880 1 4 270 2
4 5
5 6
4 880 1
5 270 2
=

= =
, , , , ,
,
x
( ) ( )
( ) ( )
70
1
70
25 24
350 11 280 11
5 5 4 6
270 2 5 880 1 6
4 5
5 6
880 1 5
270 2 6
=

= =
, , , , ,
,
y
Por lo tanto, un Kg. de piones vale 320 pesos y uno de nueces vale 70 pesos.
Comprobacin:
( ) ( )
( ) ( )
)
`

= + = +
= + = +
880 1 280 600 1 70 4 320 5
270 2 350 920 1 70 5 320 6
, ,
, ,


4) Paola tiene 27 aos ms que su hija Andrea. Dentro de 8 aos, la edad de Paola doblar a la de
Andrea. Cuntos aos tiene cada una?
Solucin.
x es la edad de Paola
y es la edad de Andrea
( )
)
`

+ = +
+ =
8 2 8
27
y x
y x

simplificando:

)
`

+ = +
=
16 2 8
27
y x
y x
)
`

=
=
8 2
27
y x
y x

resolviendo por eliminacin, se multiplica la primera ecuacin por 1 y se suma a la segunda:

19
8 2
27
=
)
`

=
= +
y
y x
y x

19
1
19
=

= y
de la primera ecuacin se despeja la otra incgnita y se sustituye el valor obtenido:
Pgina del Colegio de Matemticas de la ENP-UNAM Sistemas de ecuaciones y de desigualdades Autor: Dr. Jos Manuel Becerra Espinosa
14

46 19 27 27 = + = + = y x
Por lo tanto, Paola tiene 46 aos y Andrea tiene 19 aos.
Comprobacin:
( )
)
`

= =
=
8 38 46 19 2 46
27 19 46


5) La diferencia de dos nmeros es 14 , y la cuarta parte de su suma es 13. Hallar los nmeros.
Solucin.
x es el nmero mayor
y es el nmero menor
( )

)

= +
=
13
4
1
14
y x
y x

simplificando:
( )

)
`

= +
=
13 4
14
y x
y x
)
`

= +
=
52
14
y x
y x

resolviendo por eliminacin, se suma la primera ecuacin a la segunda:

66 2
52
14
=
)
`

= +
=
x
y x
y x

33
2
66
= = x
de la primera ecuacin se despeja la otra incgnita y se sustituye el valor obtenido:
19 33 14 14 14 = + = + = = x y x y
Por lo tanto, los nmeros son 33 y 19.
Comprobacin:
)
`

= +
=
52 19 33
14 19 33


6) Si a los dos trminos de una fraccin se aade 3 , el valor de la fraccin es
2
1
, y si a los dos trminos
se resta 1, el valor de la fraccin es
3
1
. Hallar la fraccin.
Solucin.
x es el numerador
y es el denominador
y
x
es la fraccin buscada.

=
+
+
3
1
1
1
2
1
3
3
y
x
y
x

Pgina del Colegio de Matemticas de la ENP-UNAM Sistemas de ecuaciones y de desigualdades Autor: Dr. Jos Manuel Becerra Espinosa
15

simplificando:
( ) ( )
( ) ( )

)
`

=
+ = +
1 1 1 3
3 1 3 2
y x
y x

)
`

=
+ = +
1 3 3
3 6 2
y x
y x
)
`

=
=
2 3
3 2
y x
y x

resolviendo por igualacin, de la primera ecuacin se despeja x :
2
3 y
x
+
=
de la segunda ecuacin tambin se despeja x :
3
2 y
x
+
=
se igualan estas dos ltimas ecuaciones:
3
2
2
3 y y +
=
+

resolviendo para y :
( ) ( ) y y + = + 2 2 3 3
y y 2 4 3 9 + = +
9 4 2 3 + = y y
13 = y
sustituyendo en la primera ecuacin despejada, se obtiene el valor de la otra incgnita:
5
2
10
2
13 3
= =
+
= x
Por lo tanto, la fraccin es
13
5

Comprobacin:

= =

= =
+
+
3
1
12
4
1 13
1 5
2
1
16
8
3 13
3 5


7) El precio del boleto para un concierto es de 225 pesos para pblico en general, y 150 pesos para
estudiantes. La taquilla recaud 775 77, pesos por la venta de 450 boletos. Cuntos boletos de cada
tipo se vendieron?
Solucin.
x es el nmero de boletos vendidos a pblico en general
y es el nmero de boletos vendidos a estudiantes
)
`

= +
= +
775 77 150 225
450
, y x
y x

resolviendo por eliminacin, se multiplica la primera ecuacin por 225 y se suma a la segunda:
475 23 75
775 77 150 225
250 101 225 225
, y
, y x
, y x
=
)
`

= +
=

313
75
475 23
=

=
,
y
de la primera ecuacin se despeja la otra incgnita y se sustituye el valor obtenido:
137 313 450 450 = = = y x
Por lo tanto, se vendieron 137 boletos a pblico en general y 313 a estudiantes.
Comprobacin:
( ) ( )
)
`

= + = +
= +
775 77 950 46 825 30 313 150 137 225
450 313 137
, , ,

Pgina del Colegio de Matemticas de la ENP-UNAM Sistemas de ecuaciones y de desigualdades Autor: Dr. Jos Manuel Becerra Espinosa
16

8) Una llave A tarda en llenar un depsito el doble de tiempo que otra llave B. Abiertas simultneamente,
llenan el depsito en dos horas. Cunto tarda cada una por separado?
Solucin.
x son las horas que tarda la llave A en llenar el depsito, as que en una hora llena
x
1
del depsito
y son las horas que tarda la llave B en llenar el depsito, as que en una hora llena
y
1
del depsito
Las dos llaves tardan dos horas en llenar el depsito, as que en una hora llenan
2
1
del depsito

=
= +
y x
y x
2
2
1 1 1

sustituyendo la segunda ecuacin en la primera se tiene:
2
1 1
2
1
= +
y y

multiplicando por y 2 :
3 2 1
2
1
2
1
2
1
2 = = +
|

\
|
=
|
|

\
|
+ y y y
y y
y
sustituyendo en la segunda ecuacin: ( ) 6 3 2 = = x
Por lo tanto, la llave A llena el depsito en 6 horas y la llave B lo hace en 3 horas.
Comprobacin:
( )

)

=
= =
+
= +
6 3 2
2
1
18
9
18
6 3
3
1
6
1


9) Un bote que navega por un ro recorre 15 kilmetros en hora y media a favor de la corriente y 12
kilmetros en dos horas contra la corriente. Hallar la velocidad del bote en agua tranquila y la velocidad
del ro.
Solucin.
x es la velocidad en Km. por hora del bote en agua tranquila
y es la velocidad en Km. por hora del ro
y x + es la velocidad del bote a favor de la corriente
y x es la velocidad del bote contra la corriente
velocidad
cia tan dis
tiempo
tiempo
cia tan dis
velocidad = =

=
+
2
12
5 1
15
y x
.
y x

simplificando:

)
`

=
+ =
y x
y . x .
2 2 12
5 1 5 1 15
)
`

=
= +
12 2 2
15 5 1 5 1
y x
y . x .

resolviendo por determinantes:
Pgina del Colegio de Matemticas de la ENP-UNAM Sistemas de ecuaciones y de desigualdades Autor: Dr. Jos Manuel Becerra Espinosa
17

( ) ( )
( ) ( )
8
6
48
3 3
18 30
5 1 2 2 5 1
5 1 12 2 15
2 2
5 1 5 1
2 12
5 1 15
=

=


=


=

=
. .
.
. .
.
x
( ) ( )
( ) ( )
2
6
12
3 3
30 18
5 1 2 2 5 1
15 2 12 5 1
2 2
5 1 5 1
12 2
15 5 1
=

=
. .
.
. .
.
y
Por lo tanto, la velocidad del bote en agua tranquila es de
hr
Km
8 y la velocidad del ro es de .
hr
Km
2
Comprobacin:
( ) ( )
( ) ( )
)
`

= =
= + = +
12 4 16 2 2 8 2
15 3 12 2 5 1 8 5 1 . .



VIII.4 SISTEMAS DE DOS INECUACIONES Y DOS INCGNITAS

Un sistema de inecuaciones lineales con una incgnita es el conjunto formado por dos o ms
inecuaciones lineales de la forma:

+
< +
> +
0
0
0
2 2
1 1
n n
c x a
c x a
c x a
L K


o cualquier otro signo de desigualdad, donde
n
a , , a , a L
2 1
son coeficientes reales y
n
c , , c , c L
2 1
son
trminos independientes.

La solucin de un sistema de este tipo es un conjunto de nmeros reales x que satisfagan
simultneamente todas y cada una de las desigualdades. La solucin, en caso de existir, suele
expresarse en forma de intervalo y se debe tener cuidado en expresar correctamente si es abierto o
cerrado segn el signo de desigualdad utilizado.

Particularmente, un sistema de dos inecuaciones lineales con incgnita x , es de la forma:

)
`

< +
> +
0
0
2 2
1 1
c x a
c x a


o cualquier otro signo de desigualdad. Resolver un sistema de este tipo es encontrar el intervalo de
nmeros reales x que satisface ambas inecuaciones, si existe.

Ejemplos.
Resolver los siguientes sistemas de inecuaciones con una incgnita.

Pgina del Colegio de Matemticas de la ENP-UNAM Sistemas de ecuaciones y de desigualdades Autor: Dr. Jos Manuel Becerra Espinosa
18

1)
)
`

+ > +
>
x x
x x
2 34 9 7
3 12 4 5

Solucin.
De la primera inecuacin:
2
8
16
16 8 4 12 3 5 > > > + > + x x x x x
de la segunda inecuacin:
5
5
25
25 5 9 34 2 7 > > > > x x x x x
el conjunto solucin es la interseccin de ambos intervalos que corresponde al intervalo sealado por la
flecha, por lo tanto es 5 > x







2)
)
`

> +
+ <
x x
x x
8 4 5
6 3 23 11

Solucin.
De la primera inecuacin:
4
5
20
20 5 23 3 6 11 < < < + < x x x x x
de la segunda inecuacin:
3
4
12
12 4 4 8 5 <

< > > + x x x x x


el conjunto solucin es la interseccin de ambos intervalos que corresponde al intervalo sealado por la
flecha, por lo tanto es 3 < x






3)
)
`

+ <
+ < + +
2 9 10 6 3
2 7 8 4 9
x x
x x x

x
5 4 1
2 > x
0
5 > x
2 6 3 7
5 > x
x
5 4 1
2 > x
0
5 > x
2 6 3 7
5 > x
x
5 4 1
3 < x
0
4 < x
2 6 3 7
3 < x
x
5 4 1
3 < x
0
4 < x
2 6 3 7
3 < x
Pgina del Colegio de Matemticas de la ENP-UNAM Sistemas de ecuaciones y de desigualdades Autor: Dr. Jos Manuel Becerra Espinosa
19

Solucin.
De la primera inecuacin:
1
3
3
3 3 4 7 2 8 9 >

> < < + x x x x x x


de la segunda inecuacin:
3
6
18
18 6 6 2 10 9 3 < < < + + < + x x x x x
el conjunto solucin es la interseccin de ambos intervalos que corresponde al intervalo sealado por la
flecha, por lo tanto es 3 1 < < x







4)
)
`

+ >
+ + > +
x x
x x
5 4 11 8 2
3 2 15 6 8

Solucin.
De la primera inecuacin:
2
6
12
12 6 6 3 15 2 8 > > > + > x x x x x
de la segunda inecuacin:
1
7
7
7 7 8 4 11 5 2 <

< > + > x x x x x


al no haber interseccin de ambos intervalos, no hay solucin.







5)
)
`

+ +
+
16 12 4 8
7 4 3 9
x x
x x

Solucin.
De la primera inecuacin:
x
3 2 -1
3 < x
-2
1 > x
0 4 1
5
3 1 < < x
x
3 2 -1
3 < x
-2
1 > x
0 4 1
5
3 1 < < x
x
3 2 -1
1 < x
-2
2 > x
0 4 1
5
= Solucin
x
3 2 -1
1 < x
-2
2 > x
0 4 1
5
= Solucin
Pgina del Colegio de Matemticas de la ENP-UNAM Sistemas de ecuaciones y de desigualdades Autor: Dr. Jos Manuel Becerra Espinosa
20

2
5
10
10 5 3 7 4 9 + x x x x x
de la segunda inecuacin:
3
4
12
12 4 4 16 12 8

x x x x x
el conjunto solucin es la interseccin de ambos intervalos que corresponde al intervalo sealado por la
flecha, por lo tanto es 2 3 x







Un sistema de dos inecuaciones lineales con incgnitas x y y , es de la forma:

)
`

< +
> +
2 22 21
1 12 11
b y a x a
b y a x a


o cualquier otro signo de desigualdad, donde
22 21 12 11
a , a , a , a son coeficientes reales y
2 1
b , b son
trminos independientes. En cada una de las inecuaciones, por lo menos uno de los coeficientes de las
incgnitas es diferente de cero. Resolver un sistema de este tipo es obtener el semiplano solucin de las
dos desigualdades e identificar su interseccin.

Obtener la solucin de un sistema de este tipo supone obtener el hiperplano solucin de cada una de las
inecuaciones que lo forman y determinar la interseccin de todos ellos.

La solucin de un sistema de n inecuaciones lineales con dos incgnitas es siempre un conjunto convexo.
Se llama conjunto convexo a una regin del plano tal que para dos puntos cualesquiera de la misma, el
segmento que los une est ntegramente contenido en dicha regin. Como casos particulares, un conjunto
convexo puede quedar reducido a una recta, a una semirrecta, a un segmento, a un punto o al conjunto vaco.

Los segmentos que delimitan un conjunto convexo se llaman bordes o lados y, la interseccin de ellos,
vrtices. Los vrtices y puntos de los lados que pertenezcan a la solucin del sistema de inecuaciones se
denominan puntos extremos. Un conjunto convexo puede ser cerrado o abierto respecto a cada lado o vrtice
segn se incluya ste o no en la solucin. Puede ser acotado o no acotado segn su rea sea o no finita.

Ejemplos.
Resolver los siguientes sistemas de dos inecuaciones lineales con dos incgnitas.

1)
)
`

> +
> +
0 1
0 4 2
y x
y x

Solucin.
Convirtiendo a igualdad la primera inecuacin: 0 4 2 = + y x
x
1 0 -3
2 x
-4
3 x
-2 2 -1 3
2 3 x
x
1 0 -3
2 x
-4
3 x
-2 2 -1 3
2 3 x
Pgina del Colegio de Matemticas de la ENP-UNAM Sistemas de ecuaciones y de desigualdades Autor: Dr. Jos Manuel Becerra Espinosa
21

Si 4 0 4 0 = = = y y x
Si 2
2
4
4 2 0 4 2 0 = = = = = x x x y
la recta pasa por los puntos ( ) 4 0, y ( ) 0 2,
Para representar grficamente la solucin de la primera inecuacin se elige un punto que no est en la
recta y se comprueba si verifica o no la desigualdad. Por ejemplo, tomando el punto ( ) 3 1
1
, P se aprecia
que cumple la inecuacin ya que al sustituir se obtiene ( ) ( ) 0 1 4 3 2 4 3 1 2 > = + = + . Esto significa
que la regin que incluye a ese punto es solucin de esta desigualdad.
Convirtiendo a igualdad la segunda inecuacin: 0 1= + y x
Si 1
1
1
1 0 =

= = = y y x
Si 1 0 1 0 = = + = x x y
la recta pasa por los puntos ( ) 1 0, y ( ) 0 1,
Para representar grficamente la solucin de la segunda inecuacin se elige un punto que no est en la
recta y se comprueba si verifica o no la desigualdad. Por ejemplo, tomando el punto ( ) 2 3
2
, P se observa
que cumple la inecuacin ya que al sustituir se obtiene 0 2 1 2 3 > = + . Esto significa que la regin que
incluye a ese punto es solucin de esta desigualdad.
El conjunto solucin es la interseccin de las dos regiones, formando el semiplano sombreado.



x
1 4 3 2 5 -1 -2 -3 -4 -5
4
5
y
2
1
-1
-2
3
-3
Recta 1
Recta 2
P
1
P
2




2)
)
`

> +
> +
0 10 2
0 3
y x
y x

Solucin.
Convirtiendo a igualdad la primera inecuacin: 0 3 = + y x
Si 3 0 3 0 = = = y y x
Si 3
1
3
3 0 3 0 =

= = = = x x x y
la recta pasa por los puntos ( ) 3 0, y ( ) 0 3,
Pgina del Colegio de Matemticas de la ENP-UNAM Sistemas de ecuaciones y de desigualdades Autor: Dr. Jos Manuel Becerra Espinosa
22

Se elige un punto que no est en la recta para verificar si cumple o no la desigualdad. Por ejemplo,
tomando al punto ( ) 8 3
1
, P se tiene que ( ) 0 8 3 8 3 3 8 3 > = + = + , esto es, cumple la
inecuacin, por lo que la regin que incluye a ese punto es solucin de esta desigualdad.
Convirtiendo a igualdad la segunda inecuacin: 0 10 2 = + y x
Si 5
2
10
10 2 0 = = = = y y x
Si 10 0 10 0 = = = x x y
la recta pasa por los puntos ( ) 5 0, y ( ) 0 10,
Se elige un punto que no est en la recta para verificar si cumple o no la desigualdad. Por ejemplo,
tomando al punto ( ) 6 1
2
, P se tiene que ( ) 0 3 10 12 1 10 6 2 1 > = + = + , esto es, cumple la
inecuacin, por lo que la regin que incluye a ese punto es solucin de esta desigualdad.
El conjunto solucin es la interseccin de las dos regiones, formando el semiplano sombreado.



x
5 10 -5 -10
10
y
-5
5
Recta 2
Recta 1
P
1
P
2




3)
)
`

> +
> +
0 6 3
0 8 2
y x
y x

Solucin.
Convirtiendo a igualdad la primera inecuacin: 0 8 2 = + y x
Si 8 0 8 0 = = = y y x
Si 4
2
8
8 2 0 8 2 0 = = = = = x x x y
la recta pasa por los puntos ( ) 8 0, y ( ) 0 4,
Se elige un punto que no est en la recta para verificar si cumple o no la desigualdad. Por ejemplo,
tomando al punto ( ) 2 1
1
, P se tiene que ( ) 0 4 8 2 1 2 < = + , esto es, no cumple la inecuacin, por lo
que la regin que no incluye a ese punto es solucin de esta desigualdad.
Convirtiendo a igualdad la segunda inecuacin: 0 6 3 = + y x
Si 6
1
6
6 0 6 0 =

= = = + = y y y x
Pgina del Colegio de Matemticas de la ENP-UNAM Sistemas de ecuaciones y de desigualdades Autor: Dr. Jos Manuel Becerra Espinosa
23

Si 2
3
6
6 3 0 6 3 0 =

= = = + = x x x y
la recta pasa por los puntos ( ) 6 0, y ( ) 0 2,
Se elige un punto que no est en la recta para verificar si cumple o no la desigualdad. Por ejemplo,
tomando al punto ( ) 2 4
2
, P se tiene que ( ) 0 8 6 2 12 6 2 4 3 < = + = + , esto es, no cumple la
inecuacin, por lo que la regin que no incluye a ese punto es solucin de esta desigualdad.
El conjunto solucin es la interseccin de las dos regiones, formando el semiplano sombreado.



x
5 10 -5 -10
10
y
-5
5
Recta 2 Recta 1
P
1
P
2




4)
)
`

+
+
15 3 5
10 5 2
y x
y x

Solucin.
Acomodando:
)
`

+
+
0 15 3 5
0 10 5 2
y x
y x

Convirtiendo a igualdad la primera inecuacin: 0 10 5 2 = + y x
Si 2
5
10
10 5 0 10 5 0 = = = = = y y y x
Si 5
2
10
10 2 0 10 2 0 = = = = = x x x y
la recta pasa por los puntos ( ) 2 0, y ( ) 0 5,
Se elige un punto que no est en la recta para verificar si cumple o no la desigualdad. Por ejemplo,
tomando al punto ( ) 2 3
1
, P se tiene que ( ) ( ) 0 6 10 10 6 10 2 5 3 2 > = + = + , esto es, cumple la
inecuacin, por lo que la regin que incluye a ese punto es solucin de esta desigualdad.
Convirtiendo a igualdad la segunda inecuacin: 0 15 3 5 = + y x
Si 5
3
15
15 3 0 15 3 0 = = = = = y y y x
Si 3
5
15
15 5 0 15 5 0 = = = = = x x x y
Pgina del Colegio de Matemticas de la ENP-UNAM Sistemas de ecuaciones y de desigualdades Autor: Dr. Jos Manuel Becerra Espinosa
24

la recta pasa por los puntos ( ) 5 0, y ( ) 0 3,
Se elige un punto que no est en la recta para verificar si cumple o no la desigualdad. Por ejemplo,
tomando al punto ( ) 1 4
2
, P se tiene que ( ) ( ) 0 8 15 3 20 15 1 3 4 5 > = + = + , esto es, no cumple la
inecuacin, por lo que la regin que no incluye a ese punto es solucin de esta desigualdad.
El conjunto solucin es la interseccin de las dos regiones, formando el semiplano sombreado.



x
1 4 3 2 5 -1 -2 -3 -4 -5
4
5
y
2
1
-1
-2
3
-3
Recta 2
Recta 1
P
1
P
2




VIII.5 MTODOS DE SOLUCIN DE SISTEMAS DE TRES ECUACIONES Y TRES
INCGNITAS

Un sistema de tres ecuaciones lineales con incgnitas x , y y z , tambin llamado ecuaciones simultneas
de tres por tres es de la forma:

= + +
= + +
= + +
3 33 32 31
2 23 22 21
1 13 12 11
b z a y a x a
b z a y a x a
b z a y a x a


donde
33 11
a , , a L son coeficientes reales y
3 2 1
b , b , b son trminos independientes. Resolver un
sistema de este tipo es encontrar la terna de nmeros y , x y z que satisfacen las tres ecuaciones, si
existen.


Aqu se expondrn dos mtodos para resolver sistemas de ecuaciones:

Reduccin (mtodo de eliminacin de Gauss)
Determinantes (Regla de Cramer)


VIII.5.1 MTODO DE ELIMINACIN DE GAUSS

El mtodo reduccin para la resolucin de sistemas lineales es una generalizacin del mtodo de
eliminacin expuesto en el subtema VIII.2.2 y es aplicable a sistemas lineales de cualquier tamao. En
Pgina del Colegio de Matemticas de la ENP-UNAM Sistemas de ecuaciones y de desigualdades Autor: Dr. Jos Manuel Becerra Espinosa
25

esencia consiste en hacer, al sistema de ecuaciones lineales, determinadas transformaciones
elementales a fin de obtener un sistema escalonado (un sistema es escalonado cuando cada ecuacin
tiene una incgnita menos que la anterior), ms fcil de resolver.

La idea del mtodo es muy simple: ir reduciendo en cada paso el problema a un problema que tiene una
ecuacin menos y una incgnita menos. Este mtodo es mejor conocido como mtodo de eliminacin de
Gauss
1
.

El procedimiento es el siguiente:

1. Tomando como base el signo de una de las incgnitas de una ecuacin, se procura que en las otras
dos ecuaciones esa incgnita tenga la misma magnitud y signo contrario, para que al sumarlas miembro
a miembro se elimine dicha incgnita, dando lugar a que en todas las ecuaciones desaparezca, excepto
en una.

2. Se procura que otra de las incgnitas tenga el mismo coeficiente en cualquiera de las dos ecuaciones
reducidas para que, al sumarlas miembro a miembro, se elimine dicha incgnita, dando lugar a una
ecuacin con slo la tercera incgnita, misma que se despeja.

3. Con un valor conocido, se sustituye en la ecuacin reducida para obtener el valor de otra incgnita a
travs de un despeje.

4. Con los valores de dos incgnitas se sustituye en la ecuacin que no fue reducida, y mediante un
despeje se obtiene el valor faltante.

Ejemplo.
Resolver los siguientes sistemas aplicando el mtodo de eliminacin de Gauss.

1)

=
= +
= +
12 3 2 6
3 2 5 4
13 5 3 2
z y x
z y x
z y x

Solucin.
La primera ecuacin se multiplica por 2 y se suma a la segunda. La primera ecuacin se multiplica por
3 y se suma a la tercera:

=
= +
= +
51 18 7
29 8
13 5 3 2
z y
z y
z y x

la segunda ecuacin se multiplica por 7 y se suma a la tercera:

=
= +
= +
152 38
29 8
13 5 3 2
z
z y
z y x

de la tercera ecuacin se despeja z : 4
38
152
= = z
se sustituye este valor en la segunda ecuacin y se despeja y :


1
El nombre es un reconocimiento al matemtico Carl Friedrich Gauss (1777-1855) quien desarroll el mtodo.

Pgina del Colegio de Matemticas de la ENP-UNAM Sistemas de ecuaciones y de desigualdades Autor: Dr. Jos Manuel Becerra Espinosa
26

( ) 3
1
3
3 32 29 29 32 29 4 8 =

= = = = + = + y y y y
estos valores, se sustituyen en la primera ecuacin y se despeja x :
( ) ( ) 1
2
2
2 20 9 13 2 13 20 9 2 13 4 5 3 3 2 =

= = + = = + = + x x x x
Por lo tanto la solucin del sistema es: 4 3 1 = = = z , y , x
Comprobacin:
( ) ( ) ( )
( ) ( ) ( )
( ) ( ) ( )

)

= =
= + = +
= + = +
12 12 6 6 4 3 3 2 1 6
3 8 15 4 4 2 3 5 1 4
13 20 9 2 4 5 3 3 1 2


2)

= +
= +
= +
1 2
1 2 2
6 2
z y x
z y x
z y x

Solucin.
La primera ecuacin se multiplica por 2 y se suma a la segunda. La primera ecuacin se multiplica por
1 y se suma a la tercera:

= +
= +
= +
7
11 2
6 2
z y
z y
z y x

la tercera ecuacin se multiplica por 2 y se suma a la segunda:

= +
=
= +
7
3 3
6 2
z y
z
z y x

de la segunda ecuacin se despeja z : 1
3
3
= = z
se sustituye este valor en la tercera ecuacin y se despeja y :
6 1 7 7 1 = = = + y y
estos valores, se sustituyen en la primera ecuacin y se despeja x :
( ) 5 1 12 6 6 1 12 6 1 6 2 = + = = + = + x x x
Por lo tanto la solucin del sistema es: 1 6 5 = = = z , y , x
Comprobacin:
( )
( ) ( )
( ) ( ) ( )

)

= + = +
= + = +
= + = +
1 2 6 5 1 2 6 5
1 1 12 10 1 6 2 5 2
6 1 12 5 1 6 2 5


3)

=
= + +
=
11 2 9
9 5 3 12
20 4 2 3
z y x
z y x
z y x

Solucin.
La primera ecuacin se multiplica por 4 y se suma a la segunda. La primera ecuacin se multiplica por
3 y se suma a la tercera:
Pgina del Colegio de Matemticas de la ENP-UNAM Sistemas de ecuaciones y de desigualdades Autor: Dr. Jos Manuel Becerra Espinosa
27

=
= +
=
49 14 7
71 21 11
20 4 2 3
z y
z y
z y x

la tercera ecuacin se divide por 7 :

=
= +
=
7 2
71 21 11
20 4 2 3
z y
z y
z y x

la tercera ecuacin se multiplica por 11 y se suma a la segunda:

=
=
=
7 2
6
20 4 2 3
z y
z
z y x

de la segunda ecuacin se despeja z : 6
1
6
=

= z
se sustituye este valor en la tercera ecuacin y se despeja y :
( ) 5
1
5
5 12 7 7 12 7 6 2 =

= = = = + = y y y y
estos valores, se sustituyen en la primera ecuacin y se despeja x :
( ) ( ) 2
3
6
6 24 10 20 3 20 24 10 3 20 6 4 5 2 3 = = = + = = + = x x x x
Por lo tanto la solucin del sistema es: 6 5 2 = = = z , y , x
Comprobacin:
( ) ( ) ( )
( ) ( ) ( )
( ) ( )

)

= + =
= + = + +
= + =
11 12 5 18 6 2 5 2 9
9 30 15 24 6 5 5 3 2 12
20 24 10 6 6 4 5 2 2 3



VIII.5.2 MTODO DE DETERMINANTES (REGLA DE CRAMER)

Dado un arreglo de nmeros de la forma:
(
(
(

33 32 31
23 22 21
13 12 11
a a a
a a a
a a a
, su determinante:

33 32 31
23 22 21
13 12 11
a a a
a a a
a a a


denotado por , es el resultado de la operacin:

12 21 33 11 23 32 13 22 31 23 12 31 13 32 21 33 22 11
a a a a a a a a a a a a a a a a a a + +

Si al determinante se le agregan los dos primeros renglones y se efectan los productos que indican las
flechas se tiene que:

Pgina del Colegio de Matemticas de la ENP-UNAM Sistemas de ecuaciones y de desigualdades Autor: Dr. Jos Manuel Becerra Espinosa
28




el determinante puede obtenerse calculando la diferencia de la suma de productos en la direccin hacia abajo
menos la suma de productos en la direccin hacia arriba. Es decir, representa el producto de nmeros que
conforman su diagonal principal (la que se dirige hacia abajo) y sus dos paralelas menos el producto de
nmeros que conforman su diagonal secundaria (la que se dirige hacia arriba) y sus dos paralelas.

Ejemplos.
Aplicando la frmula, calcular los siguientes determinantes:

1) ( )( ) ( )( ) ( )( ) ( )( ) ( )( ) ( )( ) 3 4 7 5 9 6 2 1 8 9 3 8 2 6 4 7 1 5
7 6 8
9 1 4
2 3 5
+ + =


237 84 270 16 216 48 35 = + =
2) ( )( ) ( )( )( ) ( )( ) ( )( ) ( )( ) ( )( )( ) 2 5 1 7 4 6 8 3 1 4 2 1 8 6 5 1 3 7
1 6 1
4 3 5
8 2 7
+ + =


451 10 168 24 8 240 21 = + =
3) ( )( ) ( )( )( ) ( )( ) ( )( ) ( )( ) ( )( ) 4 2 2 9 8 5 3 1 9 8 4 9 3 5 2 2 1 9
2 5 9
8 1 2
3 4 9
+ + =


71 16 360 27 288 30 18 = + + + =
4) ( )( ) ( )( ) ( )( ) ( )( )
|

\
|

|

\
|
|

\
|

|

\
|
|

\
|
+ + =

2
5
10 8
2
1
2
7
0 6 2 4
2
7
2
5
4 6 0 10 8 2
2
1
8 0 4
2
7
2 10
6
2
5
2
1

179 200 0 48 35 0 8 = + + + =

Dado un sistema de tres ecuaciones con tres incgnitas de la forma:

= + +
= + +
= + +
3 33 32 31
2 23 22 21
1 13 12 11
b z a y a x a
b z a y a x a
b z a y a x a


El determinante del Sistema es el determinante del arreglo formado por los coeficientes de las incgnitas.
23 22 21
13 12 11
33 32 31
23 22 21
13 12 11
a a a
a a a
a a a
a a a
a a a
23 22 21
13 12 11
33 32 31
23 22 21
13 12 11
a a a
a a a
a a a
a a a
a a a
Pgina del Colegio de Matemticas de la ENP-UNAM Sistemas de ecuaciones y de desigualdades Autor: Dr. Jos Manuel Becerra Espinosa
29

El determinante de cualquier incgnita es el que se obtiene sustituyendo en el arreglo del sistema la
columna de los coeficientes de esa incgnita por la columna de los trminos independientes.

La Regla de Cramer establece que dado un sistema de ecuaciones lineales cuyos trminos
independientes no son cero, el valor de cada incgnita se obtiene dividiendo el determinante de la
incgnita por el determinante del sistema. Esto es:

33 32 31
23 22 21
13 12 11
33 32 3
23 22 2
13 12 1
a a a
a a a
a a a
a a b
a a b
a a b
x
x =

= ;
33 32 31
23 22 21
13 12 11
33 3 31
23 2 21
13 1 11
a a a
a a a
a a a
a b a
a b a
a b a
y
y =

= ;
33 32 31
23 22 21
13 12 11
3 32 31
2 22 21
1 12 11
a a a
a a a
a a a
b a a
b a a
b a a
z
z =

=
Cuando el determinante es cero, entonces el sistema es incompatible.

Ejemplo.
Obtener la solucin de los siguientes sistemas aplicando la Regla de Cramer:

1)

= +
= + +
=
42 7 6 2
22 8 3 4
17 6 5 3
z y x
z y x
z y x

Solucin.
( )( ) ( )( ) ( )( ) ( )( ) ( )( ) ( )( )( ) 5 4 7 3 8 6 6 3 2 8 5 2 6 6 4 7 3 3
7 6 2
8 3 4
6 5 3
+ + =


=

535 140 144 36 80 144 63 = + =
( )( ) ( )( )( ) ( )( ) ( )( ) ( )( ) ( )( )( ) 5 22 7 17 8 6 6 3 42 8 5 42 6 6 22 7 3 17
7 6 42
8 3 22
6 5 17
+ + =


= x

535 770 816 756 680 1 792 357 = + + + = ,
1
535
535
=

=
x
x
( )( ) ( )( ) ( )( ) ( )( ) ( )( ) ( )( )( ) 17 4 7 3 8 42 6 22 2 8 17 2 6 42 4 7 22 3
7 42 2
8 22 4
6 17 3
+ + =

= y

070 1 476 008 1 264 272 008 1 462 , , , = + + =
2
535
070 1
=

=
, y
y
( )( ) ( )( ) ( )( ) ( )( ) ( )( ) ( )( )( ) 5 4 42 3 22 6 17 3 2 22 5 2 17 6 4 42 3 3
42 6 2
22 3 4
17 5 3
+ + =

= z

140 2 840 396 102 220 408 378 , = + + + + =
Pgina del Colegio de Matemticas de la ENP-UNAM Sistemas de ecuaciones y de desigualdades Autor: Dr. Jos Manuel Becerra Espinosa
30

4
535
140 2
=

=
, z
z

Por lo tanto la solucin del sistema es: 4 2 1 = = = z , y , x
Comprobacin:
( ) ( ) ( )
( ) ( ) ( )
( ) ( ) ( )

)

= + + = +
= + = + +
= + =
42 28 12 2 4 7 2 6 1 2
22 32 6 4 4 8 2 3 1 4
17 24 10 3 4 6 2 5 1 3


2)

= +
= + +
= + +
9 9 8 5
5 4 4
1 2 7 6
z y x
z y x
z y x

Solucin.
( )( ) ( )( ) ( )( )( ) ( )( )( ) ( )( ) ( )( )( ) 7 4 9 6 4 8 2 1 5 4 7 5 2 8 4 9 1 6
9 8 5
4 1 4
2 7 6
+ + =

=

60 252 192 10 140 64 54 = + + + =
( )( ) ( )( ) ( )( )( ) ( )( )( ) ( )( ) ( )( )( ) 7 5 9 1 4 8 2 1 9 4 7 9 2 8 5 9 1 1
9 8 9
4 1 5
2 7 1
+ + =

= x

120 315 32 18 252 80 9 = + + + =
2
60
120
=

=
x
x
( )( ) ( )( ) ( )( )( ) ( )( )( ) ( )( )( ) ( )( )( ) 1 4 9 6 4 9 2 5 5 4 1 5 2 9 4 9 5 6
9 9 5
4 5 4
2 1 6
+ + =

= y

60 36 216 50 20 72 270 = + + + =
1
60
60
=

=
y
y
( )( ) ( )( ) ( )( )( ) ( )( )( ) ( )( ) ( )( )( ) 7 4 9 6 5 8 1 1 5 5 7 5 1 8 4 9 1 6
9 8 5
5 1 4
1 7 6
+ + =

= z

180 252 240 5 175 32 54 = + + + =
3
60
180
=

=
z
z

Por lo tanto la solucin del sistema es: 3 1 2 = = = z , y , x

Comprobacin:
( ) ( ) ( )
( ) ( )
( ) ( ) ( )

)

= + = +
= + + = + +
= + + = + +
9 27 8 10 3 9 1 8 2 5
5 12 1 8 3 4 1 2 4
1 6 7 12 3 2 1 7 2 6

Pgina del Colegio de Matemticas de la ENP-UNAM Sistemas de ecuaciones y de desigualdades Autor: Dr. Jos Manuel Becerra Espinosa
31

3)

=
= +
= + +
20 6 2
68 2 8 3
23 5 3 2
z y x
z y x
z y x

Solucin.
( )( )( ) ( )( ) ( )( )( ) ( )( )( ) ( )( )( ) ( )( )( ) 3 3 6 2 2 2 5 8 1 2 3 1 5 2 3 6 8 2
6 2 1
2 8 3
5 3 2
+ + =

=

82 54 8 40 6 30 96 = + + =
( )( )( ) ( )( ) ( )( )( ) ( )( )( ) ( )( )( ) ( )( )( ) 3 68 6 23 2 2 5 8 20 2 3 20 5 2 68 6 8 23
6 2 20
2 8 68
5 3 23
+ + =

= x
820 224 1 92 800 120 680 104 1 = + + = , ,
10
82
820
= =

=
x
x
( )( )( ) ( )( ) ( )( )( ) ( )( )( ) ( )( )( ) ( )( )( ) 23 3 6 2 2 20 5 68 1 2 23 1 5 20 3 6 68 2
6 20 1
2 68 3
5 23 2
+ + =


= y

328 414 80 340 46 300 816 = + + =
4
82
328
= =

=
y
y
( )( )( ) ( )( ) ( )( )( ) ( )( )( ) ( )( )( ) ( )( )( ) 3 3 20 2 68 2 23 8 1 68 3 1 23 2 3 20 8 2
20 2 1
68 8 3
23 3 2
+ + =


= z

246 180 272 184 204 138 320 = + + + =
3
82
246
=

=
z
z

Por lo tanto la solucin del sistema es: 3 4 10 = = = z , y , x
Comprobacin:
( ) ( ) ( )
( ) ( ) ( )
( ) ( )

)

= + =
= + + = +
= + = + +
20 18 8 10 3 6 4 2 10
68 6 32 30 3 2 4 8 10 3
23 15 12 20 3 5 4 3 10 2



VIII.6 SISTEMAS CON UNA ECUACIN DE PRIMER GRADO Y UNA ECUACIN DE
SEGUNDO GRADO EN DOS VARIABLES

La ecuacin general de primer grado en dos variables es 0 = + + c by ax , donde c , b , a son coeficientes
reales. Geomtricamente determina una recta.

La ecuacin general de segundo grado en dos variables es 0
2 2
= + + + + + F Ey Dx Cy Bxy Ax , donde
F , E , D , C , B , A son coeficientes reales. Geomtricamente, por lo general, determina una curva.
Pgina del Colegio de Matemticas de la ENP-UNAM Sistemas de ecuaciones y de desigualdades Autor: Dr. Jos Manuel Becerra Espinosa
32

Un sistema de una ecuacin de primer grado y una ecuacin de segundo grado en dos variables es de la
forma:

)
`

= + + + + +
= + +
0
0
2 2
F Ey Dx Cy Bxy Ax
c by ax


Algebraicamente, el procedimiento general consiste en despejar de ambas ecuaciones la misma variable,
igualar con el objeto de resolver para la otra. Una vez obtenido cada valor, se sustituye en cualquiera de
las ecuaciones despejadas para encontrar su pareja correspondiente.

Grficamente, su solucin est dada por la interseccin de las grficas. Se pueden tener tres casos:

Si la recta corta a la curva, lo hace dos veces por lo que se tendrn dos puntos de interseccin

Si la recta es tangente a la curva, entonces slo se tendr un punto solucin.

Si la recta no corta a la curva, no tiene soluciones reales.

Ejemplos.
Resolver los siguientes sistemas de ecuaciones:

1)
)
`

= + +
= +
0 4 2 12 2
0 6 2 4
2
y x x
y x


Solucin:
Despejando y de la primera ecuacin:
3 2
2
6 4
+ =


= x
x
y
despejando y de la segunda ecuacin:
2 6
2
4 12 2
2
2
+ =
+
= x x
x x
y
igualando ambas expresiones:
0 5 4 2 6 3 2
2 2
= + + = + x x x x x
resolviendo por factorizacin se tiene:
( )( ) 0 1 5 = + x x
5 0 5
1
= = + x x
1 0 1
2
= = x x
evaluando en la primera ecuacin despejada:
( ) 7 3 10 3 5 2
1
= + = + = y
( ) 5 3 2 3 1 2
2
= + = + = y

Por lo tanto, la solucin son los puntos ( ) 7 5 , y ( ) 5 1,

Tabulacin:





Pgina del Colegio de Matemticas de la ENP-UNAM Sistemas de ecuaciones y de desigualdades Autor: Dr. Jos Manuel Becerra Espinosa
33

-10
y
-5
5
5 10 -5 -10 x
( ) 7 5 ,
( ) 5 1,


x 3 2 + = x y
2 6
2
+ = x x y
-9 -15 25
-8 -13 14
-7 -11 5
-6 -9 -2
-5 -7 -7
-4 -5 -10
-3 -3 -11
-2 -1 -10
-1 1 -7
0 3 -2
1 5 5
2 7 14
3 9 25




2)
)
`

= +
= +
0 75 3 3
0 4 2 8
2 2
y x
y x


Solucin:
Despejando y de la primera ecuacin:
2 4
2
4 8
+ =
+
= x
x
y
despejando y de la segunda ecuacin:
2
2
25
3
3 75
x
x
y =

=
igualando ambas expresiones:
2
25 2 4 x x = +
elevando al cuadrado:
( ) ( )
2
2 2
25 2 4 x x = +
2 2
25 4 16 16 x x x = + +
0 21 16 17
2
= + x x
21 16 17 = = = c , b , a
sustituyendo en la frmula general de segundo grado:
( )( )
( ) 34
428 1 256 16
17 2
21 17 4 16 16
2
,
x
+
=

=
17
421 8
34
421 2 16
34
684 1 16 +
=

=

=
,

7363 0
17
421 8
1
. x
+
=
6775 1
17
421 8
2
. x

=
evaluando en la primera ecuacin despejada:
Pgina del Colegio de Matemticas de la ENP-UNAM Sistemas de ecuaciones y de desigualdades Autor: Dr. Jos Manuel Becerra Espinosa
34

-10
y
-5
5
5 10 -5 -10 x
10
( ) 9452 4 7363 0 . , .
( ) 71 4 6775 1 . , .
( ) 9452 4 2 7363 0 4 2
17
421 8
4
1
. . y + +
|
|

\
|
+
=
( ) 71 4 2 6775 1 4 2
17
421 8
4
2
. . y + +
|
|

\
|

=
Por lo tanto, la solucin son los puntos:
|
|

\
|
+
|
|

\
|
+ +
2
17
421 8
4
17
421 8
, y
|
|

\
|
+
|
|

\
|

2
17
421 8
4
17
421 8
,
que aproximadamente son:
( ) 9452 4 7363 0 . , . y ( ) 71 4 6775 1 . , .
Tabulacin:

x 2 4 + = x y
2
25 x y =
-5 -18 0
-4 -14 3
-3 -10 4
-2 -6 4.5825
-1 -2 4.8989
0 2 5
1 6 4.8989
2 10 4.5825
3 14 4
4 18 3
5 22 0

3)
)
`

=
=
0 4
0 4 2 2
2 2
y x
y x


Solucin:
Despejando y de la primera ecuacin:
2
2
4 2
=

+
= x
x
y
despejando y de la segunda ecuacin:
4 4
1
4
4
2 2 2
2
2 2 2
= =

+
= + = x y x y
x
y x y
igualando ambas expresiones:
4 2
2
= x x
elevando al cuadrado:
( ) ( )
2
2 2
4 2 = x x
4 4 4
2 2
= + x x x
4 4 4 = + x
8 4 = x
2
4
8
=

= x
Pgina del Colegio de Matemticas de la ENP-UNAM Sistemas de ecuaciones y de desigualdades Autor: Dr. Jos Manuel Becerra Espinosa
35

evaluando en la primera ecuacin despejada:
0 2 2 = = y
Por lo tanto, la solucin es el punto ( ) 0 2, .
Tabulacin:

x 2 = x y
4
2
= x y
-10 -12 9.7979
-9 -11 8.7749
-8 -10 7.7459
-7 -9 6.7082
-6 -8 5.6568
-5 -7 4.5825
-4 -6 3.4641
-3 -5 2.2360
-2 -4 0
-1 -3 No definido
0 -2 No definido
1 -1 No definido
2 0 0
3 1 2.2360
4 2 3.4641
5 3 4.5825
6 4 5.6568
7 5 6.7082
8 6 7.7459
9 7 8.7749
10 8 9.7979

4)
)
`

= +
= + +
0 36 9 4
0 12 3 6
2 2
y x
y x


Solucin:
Despejando y de la primera ecuacin:
4 2
3
12 6
=

= x
x
y
despejando y de la segunda ecuacin:
9
4 36
2
x
y

=
igualando ambas expresiones:
9
4 36
4 2
2
x
x

=
elevando al cuadrado:
( )
2
2
2
9
4 36
4 2
|
|

\
|

=
x
x
9
4 36
16 16 4
2
2
x
x x

= + +
2 2
4 36 144 144 36 x x x = + +
0 108 144 40
2
= + + x x
0 27 36 10
2
= + + x x
-10
y
-5
5
7 10 -7 -10 x
10
( ) 0 2,
Pgina del Colegio de Matemticas de la ENP-UNAM Sistemas de ecuaciones y de desigualdades Autor: Dr. Jos Manuel Becerra Espinosa
36

-5
y
-3
3
3 5 -3 -5 x
5
-1 1
1
( ) 8698 1 0651 1 . , .
( ) 0696 1 5348 2 . , .
27 36 10 = = = c , b , a
sustituyendo en la frmula general de segundo grado:
( )( )
( ) 20
080 1 296 1 36
10 2
27 10 4 36 36
2
, ,
x

=

=
10
6 3 18
20
6 6 36
20
216 36
=

=

=
0651 1
10
6 3 18
1
. x
+
=
5348 2
10
6 3 18
2
. x

=
evaluando en la primera ecuacin despejada:
( ) 8698 1 4 0651 1 2 4
10
6 3 18
2
1
. . y
|
|

\
|
+
=
( ) 0696 1 4 5348 2 2 4
10
6 3 18
2
2
. . y
|
|

\
|

=
Por lo tanto, la solucin son los puntos:
|
|

\
|

|
|

\
|
+

+
4
10
6 3 18
2
10
6 3 18
, y
|
|

\
|

|
|

\
|


4
10
6 3 18
2
10
6 3 18
,
que aproximadamente son:
( ) 8698 1 0651 1 . , . y ( ) 0696 1 5348 2 . , .
Tabulacin:
x 4 2 = x y
9
4 36
2
x
y

=
-4 4 No definido
-3 2 0
-2 0 1.4907
-1 -2 1.8856
0 -4 2
1 -6 1.8856
2 -8 1.4907
3 -10 0
4 -12 No definido

5)
)
`

= +
=
0 156 48 12
0 25 5 10
2
x y
y x


Solucin:
Despejando y de la primera ecuacin:
5 2
5
25 10
=

+
= x
x
y
despejando y de la segunda ecuacin:
Pgina del Colegio de Matemticas de la ENP-UNAM Sistemas de ecuaciones y de desigualdades Autor: Dr. Jos Manuel Becerra Espinosa
37

-10
y
-5
5
5 10 -5 -10 x
10
( ) 1 3,
( ) 3 1 ,
x
x
y 4 13
12
48 156
=

=
igualando ambas expresiones:
x x 4 13 5 2 =
elevando al cuadrado:
( ) ( )
2
2
4 13 5 2 x x =
x x x 4 13 25 20 4
2
= +
0 12 16 4
2
= + x x
dividiendo por 4 :
0 3 4
2
= + x x
factorizando se tiene:
( )( ) 0 1 3 = x x
3 0 3
1
= = x x
1 0 1
2
= = x x
evaluando en la primera ecuacin despejada:
( ) 1 5 6 5 3 2
1
= = = y
( ) 3 5 2 5 1 2
2
= = = y
Por lo tanto, la solucin son los puntos ( ) 1 3, y ( ) 3 1 ,

Tabulacin:
x 5 2 = x y
x y 4 13 =
-10 -25 7.2801
-9 -23 7
-8 -21 6.7082
-7 -19 6.4031
-6 -17 6.0827
-5 -15 5.7445
-4 -13 5.3851
-3 -11 5
-2 -9 4.5825
-1 -7 4.1231
0 -5 3.6055
1 -3 3
2 -1 2.230
3 1 1
4 3 No definido
5 5 No definido
6 7 No definido
7 9 No definido
8 11 No definido
9 13 No definido
10 15 No definido

6)
)
`

= +
= +
0 9
0 10 2
2 2
y x
y x


Solucin:
Despejando y de la primera ecuacin:

2
10 +
=
x
y

Pgina del Colegio de Matemticas de la ENP-UNAM Sistemas de ecuaciones y de desigualdades Autor: Dr. Jos Manuel Becerra Espinosa
38

-10
y
-5
5
5 10 -5 -10 x
10
despejando y de la segunda ecuacin:
2
9 x y =
igualando ambas expresiones:
2
9
2
10
x
x
=
+

elevando al cuadrado:
( )
2
2
2
9
2
10
x
x
=
|

\
| +

2
2
9
4
100 20
x
x x
=
+ +

2 2
4 36 100 20 x x x = + +
0 64 20 5
2
= + + x x
resolviendo por frmula general:
64 20 5 = = = c , b , a
Sustituyendo en la frmula general se tiene:
( )( )
( ) 10
880 20
10
280 1 400 20
5 2
64 5 4 20 20
2

=

=

=
,
x
Como el discriminante es negativo, no hay solucin en los nmeros reales.
Tabulacin:
x
2
10 +
=
x
y


2
9 x y =

-10 0 No definido
-9 0.5 No definido
-8 1 No definido
-7 1.5 No definido
-6 2 No definido
-5 2.5 No definido
-4 3 No definido
-3 3.5 0
-2 4 2.2360
-1 4.5 2.8284
0 5 3
1 5.5 2.8284
2 6 2.2360
3 6.5 0
4 7 No definido
5 7.5 No definido
6 8 No definido
7 8.5 No definido
8 9 No definido
9 9.5 No definido
10 10 No definido

Вам также может понравиться